2019 PREP Questions

Pataasin ang iyong marka sa homework at exams ngayon gamit ang Quizwiz!

A 16-year-old boy is playing hockey on a frozen pond when he falls through the ice and rapidly submerges. Emergency medical services are able to locate the victim after 10 minutes and he is brought to the emergency department where he is noted to be in cardiac arrest. His initial core temperature is 24oC. Cardiac rhythm demonstrates asystole. Resuscitation is continued. Of the following, the factor that correlates MOST strongly with the outcome is: A. duration of submersion B. initial cardiac rhythm C. initial core body temperature D. temperature of water

A. duration of submersion Drowning is defined as submersion in a liquid, resulting in death within 24 hours of the event. Use of the term "dry," "near," or "wet" drowning is no longer advocated. The World Health Organization states that 0.7% of all deaths worldwide are due to drowning. In the United States, drowning is the second leading cause of death for children ages 1-4 years. A number of factors are associated with drowning. These include male sex, age less than 14 years, alcohol use, lack of supervision, warm weather months, lack of swimming education, epilepsy, and lower income. Following submersion, many victims initially hold their breath. Eventually, a gasp occurs resulting in water entering the larynx. This results in a coughing reflex and in some cases laryngospasm. Due to worsening hypoxia, the victim will lose consciousness and continues to aspirate liquid. Worsening hypoxemia induces tachycardia, followed by bradycardia, then pulseless electrical activity and eventually asystole. A submersion victim's outcome is often predicated by the degree of hypoxic-ischemic CNS injury. The risk of neurologic injury may be influenced by a number of factors. Numerous case reports document excellent neurologic outcomes following prolonged submersion in ice water. It was believed that rapid onset of profound hypothermia may provide some degree of neurologic protection. Hypothermia reduces CNS metabolic activity in a temperature dependent fashion, ~5% for each degree of centigrade reduction. Children were believed to cool more rapidly than adults in ice water due to their higher surface area to body weight ratio, as well as their lower subcutaneous fat. The proposed neuro-protective mechanism in ice water drownings was believed to be due to decreasing CNS oxygen consumption faster than declining oxygen delivery. However, most cases of hypothermia in drowning victims are not from ice water exposure. Rather, the low initial body temperature reflects prolonged submersion time and lack of cardiac activity. Investigators have examined prognostic factors influencing outcomes in pediatric drownings. Numerous studies show that submersion time is the single most important predictor of outcome. In warm water, submersion times longer than ~10 minutes have a poor prognosis. Duration of submersion, however, is never precise and subject to recall bias. A number of investigators have reviewed the association of water temperature and neurologic outcome. A recent study from a Swedish fire and rescue database showed no difference in outcome between water temperatures of <60C versus 150C. A northern European study evaluating 47 cold-water submersion victims showed no correlation between water temperature, initial rectal temperature, and long-term survival. Water temperatures in this study ranged from 0-330C. Additionally, children did not have better outcomes than adults. Of all factors evaluated, the investigators showed that submersion time was most predictive of survival. In the United States, Quan and colleagues performed a 20-year retrospective analysis on the association of water temperature and outcome. They evaluated data from 1,094 submersion victims from warm (>170C), cold (6-160C), or very cold (<60C) water. Again, no association was found between good outcome and water temperature. Multivariate analysis showed submersion times <6 minutes to be the factor most associated with good outcome. The authors speculate that central hypothermia would require immersion in cold water for 30 minutes, by which time the victim would have sustained significant hypoxic-ischemic CNS injury. Related to ice water submersion, hypothermia at time of presentation to medical attention has been examined. In a 10-year study from the Netherlands assessing hypothermia following drowning in 160 children, investigators noted no protective effect. Patients were included if core body temperature was <340C. Of all patients, 72% died in the emergency department, pediatric intensive care unit, or within one year of the event. About 22% survived over 1 year, but had a poor neurologic outcome. Only 6% had a good neurologic outcome that correlated strongly with shorter duration of resuscitation rather than initial body temperature. PREP Pearls Ice water drowning was historically believed to be neuro-protective. Recent data suggests no association between good clinical outcome and water temperature or hypothermia at medical presentation. Duration of submersion is the factor that correlates most consistently with good versus poor neurologic outcome. ABP Content Specifications(s)/Content Area Know the importance of water temperature in brain outcome from a submersion episode Suggested Readings Kieboom JK, Verkade HJ, Burgerhof JG, et al. Outcome after resuscitation beyond 30 minutes in drowned children with cardiac arrest and hypothermia: Dutch nationwide retrospective cohort study. BMJ. 2015;350:418-428. doi: 10.1136/bmj.h418 Quan L, Mack CD, Schiff MA. Association of water temperature and submersion duration and drowning outcome. Resuscitation. 2014;85(6):790-794. doi: 10.1016/j.resuscitation.2014.02.024 Suominen P, Baillie C, Korpela R, Rautanen S, Ranta S, Olkkola KT. Impact of age, submersion time and water temperature on outcome in near-drowning. Resuscitation. 2002; 52(3):247-254. Szpilman D, Bierens JL, Handley AJ, Orlowski JP. Drowning. N Engl J Med. 2012;366(22): 2102-2110. doi: 10.1056/NEJMra1013317

A 15-year-old adolescent boy is brought to the emergency department with altered mental status, mottled skin appearance, and hypotension. Past medical history reveals that he had a splenectomy at age 5 and his immunization status and prophylaxis status are unknown. On physical examination he is lethargic with a heart rate of 165 beats/min, blood pressure of 83/32 mm Hg, respiratory rate of 12 breaths/min, and temperature of 39.9°C. It is difficult to obtain pulse oximetry reading because his fingers are mottled and appear blackened in some areas. Mechanical ventilation is initiated, blood cultures drawn, and broad-spectrum antibiotics administered. Of the following, the microbe that is MOST likely to be isolated from the blood culture will be: A. Escherichia coli B. Pseudomonas aeruginosa C. Staphylococcus aureus D. Streptococcus pneumoniae

D. Streptococcus pneumoniae The patient in the vignette has clinical features of sepsis with multiple organ dysfunction syndrome (MODS) probably caused by a bacterial pathogen. In patients with splenectomy, the most likely bacterial pathogen causing bacteremia with sepsis and MODS is Streptococcus pneumoniae. Other encapsulated organisms such as Haemophilus influenzae type b and Neisseria meningitidis may also cause infections resulting in a similar clinical presentation, however, they are less common than S pneumoniae. Neisseria meningitidis is more common in patients with defects in the terminal components of complement. The spleen, located in the left upper quadrant of the abdomen along the 9th, 10th, and 11th rib, has a number of important physiological functions. The spleen is an integral part of the lymphoid system and an important component of the immune system. Histologically, the spleen consists of red pulp and white pulp. The red pulp consists of sinusoids where red blood cells (RBCs) are stored. The red pulp acts as a reservoir for blood that may be released in the setting of hemorrhagic shock in order to boost the intravascular blood volume. The red pulp also removes senescent RBCs from the circulation. When these cells are destroyed, the iron is stored in the spleen in the form of bilirubin and ferritin. This iron may be utilized by the bone marrow to form new RBCs. The white pulp consists of white blood cells and serves immunologic function analogous to lymph nodes. Among the cells in the white pulp are the antigen processing cells, which assist in the process of phagocytosis through opsonization. Opsonization is the process by which antibodies attach to foreign objects including pathogens in order to facilitate phagocytosis by macrophages. This process occurs in the spleen and is particularly important for destruction of encapsulated bacteria such as S pneumoniae. The spleen is also the major site for the early production of IgM following an infection by any pathogen. This helps clear any infection from the bloodstream promptly. Splenic function can be absent due to anatomical absence of the spleen or functional asplenia. Functional asplenia (hyposplenism) may be seen in association with hematological diseases such as sickle cell disease or thalassemias. Because one of the functions of the spleen is to filter abnormal RBCs; excess numbers of RBCs are filtered by the spleen in patients with sickle cell disease and thalassemias. In the case of sickle cell anemia, sickling of the RBCs inside the spleen can lead to multiple areas of infarction, a process that eventually leads to functional asplenia. Patients who have absent splenic functions are susceptible to infections with encapsulated bacterial pathogens such as S pneumoniae, H influenzae, and N meningitidis. Streptococcus pneumoniae accounts for about 50% of infections in patients with anatomic or functional asplenia with a mortality rate of around 60%. Vaccination prior to splenectomy may lower the mortality rate due to overwhelming infection by this pathogen. Patients with anatomical and/or functional asplenia are also at risk for more serious infections by organisms that live within the RBCs. These pathogens include Plasmodium falciparum (one of the causative agents for malaria) and Babesia microti (the causative agent for Babesiosis). Both of these pathogens cause an acute febrile illness with multiple organ involvement. Because both of these pathogens live within RBCs and one of the functions of the normal spleen is to filter abnormal RBCs, patients who have asplenia tend to have more severe forms of malaria and babesiosis. Overall, patients with asplenia account for one-third of all cases of Babesiosis. Another pathogen that has been identified to cause overwhelming infection with sepsis in patients with asplenia is Capnocytophaga canimorsus. The organism is a gram-negative bacteria that is normally present in canine saliva. The infection is usually acquired following a dog bite and can progress to overwhelming sepsis. The organism is able to avoid phagocytosis in patients with asplenia which leads to systemic inflammatory response syndrome with severe sepsis, with significant morbidity and mortality. Patients who have or are expected to have anatomical or functional asplenia should receive immunization against S pneumoniae, H influenzae, and N meningitidis. Influenza vaccine is also recommended prior to each season. These patients should also receive daily penicillin prophylaxis against S pneumoniae until they reach adulthood. Patients with asplenia and their families should be educated about their clinical condition and the lack of adequate immune system to fight infections. Clinicians should err on the side of caution when evaluating patients with asplenia. These patients should be evaluated promptly for fever and broad-spectrum antimicrobial agents should be initiated after appropriate samples for cultures are collected. PREP Pearls Encapsulated organisms such as Streptococcus pneumoniae and Haemophilus influenzae are common infective agents in patients with anatomical or functional asplenia. Babesiosis and malaria produce more severe disease in patients with asplenia. Capnocytophaga canimorsus causes overwhelming sepsis in patients with asplenia following a dog bite. ABP Content Specifications(s)/Content Area Know the functions of the spleen Know the clinical and laboratory manifestations of altered/absent splenic function Suggested Readings Di Sabatino A, Lenti MV, Tinozzi FP, et al. Vaccination coverage and mortality after splenectomy: results from an Italian single-center study. Intern Emerg Med. 2017;12(8):1139-1147. doi: 10.1007/s11739-017-1730-9 Morgan TL, Tomich EB. Overwhelming post-splenectomy infection (OPSI): A case report and review of the literature. J Emerg Med. 2012;43(4):758-763. doi: 10.1016/j.jemermed.2011.10.029 Sucandy I, Polavarapu HV, Pezzi CM. Hypoplasia of the spleen: review of pathogenesis, diagnosis, and potential clinical implications. N Am J Med Sci. 2015;7(8):368-370. doi: 10.4103/1947-2714.163645

A child with severe sepsis is being treated and there is a question to the effectiveness of a new drug in reversing organ dysfunction. In the literature search, there is a recently published randomized controlled trial comparing the new drug to placebo. In this study, 100 children received the drug and 100 received the placebo. Among the 80 children who had multiple organ dysfunction at day 7, 50 had received placebo and 30 had received the new drug. Based on the results of the study, the absolute risk reduction of multiple organ dysfunction associated with the new drug is: A. 0.20 B. 0.60 C. 0.80 D. 1.66

A. 0.20 When evaluating the effectiveness of an intervention, we are interested in the direction of the response (ie, did the outcome of interest increase or decrease with the intervention) and the magnitude of the response (ie, how much of an increase or decrease occurred with the intervention). The direction and magnitude of response can be expressed in relative terms or absolute terms. Relative measures, such as relative risk, express the response using ratios. The relative risk is the rate of an outcome among the patients who received the intervention divided by the rate of the outcome among patients who did not. Using the example in the vignette, we can create a 2-by-2 table as shown. The rate of multiple organ dysfunction among those who received the drug is 30 of 100 patients, whereas the rate among those who received placebo is 50 of 100 patients. The relative risk of developing multiple organ dysfunction for this study would be: Relative Risk = (30/100)/(50/100) = 0.60 The interpretation for this relative risk would be, among the children who received the interventional drug, the risk is reduced by 0.40. The absolute measures use differences in absolute rates. The absolute risk reduction (ARR), also called risk difference, is the difference in rate of outcome between those who received the intervention and those who did not. Therefore, for the study in the vignette, the ARR would be: ARR = (50/100) - (30/100) = 20/100 In other words, for every 100 patients at risk, there would be 20 fewer patients developing multiple organ dysfunction due to the intervention. The number needed to treat (NNT), the inverse of the ARR, is another way of expressing this effect. In this case, the NNT would be 100/20; therefore, you would need to treat 5 patients with the intervention to prevent one case of multiple organ dysfunction. The ARR can be more useful than the relative risk for assessing the overall impact of an intervention on a population because it is expressed as a rate. For example, if we changed the study results such that the rate of multiple organ failure among children receiving the intervention drug was 30 in 100,000 subjects as compared 50 in 100,000 children who received placebo, the relative risk would be the same. However, the ARR would be 20/100,000, and the number needed to treat becomes 5,000. PREP Pearls Absolute risk reduction (also known as risk difference) is the difference in outcome between the 2 treatment groups (ie, intervention vs placebo). The inverse of the absolute risk reduction is the number of needed to treat. ABP Content Specifications(s)/Content Area Calculate absolute risk reduction Suggested Readings Schechtman E. Odds ratio, relative risk, absolute risk reduction and the number needed to treat - which of these should we use? Value Health. 2002;5(5):431-436. doi: 10.1046/J.1524-4733.2002.55150.x

A 5-year-old boy is admitted with respiratory distress secondary to influenza infection. Despite use of noninvasive ventilation strategies, his hypercapnia worsens and he is electively intubated with a 5.0 uncuffed endotracheal tube. On the fifth day following intubation, he develops worsening hypoxia requiring significant increases in his oxygen support and positive end expiratory pressure. Concurrently, he develops a fever and tachycardia, an elevation of his white cell count to 21,000/mm3, and an increase in his endotracheal secretions. His chest radiograph demonstrates new findings in Figure 1. Of the following, the MOST likely to decrease the incidence of this condition is: A. early tracheostomy B. empiric antibiotics beginning on ventilator day 3 C. oral chlorhexidine rinse daily D. use of cuffed endotracheal tube

D. use of cuffed endotracheal tube This vignette demonstrates a typical presentation for a ventilator-associated pneumonia. Hospital-acquired infections are a major challenge to health care systems. The Centers for Disease Control and Prevention (CDC) estimates there are more than 1.7 million hospital-acquired infections leading to ~90,000 deaths/year in the United States. Ventilator-associated pneumonia is a common nosocomial event within the pediatric intensive care unit (PICU) environment. By definition, ventilator-associated pneumonia occurs in patients receiving mechanical ventilatory support, is not present at the time of intubation, and develops after initiation of ventilator support. The National Nosocomial Infections Surveillance (NNIS) system reported a ventilator-associated pneumonia incidence of 2.1 per 1,000-ventilator days in United States PICUs from 1999-2008. For many years physicians caring for children had no reproducible, accurate, and universally agreed upon definition of ventilator-associated pneumonia. While the CDC initially had diagnostic criteria for adult ventilator-associated pneumonia, it did not for children. The adult criteria required at least 2 serial radiographs showing a new, progressive, or persistent infiltrate in a patient that has been mechanically ventilated for more than 48 hours. In 2007, the CDC revised its guideline to state that utilizing a 48-hour postintubation guideline may delay treatment. It therefore amended its definition of ventilator-associated pneumonia to state that any radiographic finding following intubation, regardless of time, is consistent with a ventilator-associated pneumonia. In 2014, the CDC added the term "ventilator-associated events." It suggested that ventilator-associated events should be used to describe any number of serious conditions that develop in the mechanically ventilated patient including pneumonia, acute respiratory distress syndrome, pneumothorax, pulmonary embolism, pulmonary edema, and atelectasis. Agreement on a specific definition of ventilator-associated pneumonia in the pediatric population has been difficult, with most investigators citing the need for radiographic, clinical, and laboratory criteria. According to the latest CDC definition, radiographic evidence of pneumonia is defined as 1-2 (depending on age) serial chest radiographs showing new, progressive, or persistent infiltrate, consolidation, cavitation, or pneumatoceles. Additionally, the CDC recommends patients <1 year of age demonstrate worsening gas exchange plus have at least 3 of the following: temperature instability, bradycardia, tachycardia, cough, wheezing, rales, rhonchi, apnea, tachypnea, chest wall retractions, leukopenia leukocytosis, or changes in the character or quantity of endotracheal secretions. For children >1 year of age, worsening gas exchange can be present but is not a requirement (Figure 2 ). Sadly, these clinical criteria have very poor sensitivity and specificity as they can all be seen in sepsis, acute respiratory distress syndrome, or systemic inflammatory response syndrome. The CDC acknowledges this issue, stating: "VAE definitions were designed for adult patients. More data are needed to inform whether and how VAE can be adapted for children and neonates." Recently, the Pediatric Ventilator-Associated Pneumonia Surveillance Definition Working Group proposed a new definition criteria. They recommend use of the term "ventilator-associated condition" to describe any mechanically ventilated patient who develops a sustained period of respiratory deterioration. If a patient then develops evidence of infection requiring use of antibiotics, he would then be classified as a pediatric ventilator-associated condition with antimicrobial use. Antimicrobial ventilator-associated condition with confirmed pulmonary infection would be classified as pediatric ventilator-associated pneumonia. The implementation of strategies to reduce ventilator-associated pneumonia is a major goal of multiple national organizations such as the Children's Hospital Association, Institute for Healthcare Improvement, Centers for Disease Control and the Joint Commission on the Accreditation of Healthcare Organizations. Rather than rely on antibiotic therapy after development of ventilator-associated pneumonia, attention should be focused on ventilator-associated pneumonia prevention. Knowledge of proper infection control measures, education of ICU staff, and incorporating institutional specific data based on local flora and resistance patterns results in a dynamic and far more effective approach to the management of ventilator-associated pneumonia. The optimal preventive strategies are unclear at present, but are the subject of much research. The CDC have published clinical practice guidelines for the reduction of nosocomial pneumonias. Initial pediatric guidelines were based on adult data and recommendations. Fortunately, research into ventilator-associated pneumonia has been robust and in 2014 the CDC revised its pediatric recommendations. Hand washing, with either soap or alcohol-based hand rubs, is the single most effective measure to decrease the spread of nosocomial infections. Although health care workers realize the importance of hand washing/disinfection, their compliance is still low. The compliance rate is lowest in activities that carry the highest risk for pathogen transmission, such as an emergent procedures and intubation. Higher patient workloads also decrease compliance with hand washing. Bedside hand antiseptics, easy access to sinks, decreased workloads, improved communication, staff education, and constructive feedback all improve compliance with this simple health care measure. Patient positioning is an important preventative measure in ventilator-associated pneumonia. The PICU patients are often placed in a supine position for extended periods of time. The prolonged supine positioning decreases functional residual capacity and increases atelectasis. Prolonged immobility impairs mucociliary clearance thereby allowing accumulations of pulmonary secretions in dependent portions of the lung. Emphasis has recently been placed on decreasing the aspiration of gastric contents or subglottic secretions as a means to prevent ventilator-associated pneumonia. Cuffed endotracheal tubes are now recommended. Additionally, subglottic secretion drainage is a newer strategy designed to reduce this risk. Commercially available endotracheal tubes have been designed with a suction port on the dorsum of the tube, above the cuff. Its use however is somewhat limited in the pediatric population given the smaller endotracheal tube sizes. The use of a single measure will likely result in limited improvement in ventilator-associated pneumonia incidence. Adoption of a group or "bundle" of practices by the physicians, nurses, and respiratory therapists caring for intubated patients is far more effective. The CDC recommended practices now include the following: Avoid intubation when possible and encourage the use of noninvasive ventilator support Minimize duration of mechanical ventilation Provide regular oral care Change ventilator circuits when grossly contaminated or malfunctioning, but remove condensate from the circuit frequently Elevate head of bed to 30°-45o Use cuffed endotracheal tubes Assess readiness to extubate on daily basis Other practices with evidence of benefit in adults but with limited pediatric data include daily interruption of sedation, use of probiotics, and consideration of endotracheal tubes with subglottic drainage ports. Routine practices that are not recommended by the CDC for ventilator-associated pneumonia prevention include use of oral chlorhexidine, stress ulcer prophylaxis, early tracheostomy, thromboembolism prophylaxis, antimicrobial impregnated endotracheal tubes, and systemic antibiotics for tracheal colonization. PREP Pearls The terminology used to describe ventilator-associated pneumonia is changing to include terms such as ventilator-associated event, ventilator-associated condition, and ventilator-associated condition with antibiotic use. Cuffed endotracheal tubes are now recommended as part of a ventilator-associated pneumonia prevention bundle. ABP Content Specifications(s)/Content Area Know the pathogenesis of ventilator-associated pneumonia in a patient receiving mechanical ventilation Know how to diagnose ventilator-associated pneumonia in a patient receiving mechanical ventilation Plan appropriate therapy for a patient receiving mechanical ventilation who has suspected ventilator-associated pneumonia Know the strategies to decrease the incidence of ventilator-associated pneumonia in a patient receiving mechanical ventilation Suggested Readings Chang I, Schibler A. Ventilator associated pneumonia in children. Paediatric Respir Rev. 2016;20:10-16. doi: 10.1016/j.prrv.2015.09.005. Klompas M, Branson R, Eichenwald EC, et al. Strategies to prevent ventilator-associated pneumonia in acute care hospitals: 2014 update. Infect Control Hosp Epidemiol. 2014;35(8): 915-936. doi: 10.1086/677144. Mourani PM, Sontag MK. Ventilator-associated pneumonia in critically ill children: a new paradigm. Pediatr Clin North Am. 2017;64(5):1039-1056. doi: 10.1016/j.pcl.2017.06.005. Venkatachalam V, Hendley JO, and Willson DF. The diagnostic dilemma of ventilator-associated pneumonia in critically ill children. Pediatr Crit Care Med. 2011;12(3):286-296. Ventilator-Associated Event (VAE). Centers for Disease Control and Prevention. https://www.cdc.gov/nhsn/pdfs/pscmanual/10-vae_final.pdf.

An 8-year old boy presents with altered mental status and fever over the past 24 hours He is confused, has blurry vision, and difficulty walking. Mental status is waxing and waning with generalized irritability, motor withdrawal to painful stimuli, spontaneous uttering of nonsensical speech, and eye opening to painful stimuli. He has an acute generalized tonic-clonic seizure with accompanying hypoxia lasting 10 minutes that resolves with benzodiazepine administration. Levetiracetam is begun. His deteriorating mental status and seizure results in intubation for airway protection. Previous history reveals a nonspecific viral upper respiratory infection 2 weeks ago. Neurologic examination reveals normal spinal reflexes, asymmetric facial nerve palsy, reactive pupils, intact corneal, gag, cough reflexes, and normal lower extremity tone. His asymmetric cranial nerve findings prompt a noncontrast computed axial tomography scan that shows no acute hemorrhage or mass effect. Initial laboratory data included a complete blood cell count with leukocytosis and lymphocytic predominance, elevated C-reactive protein of 8 mg/dL, and erythrocyte sedimentation rate of 12 mm/h. Cerebral spinal fluid obtained by lumbar puncture is clear and colorless with 4 white blood cells per high power field, glucose 71 mg/dL, and protein of 511 mg/dL. Blood, urine, respiratory cultures, and encephalitic viral pathogen panels are pending. A urine toxicology screen is negative. The child is started on broad spectrum antimicrobials including acyclovir and admitted to the pediatric intensive care unit. Brain and spine MRI are completed with T2 and fluid-attenuated inversion recovery imaging that reveals bilateral symmetric basal ganglia grey matter lesions and multifocal asymmetric supratentorial white matter lesions of varying size sparing the periventricular spaces. Of the following, the next MOST appropriate intervention should be: A. administer a loading dose of fosphenytoin B. administer high-dose methylprednisolone C. consent the family for placement of a central line for plasma exchange therapy D. order a STAT transesophageal echocardiogram to rule out endocarditis

B. administer high-dose methylprednisolone In the described case, this child has acute disseminated encephalomyelitis. The incidence of acute disseminated encephalomyelitis has been reported to be 0.4-2/100,000 children per year with a slight male predominance and seasonal distribution (winter and spring). Often a preceding infectious process, viral or bacterial, is associated with acute disseminated encephalomyelitis, also known as postinfectious encephalomyelitis. Common viruses associated with acute disseminated encephalomyelitis include cytomegalovirus, Epstein-Barr, influenza, measles, rubella, varicella zoster, herpes simplex, coxsackie, and coronavirus. Other infectious agents include chlamydia, mycoplasma pneumoniae, Borellia burgdorferi, and beta hemolytic streptococcus. Postvaccination acute disseminated encephalomyelitis has been described and is most common following measles, mumps, and rubella vaccination with an incidence of 1-2/1 million children. The pathophysiology of acute disseminated encephalomyelitis is not completely understood. It has been proposed that acute disseminated encephalomyelitis is an autoimmune disorder resulting in T-cell activation to automyelin antigens within the CNS in genetically susceptible individuals. Diagnosing acute disseminated encephalomyelitis relies on clinical and radiographic features that include: (1) polyfocal neurologic abnormality with encephalopathy and an inflammatory demyelinating cause, (2) encephalopathy that cannot be explained by fever, systemic illness, or seizure, (3) abnormal neuroimaging findings during acute illness, and (4) no emergence of new clinical symptoms >3 months from onset. The differential diagnosis includes meningitis, encephalitis, multiple sclerosis, optic neuritis, transverse myelitis, and neuromyelitis optica. Children with acute disseminated encephalomyelitis typically have a history of a febrile illness within 4 weeks before onset of neurologic symptoms, but more commonly within 2 weeks of an infectious process or immunization. Acute presentation includes fever, headache, vomiting, and meningismus that rapidly progresses to encephalopathy (ie, irritability, confusion, lethargy, coma) with multi-focal neurologic symptoms (ie, pyramidal signs, acute hemiparesis, ataxia, optic neuritis, spinal cord dysfunction) peaking within 1 week. Seizures during acute presentation are also common occurring in nearly one third of children with acute disseminated encephalomyelitis. Less commonly, children have aphasia, sensory deficits, or movement disorders. Acute hemorrhagic variants of acute disseminated encephalomyelitis are rare and associated with a worse clinical outcome. The preferred neuroimaging modality is brain and spine MRI with T2-weighted images and fluid-attenuated inversion recovery sequences. Typical findings include asymmetric, bilateral, demyelination lesions within the deep and subcortical white matter that spare the periventricular spaces of variable size. These lesions may be associated with cerebral edema. Symmetric basal ganglia and thalami grey matter lesions have been reported more frequently in children. Rarely are T1 hypointense (dark) lesions noted within the white matter in acute disseminated encephalomyelitis. Computed tomography imaging is often within normal limits. Laboratory studies are required to rule out other diagnoses such as meningitis or infectious encephalitis. Laboratory studies for patients with acute disseminated encephalomyelitis may show leukocytosis, elevated C-reactive protein, erythrocyte sedimentation rate, and cerebrospinal fluid (CSF) pleocytosis with elevation in CSF protein. Electroencephalographic data are also nondiagnostic showing encephalopathic slowing typical for clinical symptoms of nonspecific encephalopathy. Electroencephalography may be of value in children with status epilepticus or seizures associated with acute disseminated encephalomyelitis presentation. Acute management includes empiric broad spectrum antimicrobials and acyclovir until bacterial and herpetic meningoencephalitis can be ruled out. Pharmacologic therapy for acute disseminated encephalomyelitis is immunomodulatory. The mainstay of treatment includes high-dose IV glucocorticoids (preferably methylprednisolone) for 3-5 days with variable duration of a tapered regimen to follow for 4-6 weeks. Children who fail to respond to glucocorticoids may be treated with IV immunoglobulin and/or plasma exchange therapy. There have been no prospective clinical trials comparing the use of any of the above therapies. Acute disseminated encephalomyelitis tends to be monophasic with resolution of clinical and radiographic findings in the majority of cases. Most children (60% to 90%) have a full recovery from clinical symptoms without residual deficits. However, a multiphasic or relapsing acute disseminated encephalomyelitis is reported and typically seen more than 3 months after the initial episode. If more than one relapse occurs, strong suspicion for an alternative diagnosis such as multiple sclerosis or neuromyelitis optica should be considered. In most cases, neuroimaging findings are completely to partially resolved on follow up testing in longitudinal case-series. In the vignette described, the child presents in the acute phase of acute disseminated encephalomyelitis and has been triaged and treated suitably for potential meningoencephalitis while awaiting culture, antigen/antibody, and polymerase chain reaction results. The child's seizures appeared controlled and, therefore, loading with an additional antiepileptic agent is unwarranted. Given the constellation of clinical and radiographic findings, empiric therapy with glucocorticoids is appropriate. The use of plasma exchange should be considered if the patient is unresponsive to high-dose glucocorticoid therapy. A transesophageal echocardiogram to assess for endocarditis is unlikely to be of diagnostic benefit given the distribution and type of neuroimaging and clinical findings. PREP Pearls A preceding infectious process, viral or bacterial, is associated with acute disseminated encephalomyelitis also known as postinfectious encephalomyelitis. High-dose IV glucocorticoid therapy is the preferred initial treatment for acute disseminated encephalomyelitis. Unresponsive patients to glucocorticoid treatment can be treated with plasmapheresis or IV immunoglobulin therapy. ABP Content Specifications(s)/Content Area Recognize acute demyelinating disorder as an etiology of seizures Recognize demyelinating disorder as an etiology of coma Suggested Readings Beatty C, Bowler RA, Farooq O, et al. Long-term neurocognitive, psychosocial, and magnetic resonance imaging outcomes in pediatric-onset acute disseminated encephalomyelitis. Pediatr Neurol. 2016;57:64-73. doi: 10.1016/j.pediatrneurol.2016.01.003 Koelman DL, Chahin S, Mar SS, et al. Acute disseminated encephalomyelitis in 228 patients: a retrospective, multicenter US study. Neurology. 2016;86(22):2085-2093. doi: 10.1212/WNL.0000000000002723 Koelman DL, Mateen FJ. Acute disseminated encephalomyelitis: current controversies in diagnosis and outcome. J Neurol. 2015;262(9):2013-2024. doi: 10.1007/s00415-015-7694-7 Pohl D, Apler G, Van Haren K, et al. Acute disseminated encephalomyelitis: updates on an inflammatory CNS syndrome. Neurology. 2016;87(9 Suppl 2):S38-45. doi: 10.1212/WNL.0000000000002825 Waubant E, Ponsonby AL, Pugliatti M, Hanwell H, Mowry EM, Hintzen RQ. Environmental and genetic factors in pediatric inflammatory demyelinating diseases. Neurology. 2016;87(9 Suppl 2):S20-S27. doi: 10.1212/WNL.0000000000003029

A 14-year-old was admitted last night with acute respiratory failure in the setting of septic shock. She had been intubated in the emergency department with etomidate and rocuronium, and was requiring moderate ventilator support. Her medications included fentanyl, midazolam, vancomycin, and ceftriaxone. She was also requiring norepinephrine infusion to maintain adequate hemodynamics. The overnight team sent a morning cortisol level, which was low at 5 μg/dL. Of the medications this patient has received, which one is MOST likely to be associated with this lab result: A. etomidate B. fentanyl C. midazolam D. rocuronium

A. etomidate After its introduction in the early 1960's, etomidate was commonly used as a continuous infusion for anesthesia and sedation. However, growing reports of significant adrenal suppression and increased mortality with continuous etomidate led to discontinuation of this practice and warnings against its use for prolonged periods of time. Etomidate has continued to be used as a single dose for induction with rapid sequence intubation because of its rapid onset of action with minimal hemodynamic effects. Etomidate inhibits the production of corticosteroids by inhibiting the 11-β-hydroxylase pathway of converting cholesterol to cortisol, thus, blocking the normal response to stress such as surgery or critical illness. This inhibitory mechanism results in lower random cortisol levels as well as an impaired response to ACTH stimulation (ie, relative adrenal insufficiency). Although originally described with prolonged use, this adrenal suppression also occurs with a single dose of etomidate in both healthy and critically ill adults and children. Although the sedative effects of a single dose of etomidate generally last for minutes, the adrenal inhibition lasts much longer because the plasma etomidate level required for adrenal suppression is lower than that for sedation. The abnormal cortisol response is reversible with time, but the duration can be variable depending upon the individual physiologic condition. In most studies, the changes are observed within the first 12-24 hours, with reversal by 48 hours. Despite clear evidence of adrenal suppression, the impact of single-dose etomidate on clinical outcomes such as mortality and lengths of mechanical ventilation or stays is much less certain. A recent Cochrane review that included 8 randomized controlled trials comparing etomidate to other induction agents for intubation did not find any difference in outcomes. These studies included a heterogeneous critically ill population, but did not include any pediatric studies. Adrenal insufficiency in patients with sepsis has been well recognized, and thus, the use of etomidate in this population is of particular concern. In the Corticosteroid Therapy of Septic Shock (CORTICUS) trial, the rate of adrenal insufficiency was higher among those receiving etomidate as compared to other induction agents. Furthermore, the risk of mortality among those receiving etomidate was also higher. However, subsequent meta-analyses have had conflicting results. Therefore, while etomidate may be an acceptable option for critically ill patients without sepsis, use of alternative induction agents, if available, should be considered among patients with sepsis. Additionally, it is important to note that corticosteroid replacement after etomidate exposure has not been shown to affect the risk of mortality for critically ill patients with and without sepsis. PREP Pearls Etomidate suppresses cortisol production by inhibiting the 11-β-hydroxylase pathway, which can occur with a single induction dose. Association between mortality and single-dose etomidate is uncertain, but may be increased among patients with septic shock. Therefore, alternative induction agents should be considered in this population. Administration of steroids after etomidate exposure has not been shown to alter the risk of adverse outcomes. ABP Content Specifications(s)/Content Area Understand the high potential for adrenal suppression from a single dose of etomidate and the consequences in certain patients (eg, with septic shock) Suggested Readings Bruder EA, Ball IM, Ridi S, Pickett W, Hohl C. Single induction dose of etomidate versus other induction agents for endotracheal intubation in critically ill patients. Cochrane Database Syst Rev. 2015;1:CD010225. doi: 10.1002/14651858.CD010225.pub2. Chan CM, Mitchell AL, Shorr AF. Etomidate is associated with mortality and adrenal insufficiency in sepsis: a meta-analysis. Crit Care Med. 2012;40(11):2945-2953. doi: 10.1097/CCM.0b013e31825fec26. Forman SA. Clinical and molecular pharmacology of etomidate. Anesthesiology. 2011;114(3):695-707. doi: 10.1097/ALN.0b013e3181ff72b5. Gu WJ, Wang F, Tang L, Liu JC. Single-dose etomidate does not increase mortality in patients with sepsis: a systematic review and meta-analysis of randomized controlled trials and observational studies. Chest. 2015;147(2):335-346. doi: 10.1378/chest.14-1012.

A 10-day-old male neonate is admitted to the pediatric intensive care unit in shock. He was born at 33 weeks gestation and recently discharged at 1 week of life from the neonatal intensive care unit. Parents report a 2-day history of poor feeding. On the day of admission, he was noted to have a distended abdomen and new onset of hematochezia. Examination shows a lethargic pale neonate with a grossly distended and erythematous abdomen. Bowel sounds are absent. His admission labs show a pH of 7.08, pCO2 of 33 mm Hg, and a lactate level of 4.5 mmol/L. His complete blood work is significant for a white blood count of 20,000 with a left shift. Platelet count is 65,000. The patient is intubated. Vascular access is obtained and fluid resuscitation and broad-spectrum antibiotics are started. Of the following, the abdominal radiograph that warrants immediate surgical intervention is:

Answer: B- shows free air in the abdomen In the clinical scenario described, both volvulus and necrotizing enterocolitis (NEC) must be considered. While NEC is often thought of as a disease affecting primarily the premature newborn, it can also occur in newborns of normal gestation. Necrotizing enterocolitis is an inflammatory intestinal illness, originally described in 1952 in term newborns. Severity is defined through use of Bell's staging criteria. Bell's criteria are a composite of clinical, intestinal, and radiographic signs. Clinical criteria include temperature instability, apnea, bradycardia, acidosis, thrombocytopenia, hypotension, disseminated intravascular coagulation, and shock. Intestinal signs include gastric residuals, abdominal distension, hematochezia, ileus, and abdominal wall tenderness/erythema/edema. Radiographic criteria include: dilated bowel loops, pneumatosis, ascites, and pneumoperitoneum. Histopathology of NEC shows an inflammatory infiltrate, hemorrhage, coagulation necrosis of bowel wall, and edema of the lamina propria (Figure 1 and Figure 2). The etiology of NEC is a subject of intense research and is likely multifactorial. Prematurity is a known risk factor, as is congenital heart disease. Exposure of the immature gut to hyperosmolar formulas and fluids is another well-defined risk factor. Hypoxemia and umbilical vessel catheterization increase risk. Recently, alteration of the native intestinal microbiome has been identified as a key risk factor for NEC. This has lead to increasing use of probiotics in neonatal intensive care units. A recent Cochrane review supported use of enteric probiotics as a means to reduce incidence of NEC and improve mortality for newborns with a birthweight <1,000 grams. Lastly, there is increasing evidence supporting a genetic predisposition to NEC. Patients with alteration in their SIGIRR loci have aberrant intestinal toll-like receptor signaling and are at higher risk for NEC development. NEC treatment includes cardiopulmonary support, gut rest, correction of acidosis, management of coagulopathy, and broad-spectrum antibiotic therapy. To date, there is no consensus on the best antibiotic strategy. Despite medical support, over a third of newborns ultimately require surgical intervention. Clinical parameters alone do not predict which newborns ultimately fail medical management. Subtle signs of deterioration may include use of inotropes, persistent acidosis or thrombocytopenia, or rising leukocytosis. More obvious surgical signs include abdominal wall discoloration, palpable abdominal mass, and clinical deterioration despite maximal medical management. Ultimately, no specific clinical sign(s) is pathognomonic for surgical NEC and the decision to pursue surgery is largely at clinical discretion of the surgical team. Surgical procedures can vary from exploratory laparotomy with resection of necrotic bowel to a simple percutaneous insertion of a peritoneal drain. In larger newborn (>1500 gm), 2 large prospective trials failed to show superiority of peritoneal drainage versus laparotomy. A Cochrane review also found no statistical difference in mortality or nutritional requirements with either procedure in NEC. A large, multi-center NIH-sponsored trial comparing these 2 surgical options is currently underway. Necrotizing enterocolitis has a high mortality, approaching 40% in the extremely low-birth weight newborns requiring surgery. Postsurgical morbidity is common. Complications include development of intestinal strictures, intestinal failure, cholestasis, and impaired neurodevelopment. About 10% of NEC newborns develop recurrent episodes, often resulting in parenteral nutrition dependency. Radiographic findings help establish the diagnosis of NEC and can assist with medical decision making. Consensus opinion is that pneumoperitoneum mandates immediate surgical intervention. The presence of intestinal interstitial air (pneumatosis) or gas in the portal tract confirms the diagnosis of NEC, but can often be managed medically. In this vignette, image B demonstrates free peritoneal air. Image A demonstrates pneumatosis intestinalis and gastric emphysema. Image C demonstrates portal venous air. Image D showed dilated loops of bowel consistent with an ileus. PREP Pearls The diagnosis of necrotizing enterocolitis requires a combination of clinical, laboratory, and radiographic findings. Probiotic administration has significantly decreased the prevalence of necrotizing enterocolitis in newborns with a birthweight <1,000 grams. Consensus opinion is that free peritoneal air is the one radiographic finding that warrants immediate surgical intervention. ABP Content Specifications(s)/Content Area Understand the indications for surgical intervention in an infant with necrotizing enterocolitis Suggested Readings Bhatt D, Travers C, Patel RM, et al. Predicting mortality or intestinal failure in infants with surgical necrotizing enterocolitis. J Pediatr. 2017;191:22-27. doi: 10.1016/j.jpeds.2017.08.046 Frost BL, Modi BP, Jaksic T, Caplan MS. New medical and surgical insights into neonatal necrotizing enterocolitis. JAMA Pediatr. 2017;171(1):83-88. doi: 10.1001/jamapediatrics.2016.2708 Raval MV, Moss L. Current concepts in the surgical approach to necrotizing enterocolitis. Pathophysiology. 2014;21(1):105-110. doi: 10.1016/j.pathophys.2013.11.017 Stey A, Barnert ES, Tseng CH, et al. Outcomes and costs of surgical treatments of necrotizing enterocolitis. Pediatrics. 2015;135(5):e1190. doi: 10.1542/peds.2014-1058

An infant is admitted is to the intensive care unit with septic shock. There is concern regarding the development of acute kidney injury, and uncertainty about the ability of a new blood test to identify acute kidney injury compared to the current methods. A recent study presented the following information: among 200 children admitted with septic shock, 100 developed acute kidney injury. Of those who developed acute kidney injury, the test was positive in 70 children. However, the test was also positive in 10 children who did not develop acute kidney injury. Based on these results, the sensitivity of the new blood test for detecting acute kidney injury is: A. 10% B. 30% C. 70% D. 90%

C. 70% Diagnostic testing is an essential component of critical care medicine. However, the accuracy of a test (ie, ability to identify correctly a condition) can be variable, leading to risk of false positive or false negative results. Therefore, it is important to understand the test characteristics when interpreting results and applying them to management decisions. Sensitivity and specificity are common measures of a test's accuracy. Sensitivity describes the ability of a test to identify patients with the condition of interest (ie, true positives), and is expressed as the percentage of patients with the condition who have a positive test. Specificity, on the other hand, describes the ability to identify patients without the condition of interest (ie, true negatives), expressed as the percentages of patients without the condition who have a negative test. Although high sensitivity and high specificity are both desirable, there is often a trade-off between sensitivity and specificity. The relative importance of higher sensitivity versus higher specificity depends upon how the test will be used and the perspective of the user. High sensitivity tests have a lower risk of missing anyone with the disease, and are considered better for ruling out disease (when negative). High specificity tests are more likely to identify the correct disease, and thus, are considered better for ruling in a disease (when positive). To determine the sensitivity and specificity of a test, there must be an accepted gold standard considered to be "truth," against which the test is compared. In the vignette above, the gold standard is the current methods of detection. In calculating the sensitivity and specificity, it is often helpful to create a table (Table 1). Using the information from the vignette, we can enter the information into the table (Table 2). The sensitivity of the test would be the number of true positives divided by the total number of children with the disease: 70/100 (ie, 70%). The specificity would be the number of true negatives divided by the total number of children without the disease: 90/100 (ie, 90%). Based on these results, the specificity of the test is higher than the sensitivity. Therefore, if the test were positive, it is likely the patient has acute kidney injury. However, if the test were negative, there is still a significant chance the patient has acute kidney injury. PREP Pearls The sensitivity of a test is the number of positive results among those with the condition. A highly sensitive test is good for ruling out disease when negative. The specificity of a test is the number of negative results among those without the condition. A highly specific test is good for ruling in disease when positive. ABP Content Specifications(s)/Content Area Calculate and interpret sensitivity and specificity Suggested Readings Fischer JE, Bachmann LM, Jaeschke R. A readers' guide to the interpretation of diagnostic test properties: clinical example of sepsis. Intensive Care Med. 2003;29(7):1043-1051. doi: 10.1007/s00134-003-1761-8

A 9-month-old boy was admitted to the pediatric intensive care unit for status epilepticus 2 months ago. At that time, imaging of the brain showed evidence of multiple subarachnoid hemorrhages along with a subdural hematoma that was evacuated surgically. The head circumference which was tracking on the 50th percentile 2 months ago is above the 95th percentile today. The child is awake and afebrile today. An MRI of the head was completed today and a representative image is shown in the Figure. Of the following, the MOST likely diagnosis is: A. communicating hydrocephalus B. herpes simplex encephalitis C. stenosis of aqueduct of Sylvius D. stenosis of foramen of Monroe III

A. communicating hydrocephalus The cerebrospinal fluid (CSF) is produced in the choroid plexus located in the ventricles of the brain; however, the anterior and posterior horns of the lateral ventricles and the aqueduct of Sylvius are devoid of choroid plexus. Because the CSF is derived from the plasma, it has many similarities to plasma except that it is low in protein and is higher in chloride compared to the serum. The CSF flows in a unidirectional fashion from the lateral ventricles into the third ventricle via the foramen of Monro. It then flows from the third ventricle via the aqueduct of Sylvius into the fourth ventricle. The CSF then escapes the ventricular system into the subarachnoid space via the foramen of Luschka and Magendie. The subarachnoid space is the space between the pia mater and the arachnoid mater, 2 of the 3 layers of meninges that surround the brain. It is believed that the CSF does maintain pulsatility in the subarachnoid space and that this phenomenon is a reflection of the pulsation of the surrounding blood vessels. The CSF provides a buoyant force that supports the brain and acts as a cushion for the brain. The CSF is absorbed back into the circulation via the arachnoid villi that project into the dural sinuses, particularly the superior sagittal sinus. Because CSF is being constantly produced, but also is being constantly reabsorbed back into the blood, any interruption to its normal flow or reabsorption lead to accumulation of the CSF in the cranial vault, a condition that is termed hydrocephalus. The interruption to normal CSF flow can occur in the ventricular system, for instance due to obstruction of the one of the foramina, or in the subarachnoid space due to obstruction of the arachnoid granulations. Traditionally, hydrocephalus has been classified into 2 types: communicating hydrocephalus and noncommunicating hydrocephalus. Communicating hydrocephalus is also called nonobstructive hydrocephalus implying that there is no obstruction to the flow of CSF from the lateral ventricles into the subarachnoid space. In this type of hydrocephalus the CSF can flow freely from the lateral ventricles, to the third ventricle, then via the aqueduct of Sylvius into the the fourth ventricle. The CSF can then exit into the cisterns and the subarachnoid space. However, reabsorption back into the circulation is interrupted at the level of arachnoid granulation located in the dural sinuses. Communicating hydrocephalus is typically seen following inflammatory and infectious conditions of the meninges such as bacterial meningitis that is characterized by intense inflammatory response during the acute stage and is then followed by fibrosis around the arachnoid granulations in the dural sinuses. This leads to disruption of the reabsorption of CSF back to the circulation at the level of dural sinuses. As a result, all of the ventricles are dilated in communicating hydrocephalus as seen in the image of the case of the vignette. In obstructive hydrocephalus, there is usually obstruction at some point in the ventricular systems that leads to disruption of the flow of CSF. This obstruction could be between the lateral ventricles and the third ventricle at the level of foramen of Monro, within or surrounding aqueduct of Sylvius, or the outlet of the fourth ventricle. In noncommunicating hydrocephalus, only the area proximal to the level of obstruction is dilated. For instance, if there is narrowing or obstruction of the aqueduct of Sylvius, the lateral and third ventricles become dilated, but the fourth ventricle would appear normal in size. The case in the vignette has had a previous episode of intraventricular and subarachnoid hemorrhage likely due to abusive head trauma. Formation of clots and the inflammation associated with the initial injuries most likely to have led to fibrosis of the arachnoid granulations resulting in impaired absorption of the CSF; the consequence of these phenomena is accumulation of the CSF in and dilation of the all the ventricles as seen in the Figure. Communicating hydrocephalus is a term that has been used to imply that it is the opposite of obstructive hydrocephalus and to suggest that there no obstruction to flow of CSF from the ventricular system; however, it is important to recognize that in most cases of communicating hydrocephalus there is some element of obstruction to the normal CSF flow. While these nomenclatures can be confusing, they continue to be used and it is important that clinicians become familiar with them. Treatment of hydrocephalus includes medical and surgical approaches. Acetazolamide, a carbonic anhydrase inhibitor, has been used in the management of hydrocephalus. Carbonic anhydrase is involved in the production of CSF in the brain and inhibition of this enzyme may help lower CSF production as a temporary measure until the definitive surgical intervention is instituted. The definitive intervention for hydrocephalus is insertion of a shunt that drains and diverts the CSF. In the past ventriculo-atrial shunts were utilized, however, frequent occurrence of infections and shunt nephritis has made this approach unpopular today. Currently, ventriculo-peritoneal shunt is the most commonly utilized approach to shunting CSF in patients with hydrocephalus. The proximal end of the ventriculo-peritoneal shunt is usually placed in the right lateral ventricle. A one-way valve is usually placed in the proximal portion of the shunt as it exits the lateral ventricle. Often, there is a reservoir proximal to the valve. This reservoir may be compressed to drain the CSF distally. When the ventriculo-peritoneal shunt is functioning properly, the reservoir fills promptly with CSF indicating that the proximal and distal portions of the shunt are patent. Herpes simplex virus encephalitis is characterized by fever, alterations in mental status, and sometimes focal neurological signs. Herpes simplex virus typically involves the temporal lobes and orbital surfaces of the frontal lobes often in a unilateral fashion. Imaging of the brain in these patients will demonstrate abnormalities in the temporal lobes such as high intensity signal changes on MRI of the brain or luxury perfusions in the temporal lobe on CT of the brain. PREP Pearls Communicating hydrocephalus is characterized by dilation of all the ventricles. Ventriculo-peritoneal shunt is the mainstay of therapy for hydrocephalus. Shunt nephritis is a recognized complication of ventriculo-atrial shunt. ABP Content Specifications(s)/Content Area Distinguish between communicating and noncommunicating hydrocephalus Recognize hydrocephalus as a complication of central nervous system infection, infestation, hemorrhage, or tumor Know the medical and surgical treatments of hydrocephalus Suggested Readings Hamilton MG.Treatment of hydrocephalus in adults. Semin Pediatr Neurol. 2009;16(1):34-41. doi: 10.1016/j.spen.2009.02.001 Miller JP, Fulop SC, Dashti SR, Robinson S, Cohen AR. Rethinking the indications for the ventriculoperitoneal shunt tap. J Neurosurg Pediatr. 2008;1(6):435-438. doi: 10.3171/PED/2008/1/6/435 Rekate HL. A contemporary definition and classification of hydrocephalus. Semin Pediatr Neurol. 2009;16(1):9-15. doi: 10.1016/j.spen.2009.01.002

A 14-year-old previously healthy adolescent boy is admitted to your intensive care unit 1 hour after a bicycle accident in which he hit his head on the curb. He was not wearing a helmet. The initial head computed tomography scan showed a small epidural hematoma. Shortly after admission, he has become restless and agitated, and vomited recently ingested solids. The neurosurgeon is concerned that the hematoma could be expanding, and would like you to sedate the patient for an urgent repeat computed tomography scan. Of the following, the NEXT best step to prepare this child for emergent imaging is: A. administer ondansetron B. orotracheally intubate the patient C. place a nasogastric tube D. sedate with propofol

B. orotracheally intubate the patient The patient in the vignette, requiring emergent imaging, is at increased risk of aspirating gastric contents as he has vomited solids recently and can be presumed to still have a full stomach. Aspiration of solid food matter and acidic gastric contents into the larynx and respiratory tract can obstruct the airway, and may lead to pneumonitis with hypoxia, wheezing, patchy atelectasis, or pneumonia. Sedation decreases the ability to protect one's airway. Airway protection by orotracheal intubation would be the appropriate next step in preparing for this child's emergent computed tomography (CT) scan. According to the latest American Heart Association Pediatric Advanced Life Support (PALS) guidelines, there is insufficient evidence for applying cricoid pressure during intubation. Clinical studies indicate that lower gastric volumes (< 25 mL in adults, < 0.4 mL/kg in children) and less acidic pH (> 2.5) reduce the risk and consequences associated with aspiration. To prevent aspiration of gastric contents into the lungs, patients are advised to fast for several hours before elective procedures requiring general anesthesia, regional anesthesia, or procedural sedation and analgesia. The nil per os (npo) period for elective procedures depends on the composition of the last meal ingested. The American Society of Anesthesiologists recommends that an otherwise healthy individual remain npo for a minimum of 6 hours after eating a solid light meal, nonhuman milk, or infant formula, 4 hours after ingesting breast milk, and 2 hours after clear liquids. Otherwise healthy children who are allowed to drink clear fluids up to 2 hours preoperatively do not experience higher gastric volumes or lower gastric pH values than those who fast for longer periods. They are less thirsty and hungry, better behaved, and more comfortable. Fasting for 8 hours should be maintained if the meal included fried or fatty foods. However, fasting for much longer than 8 hours is not advised as it may be associated with lower blood glucose levels. A patient who needs sedation or anesthesia for an emergency procedure should be presumed to have a full stomach and should be considered to have an increased risk for aspiration. Medical conditions such as obesity, hiatal hernia, diabetes, and gastroesophageal reflux disease can affect gastric emptying, and predispose to the regurgitation of gastric contents, thereby increasing aspiration risk. Other conditions such as ileus, bowel obstruction, and tube feeding can also increase risk by predisposing to higher volume of gastric contents. Strategies to reduce aspiration risk include: Reducing gastric volume by nasogastric aspiration and using prokinetic medications Reducing acidity of gastric contents with antacids Airway protection by tracheal intubation Prevention of regurgitation of gastric contents by anesthetic techniques such as rapid sequence induction Extubation in a lateral position, and only after the patient is fully awake and airway protective reflexes have been restored For patients at increased risk for pulmonary aspiration, proactive modification of the volume and acidity of gastric contents is indicated. Medications that block gastric acid secretion such as histamine-2 receptor antagonists or proton pump inhibitors, and prokinetic agents such as metoclopramide may be administered preoperatively in addition to the anesthetic strategies listed above. PREP Pearls Airway protection by tracheal intubation is indicated in patients with a full stomach. Reducing gastric volume by using prokinetic medications may decrease aspiration risk. Reducing acidity of gastric contents with antacids decreases aspiration risk. ABP Content Specifications(s)/Content Area Plan preoperative airway management for a patient with a "full stomach" Understand which patients are at risk for a "full stomach" for anesthesia/sedation Suggested Readings Birenbaum A, Hajage D, Roche S, et al. Effect of cricoid pressure compared with a sham procedure in the rapid sequence induction of anesthesia: the IRIS randomized clinical trial. JAMA Surg. 2019;154(1):9-17. doi:10.1001/jamasurg.2018.3577 Brady M, Kinn S, Ness V, et al. Preoperative fasting for preventing perioperative complications in children. Cochrane Database Syst Rev. 2009;(4):CD005285. doi: 10.1002/14651858.CD005285.pub2 Kojima T, Harwayne-Gidansky I, Shenoi AN, et al. Cricoid pressure during induction for tracheal intubation in critically ill children: a report from national emergency airway registry for children. Pediatr Crit Care Med. 2018;19(6):528-537. doi: 10.1097/PCC.0000000000001531 Practice Guidelines for Preoperative Fasting and the Use of Pharmacologic Agents to Reduce the Risk of Pulmonary Aspiration: Application to Healthy Patients Undergoing Elective Procedures. An Updated Report by the American Society of Anesthesiologists Task Force on Preoperative Fasting and the Use of Pharmacologic Agents to Reduce the Risk of Pulmonary Aspiration. Anesthesiology. 2017;126(3):376-393. doi: 10.1097/ALN.0000000000001452 Robinson M, Davidson A. Aspiration under anaesthesia: risk assessment and decision-making. Continuing Education in Anaesthesia Critical Care & Pain. 2014;14(4):171-175. https://doi.org/10.1093/bjaceaccp/mkt053

A 16-year-old girl is admitted to the pediatric intensive care unit with pallor, peripheral edema, and hypertension. Two weeks ago she had a miscarriage in her second trimester. Laboratory evaluation shows the following values: Laboratory Test Result BUN 92 mg/dL (32.8 mmol/L) Creatinine 5.1 mg/dL (388.9 µmol/L) Albumin 1.9 g/dL (19 g/L) Hemoglobin 5.5 g/dL (55 g/L) Platelet count 88 cells/mm3 (88 × 109 cells/L) Her urinalysis results show 3+ proteinuria and red blood cells that are too numerous to count. Direct Coombs test is negative. Her peripheral smear is shown (Figure). Her chest radiograph shows no evidence of air space disease. Blood and stool cultures are negative. Her von Willebrand activity is normal. She denies a history of diarrhea, cough, rash, or recent infections. Of the following, the biochemical abnormality MOST likely involved in this patient's disease process is: A. ADAMTS13 deficiency B. complement factor H deficiency C. glycolipid Gb3 binding D. Thomsen-Friedenreich antigen exposure

This scenario depicts an atypical form of hemolytic uremic syndrome (HUS). Hemolytic uremic syndrome belongs to a group of disorders known collectively as thrombotic microangiopathies. The common features for thrombotic microangiopathies are hemolysis, thrombocytopenia, and thrombus formation in small vessels causing end-organ damage. Commonly encountered thrombotic microangiopathies include toxin-associated HUS, thrombotic thrombocytopenic purpura (TTP), atypical HUS, and HUS associated with a coexisting disease (called secondary HUS). Hemolytic uremic syndrome is often considered a pediatric disease, whereas TTP affects primarily adults, although overlap is increasingly noted in the literature. The symptoms, clinical signs, and cellular involvement (ie, endothelial cells, red blood cells, platelets) in all forms of thrombotic microangiopathies are similar. Kidneys, brain, gastrointestinal tract, and the heart may all be affected. Patients typically present with anemia, thrombocytopenia, edema, oliguria, proteinuria, hematuria, and impaired renal function. Involvement of the CNS vasculature can cause seizures, decreased level of consciousness, and coma. Classic histologic findings in HUS include thickening of the vascular wall, endothelial cell edema, detachment of the endothelium, and exposure of the basement membrane. This causes platelet activation leading to the formation of microthrombi that partially obstruct the lumen. As a result, many red blood cells are sheared as they pass through the thrombus, resulting in schistocyte formation (Figure). Understanding the pathogenesis of the various forms of thrombotic microangiopathies has steadily increased. Most pediatric cases of HUS are caused by Shiga toxin-producing E. coli, with an annual incidence in children under the age of 5 years of 6.1 per 100,000 cases. Although E. coli O157:H7 is the most recognized serotype causing infection-associated HUS, other E. coli serotypes such as O26, O45, O103, O111, O121, and O145 are also associated with toxin-related HUS. These organisms express Shiga toxin (or Shiga-like toxin), which binds to the cell membrane receptor glycolipid Gb3. After internalization, the Shiga toxin halts protein synthesis and induces apoptosis. Kidneys are the primary organ affected in HUS due to the abundant expression of Gb3 on the glomerular endothelium. Secondary HUS is not associated with bacterial toxin. It is frequently associated with a coexisting condition such as infections, cancer, autoimmune disease, organ transplantation, and certain cytotoxic drugs. The common feature of these coexisting conditions is that they may cause direct cell damage, promote activation of the complement system, or enhance activation of complement against self. The direct Coombs test is positive when HUS is triggered by Streptococcus pneumoniae. The Thomsen-Friedenreich antigen, found on platelets and endothelium, is normally "hidden" by neuraminic acid but is exposed by pneumococcal neuraminidase. Host antibodies form, recognize this exposed antigen, and initiate an inflammatory cascade. The direct Coombs test detects these circulating antibodies. Hemolytic uremic syndrome without coexisting disease and not associated with infections is referred to as atypical HUS. Atypical HUS is seen in 5% of HUS cases and affects people of all ages; it may be sporadic or familial. According to the currently prevailing classification, atypical HUS is associated with a genetic or acquired defect in regulation of complement activation. There are 3 main pathways causing activation of the complement system: the classical pathway, lectin pathway, and the alternative pathway. Investigators have shown that atypical HUS involves unopposed activation of the alternate pathway. The main complement regulators of the alternative pathway are plasma proteinfactor H, as well as the membrane proteins CD35, CD46, CD55, and CD59. These agents act as co-factors for factor I in proteolytic inactivation of C3b, compete with factor B in binding to C3b, or accelerate decay of formed C3b. These proteins all contribute to the protection of endothelial cells. In the event these regulatory proteins are missing, the alternate complement pathway is unopposed, causing resultant cell damage from prolonged inflammation. To date, mutations in complement regulatory proteins have been detected in 60% of the patients with atypical HUS. Unlike atypical HUS, the physiologic defect in TTP does not involve the alternate complement pathway. Rather, it is caused by a deficiency in ADAMTS13 activity, a metalloprotease that cleaves von Willebrand factor. In atypical HUS, unopposed activation of the alternate pathway injures red blood cells, platelets, leukocytes, and endothelial cells. The cells most vulnerable to membrane damage are red blood cells as they lack an efficient membrane repair system. This explains the presence of hemolysis and red blood cell fragments in the peripheral blood of patients with atypical HUS. Platelets are easily activated by complement activity. Platelets are normally protected from complement attack by the concerted action of factor H and other membrane regulators. This protection is impaired in atypical HUS, leading to microthrombi formation. Compared with other forms of HUS, the prognosis for patients with atypical HUS is poor. Up to 40% of the patients die or progress to end-stage renal disease despite aggressive therapy. Before the advent of new treatment options for atypical HUS, plasmapheresis therapy was recommended despite the lack of controlled randomized trials. Current guidelines recommend that all patients with HUS be tested for complement anomalies whenever possible. With the clearer understanding of the pathogenesis of atypical HUS, the focus of treatment has changed. Eculizumab, a humanized monoclonal antibody approved by the Food and Drug Administration, which acts as an inhibitor of the terminal pathway of the complement cascade, is now recommended as initial therapy in all patients with atypical HUS. PREP Pearls Toxin-associated hemolytic uremic syndrome is mediated through shiga or shiga-like toxins binding to the Gb3 receptor on endothelium and inducing apoptosis with subsequent exposure of the extracellular matrix, a known activator of the coagulation cascade. Hemolytic uremic syndrome due to Pneumococcus will often have a positive direct Coombs test result. Atypical hemolytic uremic syndrome is not associated with bacterial toxins. Rather, it is caused by inappropriate activation of the alternate complement pathway. Atypical hemolytic uremic syndrome is frequently associated with genetic mutations causing decreased expression of factor H, a known inhibitor of the alternate complement pathway. Eculizumab, a humanized monoclonal antibody that inhibits activation of the terminal pathway of the complement cascade, is considered the drug of choice for the treatment of atypical hemolytic uremic syndrome. ABP Content Specifications(s)/Content Area Recognize non-infectious hemolytic-uremic syndrome Know the long-term complications associated with hemolytic-uremic syndrome Suggested Readings Greenbaum LA. Atypical hemolytic uremic syndrome. Adv Pediatr. 2014;61(1):335-356. doi:10.1016/j.yapd.2014.04.001 Loirat C, Fakhouri F, Ariceta G, et al; HUS International. An international consensus approach to the management of atypical uremic syndrome in children. Pediatr Nephrol. 2016;31(1):15-39. doi:10.1007/s00467-015-3076-8 Trachtman H. HUS and TTP in children. Pediatr Clin N Am. 2013;60(6):1513-1526. doi:10.1016/j.pcl.2013.08.007

A 3-year-old boy is admitted to your pediatric intensive care unit while awaiting surgical removal of an ingested foreign body. He initially presented with a history of respiratory distress, dysphagia, and increased drooling. Prior to arrival in the pediatric intensive care unit, a lateral chest film was obtained (Figure). Of the following, a mechanism MOST likely to cause tissue injury in this scenario is: A. heavy metal exposure B. hydroxide formation C. localized secondary infection D. pressure necrosis

B. hydroxide formation Pediatric foreign body ingestions are common. Frequently ingested items in the toddler-aged child include coins, toys, bones, magnets, and button batteries. Both button battery and magnet ingestions have increased over the past decade and can be a significant cause of pediatric injury. Clinical presentation can be nonspecific and include stridor, coughing, irritability, feeding intolerance, chest pain, and respiratory distress. Often, the diagnosis is made by chest radiography. Foreign bodies lodge in the esophagus twice as commonly as in a bronchus and can cause serious complications including esophageal perforation, tracheoesophageal fistula formation, strictures, abscess, recurrent laryngeal nerve injury, and upper gastrointestinal bleeding. A devastating complication of erosion into the aorta can also occur. A large 2010 study in the United States of over 8,000 battery ingestions demonstrated that 12.6% of patients experienced serious complications or death. A newer US study of almost 9,000 battery ingestions demonstrated 1% of cases developed life-threatening injury, with a mortality rate of 0.15%. Button batteries are currently responsible for almost 10% of foreign body ingestions in children. The incidence of button battery ingestions has steadily risen with the increasing prevalence of miniaturized electronics. Button batteries are typically 5 to 25 mm in diameter and composed of a "body" (positive terminal) and a "cap" (negative terminal). This gives button batteries a bi-leveled appearance on radiographs, as noted in the lateral neck film of this vignette. Recent developments in battery technology have led to more powerful button batteries with a concurrent increase in tissue injury following ingestion. Most new lithium button batteries have 3 V or greater capacity, compared to older batteries with 1.5 V capacity. In pediatrics, button batteries sizes 20 mm or larger are more likely to lodge in the esophagus. According to data from the National Battery Ingestion Hotline, 90% of button battery ingestions occur in children less than 3 years of age. When lodged in the esophagus, tissue necrosis quickly develops. Initially, 3 factors were thought to contribute to tissue injury: leakage of alkaline electrolytes, pressure necrosis, and generation of an external current. It is now known that electrical current generation is the proximate cause of tissue injury following battery button ingestions. The electrical current causes isothermic electrolysis of the adjacent tissue fluids, generating hydroxide formation at the battery's negative pole. Hydroxide formation results in development of a highly alkaline local environment with resultant tissue necrosis. Several recent studies have explored strategies to decrease the generation of hydroxide radicals and resultant alkaline tissue injury prior to or at the time of endoscopic removal. These include administration of honey or sucralfate in an attempt to coat the negative pole of the battery, or acetic acid irrigation at the time of battery removal to decrease alkaline injury. In in vivo animal models, esophageal mucosal injury began as little as 2 hours following battery exposure and progressed to deep tissue necrosis by 6 hours. A study of battery ingestions in the Washington DC area showed that the time to esophageal perforation was variable, with several patients demonstrating perforation within 12 hours. In 2015, the North American Society for Pediatric Gastroenterology, Hepatology, and Nutrition published guidelines on management of ingested foreign bodies. These include immediate removal (<2 hours) of all esophageal button batteries, regardless of whether the patient is symptomatic or asymptomatic. The European Society for Paediatric Gastroenterology, Hepatology and Nutrition adopted these recommendations in 2017. The North American Society for Pediatric Gastroenterology, Hepatology, and Nutrition also recommends that endoscopic removal may be delayed up to 48 hours if the battery has passed beyond the gastroesophageal junction and the child is asymptomatic. These differ slightly from the guidelines of the National Battery Ingestion Hotline who recommends removal if the battery remains in the stomach 4 days after ingestion. PREP Pearls Button battery ingestions are increasingly common in pediatrics and can cause tissue injury in as little as 2 hours and esophageal perforation within 12 hours. Hydroxide formation due to electrical current is the likely mechanism of injury in button battery ingestions. Immediate endoscopic removal is currently recommended for ingested batteries lodged in the esophagus. This can be delayed for 2 to 4 days in asymptomatic patients when the battery has passed distal to the gastroesophageal junction. ABP Content Specifications(s)/Content Area button battery ingestion Recognize the clinical and laboratory manifestations of acute alkali ingestion Understand the pathogenesis and toxic effects of ingested alkali Plan the diagnostic assessment and evaluation of a child suspected of alkali ingestion Plan appropriate therapy for a child with acute alkali ingestion Suggested Readings Bolton SM, Saker M, Bass LM. Button battery and magnet ingestion in the pediatric patient. Curr Opin Pediatr. 2018;30(5):653-659. doi: 10.1097/MOP.0000000000000665 Eliason MJ, Ricca RL, Gallagher TQ. Button battery ingestion in children. Curr Opin Otolaryngol Head Neck Surg. 2017;25(6):520-526. doi: 10.1097/MOO.0000000000000410 Kramer RE, Lerner DG, Lin T, et al; North American Society for Pediatric Gastroenterology, Hepatology, and Nutrition Endoscopy Committee. Management of ingested foreign bodies in children: a clinical report of the NASPGHAN Endoscopy Committee. J Pediatr Gastroenterol Nutr. 2015;60(4):562-574. doi: 10.1097/MPG.0000000000000729 Krom H, Visser M, Hulst JM, et al. Serious complications after button battery ingestion in children. Eur J Pediatr. 2018;177(7):1063-1070. doi: 10.1007/s00431-018-3154-6

Three days ago a 13-year-old teenager was found by his father hanging by a belt from his upper bunk bed post. He was intubated without difficulty in the field and never required chest compressions. The teenager has significant hypoxic-ischemic brain injury as evidenced by his clinical examination and imaging. You are called to the bedside to evaluate the patient for respiratory distress following extubation. Of the following, which is MOST likely to be correct regarding his injury complex: A. angiography demonstrates carotid artery injury B. computed tomography demonstrates a Hangman's fracture C. flexible bronchoscopy demonstrates airway edema D. radiograph demonstrates hyoid bone fracture

C. flexible bronchoscopy demonstrates airway edema Strangulation events may occur in children accidentally (eg, entanglement in window blind cords and swing set chains, head entrapment), pseudo-accidentally (eg, choking game, autoeroticism), or intentionally (eg, child abuse, homicide, suicide). Hanging events are defined as complete or incomplete depending on the suspension of the body. A complete hanging involves suspension from a height greater than the child's height and typically is a lethal injury. Incomplete or near-hanging is one in which the body is in contact with a surface and not completely suspended. Interestingly, cervical spine injury is extremely rare in pediatric near-hanging, though it is reported in complete hanging. Rather, injury results from compression and obstruction of vascular (ie, venous, not arterial) structures and from direct airway obstruction. Hypoxemia and brain injury result, which can lead to cardiac arrest. External physical signs do not correlate with the severity of injury either to neck structures or the brain. Hyoid bone, tracheal cartilage, and cricoid cartilage fractures are rare in children due to the greater compliance of the immature structures. Hyoid bone fracture is rarely reported in children, but has been noted in victims as young as 14-years-old. In adults, laryngeal and pharyngeal injury from near-hanging is seen in 5% to 7% of cases, with hyoid fracture not uncommonly observed; whereas a fractured hyoid bone in a child indicates a severe, occult soft-tissue injury. This degree of neck trauma typically is seen only in fatal hanging (complete) events and discovered on autopsy. Of note, fracture of the cricoid is virtually pathognomonic for homicidal rather than suicidal strangulation injury. Hangman's fracture (both pedicles of C2 fractured), spine, and spinal cord injury are exceedingly rare in pediatric near-hanging events. Strangulation by suspension, as in the vignette, most typically results in laryngeal edema. A recent single-institution 10-year retrospective study of children with accidental and nonaccidental near-hanging events reported laryngeal injuries including: vocal cord fixation, subglottic and supraglottic edema, and glottic hematoma. Additionally, negative pressure pulmonary edema is not uncommon. Laryngotracheal injuries are more common in suicide attempts than in accidental near-hangings. Laryngeal injuries severe enough to impede the ability to obtain an artificial airway are rare. Mucosal and cutaneous petechiae, also known as Tardieu's spots, do not correlate to the severity of injury and even severe injury may lack clinical correlates. Cardiac arrest, seizures, and metabolic acidosis (as a function of asphyxial time) are associated with poor outcome. In an Australian study of pediatric out-of-hospital cardiac arrests, 7.8% were due to hanging, of which only 10% were accidental. Of hanging victims with cardiac arrest in their study (n=53), only 3 survived and only 1 of those survivors was neurologically intact. Although significant laryngeal injury in a child near-hanging victim is uncommon, signs and symptoms such as: pain on tongue movement or swallowing, hoarse voice, dyspnea, stridor, crepitus, or hemoptysis should trigger further investigation. Otorhinolaryngology may be consulted for flexible fiberoptic examination or direct laryngoscopy. Imaging via CT should be considered to assess for hyolaryngeal fracture, especially in teenage suicide attempts. Angiography may be obtained although there are no reports of vascular injury in pediatric near-hanging. Minor laryngeal injury may be managed with head of bed elevation and anti-reflux therapy, steroids for edema, and, as warranted, airway protection. Major injury will require exploration and repair of any laryngeal mucosa tears and evaluation of vocal cord function. PREP Pearls Spine and spinal cord injury are rare in near-hanging events. Mucosal and cutaneous petechiae, also known as Tardieu's spots, do not correlate to the severity of injury. Laryngeal injuries severe enough to impede the ability to obtain an artificial airway are rare. ABP Content Specifications(s)/Content Area Understand the pathophysiology of a strangulation or hanging injury Know that laryngeal injury may occur following hanging Suggested Readings Deasy C, Bray J, Smith K, Harriss LR, Bernard SA, Cameron P; Victoria Ambulance Cardiac Arrest Registry steering committee. Paediatric hanging associated out of hospital cardiac arrest in Melbourne, Australia: characteristics and outcomes. Emerg Med J. 2011;28(5):411-415. doi: 10.1136/emj.2010.105510. Hackett AM, Kitsko DJ. Evaluation and management of pediatric near-hanging injury. Int J Pediatr Otorhinolaryngol. 2013;77(11):1899-1901. doi: 10.1016/j.ijporl.2013.09.003. Sep D, Thies KC. Strangulation injuries in children. Resuscitation 2007;74(2):386-391. doi: 10.1016/j.resuscitation.2006.09.019.

A 14-year-old boy who was otherwise healthy was crossing the street while sending a text message to his friend. He was struck by a vehicle traveling at 72 km/h (45 mph). After his admission, the Glasgow Coma Scale score declined from 13 to 9, at which point he was intubated and mechanical ventilation was initiated. On physical examination, the heart rate is 85 beats/min, blood pressure is 115/65 mm Hg, the respiratory rate is 28 breaths/min on the ventilator with an end-tidal carbon dioxide concentration of 35 mm Hg, and the oxygen saturation measured by pulse oximetry is 100% on fraction of inspired oxygen of 0.50. The pupils are midsize and only sluggishly reactive to light. Drainage of serosanguineous fluid from the right ear canal is noted, but the remaining results of the physical examination are reported to be within the normal limits. Diagnostic imaging of the head is performed. Of the following images, the one that shows findings that are MOST likely in this case is

Choice A Computed tomography (CT) and magnetic resonance imaging (MRI) of the brain are used frequently in the clinical diagnosis and management of patients who present with neurologic symptoms. Clinicians should become familiar with these imaging modalities and equip themselves with the basic knowledge necessary for the initial interpretation of these images so that they can provide appropriate and timely medical care to their patients. Consultation with a pediatric neuroradiologist should be considered when appropriate. Computed tomography can be performed expeditiously in the emergency setting and may be completed in few minutes. In the United States, CT is readily available in most clinical settings around the clock and can be interpreted within a reasonable time by a radiologist with expertise in neuroimaging. Magnetic resonance imaging takes longer to complete and in order to produce good-quality images, it requires the patient to stay still during the procedure. In children, this often necessitates administration of deep sedation or anesthesia to immobilize the patient. Therefore, MRI is often less practical for emergency imaging of the brain. However, unlike CT scanning of the brain, MRI is not associated with radiation exposure. Computed tomography of the brain is helpful in the diagnosis of various types of intracranial hemorrhages, including epidural hematomas, subdural hematomas, and intraparenchymal hemorrhages. Acute bleeding appears as high-intensity signals on the CT scan. In the case described in the vignette, acute bleeding is noted around the right petrous temporal bone and in the posterior fossa around the right lobe of the cerebellum (A). Figure 1 also demonstrates a fracture of the right petrous temporal bone. This injury represents a type of basilar skull fracture. Basilar skull fractures are often associated with cerebrospinal fluid (CSF) leak. In the case in the vignette, the CSF leak is manifesting as CSF otorrhea from the ipsilateral (right) ear canal. On a CT scan of the head from a normal child, the ventricles, cisterns , sulci, and gyri are clearly identifiable. The suprasellar cistern and the quadrigeminal cistern are open and clearly identified (arrows in Figure 2). Also, the differentiation between the white matter (darker, or low-intensity, areas) and gray matter (lighter, or higher-intensity, area) is clearly delineated. When cerebral edema develops, the gray-white matter differentiation becomes blurred. As the cerebral edema progresses, the cisterns become less distinct, and in severe cerebral edema the cisterns may become completely effaced (B showing complete effacement of the suprasellar cistern). The quadrigeminal cistern is also effaced, suggesting impending or evolving central herniation. The patient in the vignette is in a coma with a low Glasgow Coma Scale score; however, he does not yet exhibit signs of impending herniation such as anisocoria, irregularities in respiratory pattern, hypertension, or bradycardia. The high-intensity signal changes in the temporal lobe on the T2-weighted MRI of the brain (C) are most consistent with herpes simplex virus (HSV) infection of the brain, which characteristically involves the temporal lobes and orbital surfaces of the frontal lobes. Clinical features of HSV infection of the brain range from fever and confusion to rapidly progressive neurologic symptoms with agitation and disorientation that may culminate in coma. The patient in the vignette was previously healthy and did exhibit any of these clinical features before being struck by a vehicle traveling at a high speed. Magnetic resonance imaging is considered more sensitive than CT scanning in diagnosing HSV infection in the early stages of the disease. Hydrocephalus is accumulation of excess CSF in the intracranial vault, typically in the ventricular system leading to ventriculomegaly. Traditionally, hydrocephalus has been classified into 2 categories: communicating hydrocephalus and noncommunicating hydrocephalus. Communicating hydrocephalus is also called nonobstructive hydrocephalus, implying that there is no obstruction to CSF flow in the ventricular system and that CSF can flow freely between the the lateral ventricles, the third ventricle, the aqueduct and the fourth ventricle, and the subarachnoid space. In communicating hydrocephalus, all the ventricles are dilated (D). Communicating hydrocephalus is typically seen after bacterial meningitis in which the intense inflammation is followed by fibrosis/gliosis around the arachnoid granulations in the dural sinuses, leading to disruption of reabsorption of CSF back to the circulation via the arachnoid granulations. Consequently, the entire ventricular system and the interconnections will be dilated; this includes the lateral ventricles, third ventricle, aqueduct of Sylvius, and fourth ventricle. However, there is no obstruction at any point between the ventricles and the aqueduct, thus the term nonobstructive hydrocephalus. In obstructive hydrocephalus, there is usually an obstruction at some point in the ventricular system where the flow of CSF is disrupted. This obstruction could be between the lateral ventricles and the third ventricle, in the aqueduct of Sylvius, or at the outlet of the fourth ventricle at the level of foramen of Luschka or the foramen of Magendie. In this type of hydrocephalus, the area before the level of obstruction will be dilated. The history in the case in the vignette is not suggestive of a previous episode of meningitis, and the clinical features are inconsistent with slowly progressive increase in intracranial pressure that would be consistent with hydrocephalus. PREP Pearls Cerebrospinal fluid otorrhea is characteristic of basal skull fracture, including fracture of the petrous temporal bone. In the setting of trauma, the cerebrospinal fluid drainage from the ear may not be clear and may be serosanguinous because of associated bleeding as a result of head injury. Changes on the magnetic resonance images in the setting of herpes simplex virus infection of the brain characteristically involve the medial temporal lobe that do not respect hippocampal borders. However, the basal ganglia typically are spared. Effacement of the basal cisterns are highly suggestive of impending herniation. ABP Content Specifications(s)/Content Area Recognize communicating and noncommunicating hydrocephalus on CT or MRI of the head Recognize intracranial hemorrhage on CT or MRI of the head Recognize herpes encephalitis on CT or MRI of the head Recognize herniation on CT or MRI of the head Recognize cerebral edema on CT or MRI of the head Suggested Readings Granerod J, Ambrose HE, Davies NW, et al; UK Health Protection Agency (HPA) Aetiology of Encephalitis Study Group. Causes of encephalitis and differences in their clinical presentation in England: a multicenter, population-based prospective study. Lancet Infect Dis. 2010;10(12):835-844. doi:10.1016/S1473-3099(10)70222-X Mutch CA, Talbott JF, Gean A. Imaging evaluation of acute traumatic brain injury. Neurosurg Clin N Am. 2016;27(4):409-439. doi:10.1016/j.nec.2016.05.011 Pearce MS, Salotti JA, Little MP. Radiation exposure from CT scans in childhood and subsequent risk of leukaemia and brain tumours: a retrospective cohort study. Lancet. 2012;380(9840):499-505. doi:10.1016/S0140-6736(12)60815-0

A 7-month-old infant girl with a ventricular septal defect underwent surgical repair at 6-months of age for protracted heart failure and failure to thrive. Following surgery, it was noted that the infant has a patch leak with a left-to-right shunt. The oxyhemoglobin saturation in the right atrium as measured by cardiac catheterization is 60%, in the pulmonary artery is 90%, and the oxyhemoglobin saturation measured by pulse oximetry in the right hand is 100%. Of the following, the MOST likely estimated ratio of pulmonary to systemic flow in this patient is: A. 1:1 B. 2:1 C. 3:1 D. 4:1

D. 4:1 It is possible to estimate the ratio of the pulmonary to systemic blood flow (Qp/Qs) using the oxyhemoglobin saturation (sat) in the vena cava or right atrium and pulmonary artery. The simplified equation for calculating the ratio of pulmonary to systemic blood flow is: Qp/Qs = (Aorta oxygen sat - vena cava sat)/(Pulmonary vein sat - Pulmonary artery sat) In this equation, a number of assumptions are made, including that the pulmonary vein sat is 100% and that the left ventricular sat is the same as the sat in the aorta and the peripheral tissues as measured by blood gas analysis or pulse oximetry. Applying this equation in the case in the vignette would yield the following results: Qp/Qs = (100-60)/(100-90) = 40/10 = 4 Indicating that the pulmonary blood flow is 4 times that of systemic flow or a pulmonary to systemic blood flow ratio (Qp/Qs) of 4:1. In order to understand how the simplified equation is derived, one needs to understand how to calculate cardiac output using the Fick principle, using oxygen consumption as a marker. When the blood flow is normal and there is no shunt, the systemic blood flow and the pulmonary blood flow are equal and therefore the Qp/Qs ratio is 1:1. However, when there is a left to right shunt via a ventricular septal defect (VSD) or a VSD patch leak as seen in the case in the vignette, a portion of the cardiac output pumped by the left ventricle is shunted across the defect into the right side of the heart and is added to the right sided output in the pulmonary circulation as seen in Figure. In this clinical situation, there is higher pulmonary blood flow than systemic blood flow because some of the systemic blood is being shunted from the left sided circulation to the right side of the circulation. The ratio of this excess pulmonary blood flow can be calculated using the modification of the equation of the Fick principle as follows (1.36 mL O2/g hemoglobin [Hgb] is the oxygen carrying capacity of Hgb): Systemic oxygen consumption = Hgb x 1.36 (arterial sat - vena cava sat) x Qs Pulmonary oxygen consumption = Hgb x 1.36 x (pulmonary vein sat - pulmonary artery sat) x Qp Solving the equation will lead to the simplified shunt equation (Qp/Qs) as follows: Qp x Hgb x 1.36 x (pulmonary vein sat - pulmonary artery sat) = Qs x Hgb x 1.36 x (aorta sat - vena cava sat) Since 1.36 and Hgb cancel each other on either side of the equation, we are left with the following: Qp (pulmonary vein sat - pulmonary artery sat) = Qs (aorta sat - vena cava sat) Thus, Qp/Qs = (aorta sat - vena cava sat)/(pulmonary vein sat- pulmonary artery sat). Normally, the pulmonary and systemic flow are equal and, therefore, the Qp/Qs is 1. Estimation of Qp/Qs using the Fick principle requires cardiac catheterization in order to measure certain oxyhemoglobin saturations. It is possible to estimate the pulmonary blood flow and the systemic blood less invasively using magnetic resonance imaging of the heart. In this method, systemic blood flow (Qs) may be calculated by measuring the blood flow in the aorta and the pulmonary blood flow (Qp) by measuring the flow in the pulmonary conus. From these 2 measurements, the Qp/Qs can be calculated. Measurement of the Qp/Qs has clinical implications; for example, in patients with atrial septal defect, closure of the defect is indicated when the Qp/Qs of > 1.5:1 in association with atrial dilation. PREP Pearls Qp/Qs can be estimated if the oxyhemoglobin saturation in the pulmonary artery and the vena cava are known. Qp/Qs may also be calculated using magnetic resonance imaging of the heart. ABP Content Specifications(s)/Content Area Understand how to calculate systemic blood flow in the presence of a left-to-right shunt Suggested Readings Heathfield, E, Hussain T, Qureshi S, et al. Cardiovascular magnetic resonance imaging in congenital heart disease as an alternative to diagnostic invasive cardiac catheterization: as single center experience. Congenit Heart Dis. 2013;8(4):322-327. doi: 10.1111/chd.12032 O'Laughlin M, Ringel R. Diagnostic and therapeutic cardiac catheterization. In: Nichols DG, editors. Critical Heart Disease in Infants and Children. 2nd Ed. Philadelphia, PA: Mosby;2006: 463-478. Stephensen SS, Steding-Ehrenborg K, Thilen U, et al. Changes in blood volume shunting in patients with atrial septal defect: assessment of heart function with cardiovascular magnetic resonance imaging during dobutamine stress. Eur Heart J Cardiovasc Imaging. 2017;18(10):1145- 1152. doi: 10.1093/ehjci/jew176

A 3-year-old child is being cared for in the intermediate care unit with 30% total body surface area second and third degree scald burns. Daily dressing changes and debridement are being performed by the surgical service. The child has no airway related injury, has remained on room air, and has been taking nutrition by mouth along with nocturnal continuous tube feedings. The daily dressing change and debridement was completed 30 minutes earlier using sedation. The bedside nurse is concerned the child is tachycardic with a heart rate of 170 beats/min and hypertensive with a noninvasive blood pressure of 130/76 mm Hg. Of the following, the drug MOST likely used for sedation was: A. dexmedetomidine B. fentanyl C. ketamine D. propofol

C. ketamine Ketamine is commonly used in moderate sedation and intravascular anesthesia in children for procedures where maintenance of spontaneous respiration and airway reflexes are desired. Considerations must be made for age, comorbidities, intravenous access, and fasting status when any sedative, analgesic, or anesthetic agent is being considered for procedural sedation. Ketamine is a phencyclidine derivative that can be administered intravascular or intramuscular. It is unique when compared to other intravascular sedatives and analgesics. Ketamine inhibits cortical activity while simultaneously stimulating the limbic system. The result is a combination of dissociation, analgesia, sedation, and hypnosis that is highly effective for brief, painful procedures, such as burn debridement as in our scenario. Ketamine is a myocardial depressant, however, this is overcome by its overwhelming sympathomimetic action resulting in the release of endogenous catecholamines making this the correct answer. This results in tachycardia and hypertension. The release of endogenous catecholamines promotes bronchodilation and pulmonary compliance. At the same time, ketamine preserves respiratory drive and airway reflexes making it highly attractive in the intensive care unit environment. Ketamine can be administered intravascular, intramuscular, or intranasally. One of the most commonly observed ketamine-related adverse effects is reemergence reactions or hallucinations. As a child emerges from this dissociative anesthetic agent they may have unpleasant hallucinations or dreams. A benzodiazepine is commonly administered along with ketamine to reduce this emergence reaction. Another important adverse reaction includes the incidence of laryngospasm (0.4% of cases) following ketamine administration. For this reason, ketamine should be used with caution in extremely young infants (<3 months of age), those where airway manipulation is intended as part of procedure, or when there is a concurrent upper respiratory infection. However, recent data in young children undergoing peritonsillar procedures and endoscopy did not find an increased incidence of laryngospasm. The sympathomimetic effects of ketamine increase hypersalivation that has been considered contributory to laryngospasm. Administration of an anticholinergic agent such as glycopyrrolate or atropine to reduce secretions may be beneficial. Other adverse effects include nausea/emesis and nonpurposeful movements during dissociation. Ketamine use for airway control in head injury patients has been avoided because of the sympathomimetic effects and potential increase in intracranial pressure. Recent studies have shown that ketamine can be used safely for rapid sequence intubation and continued sedation and pain control in this patient population. PREP Pearls Ketamine is a commonly used anesthetic agent in the intensive care unit that preserves spontaneous respiration and airway reflexes. Myocardial depression can occur in a patient receiving ketamine if they are catecholamine depleted. Bronchodilation produced by ketamine is a secondary effect related to the stimulation of endogenous catecholamines. Emergence reaction can be treated with keeping the patient calm and the use of benzodiazepines. ABP Content Specifications(s)/Content Area Recognize the sympathetic stimulation produced by ketamine, as well as the consequences of this stimulation Understand that ketamine maintains airway reflexes Know that ketamine is a cause of hallucinations Understand that ketamine-induced hallucinations can be prevented/ treated with benzodiazepines Suggested Readings Bailey AM, Baum RA, Horn K, et al. Review of intranasally administered medications for use in the emergency department. J Emerg Med. 2017;53(1):38-48. Doi: 10.1016/j.jemermed.2017.01.020 Radvansky BM, Puri S, Sifonios AN, Eloy JD Le V. Ketamine-a narrative review of its uses in medicine. Am J Ther. 2016;23(6):e1414-e1426. doi: 10.1097/MJT.0000000000000257 Zeiler FA, Teitelbaum J, West, M, Gillman LM. The ketamine effect on ICP in traumatic brain injury. Neurocrit Care. 2014;21(1):163-173. doi: 10.1007/s12028-013-9950-y

A 5-year-old boy is admitted to the pediatric intensive care unit following a prolonged grand mal seizure. The mother reports she has not been giving the patient his home antiepileptic medications due to financial difficulties. The patient was given a single dose of rectal diazepam by the mother, and emergency medical services were called to the patient's home. There was no apnea or loss of consciousness noted prior to the paramedics arriving at the scene. The patient was provided with oxygen and a fluid bolus was administered. The patient was then taken to a local emergency department where he continued to have tonic-clonic seizure activity. The patient was intubated for airway protection following multiple doses of benzodiazepines to control the seizures. Before transfer to your hospital's pediatric intensive care unit, the patient underwent a brain computed tomography scan, which did not reveal any structural pathology. In the pediatric intensive care unit, the child was given a loading dose of fosphenytoin and sedated to tolerate ventilation with a low-dose midazolam infusion. Continuous electroencephalogram monitoring was initiated. The patient is now on pressure support ventilation with a low fraction of inspired oxygen (FiO2) of 0.25 and an oxygen saturation of 98%. The heart rate is 100 beats/min, spontaneous respirations are in the mid 20's, and the blood pressure is normal. The patient is currently afebrile. Physical examination of the heart, lungs, and abdomen are unremarkable. A limited neurological examination reveals a sedated patient with pupils that are round, equal and reactive, appropriate cough, and gag reflexes, and a normal response to painful stimulation as well as normal deep tendon reflexes. The serum comprehensive metabolic panel is normal. A complete blood count has increased white blood cells with no left shift and is otherwise unremarkable. The arterial blood gas done in the pediatric intensive care unit shows: pH 7.29, PCO2 40, PaO2 95 and BE of -3. Serum lactate sent at the outside hospital is 7.5 mmol per liter (normal 0.5-2.2 mmol per liter). Of the following, the BEST intervention at this time is: A. measure carboxyhemoglobin B. measure central venous oxygen saturation C. order echocardiogram D. repeat serum lactate

D. repeat serum lactate In the patient described in the vignette the most common reason for the elevated serum lactate are the grand mal seizures. The immediate next step is to repeat the serum lactate and follow a trend. Lactic acid can be drawn from free-flowing venous or arterial blood. It is important to know the upper limit of a normal range for a particular institution as there is considerable variation among laboratories. The patient's history and physical examination are not suggestive of a low flow state such as shock, cardiac arrest, or myocardial depression. Therefore, there is no indication for performing an echocardiogram at this stage. Furthermore, the central venous oxygen saturation is an invasive test (ie, requires placement of a central venous line) and is not warranted in a patient who has no evidence of shock or hypoperfusion due to sepsis. A carboxyhemoglobin level is used in carbon monoxide poisoning. There is no history suggestive of exposure to carbon monoxide in this patient. Lactate is produced in the body from pyruvate during anaerobic glycolysis. The reaction is denoted by: Pyruvate + NADH + H+ ↔ lactate + NAD+ The above reversible reaction is catalyzed by an enzyme called lactate dehydrogenase (LDH). The synthesis of lactate is favored giving rise to a lactate-pyruvate ratio of 10:1. Lactate dehydrogenase exists as LDHA (which drives pyruvate to lactate) and LDHB, which drives the reconversion of lactate to pyruvate. Even under normal conditions, the body produces lactate. In the adult, about 20 mmol/kg is produced daily due to muscle activity. The lactate produced in the muscles under aerobic conditions is converted to pyruvate (an LDHB driven reaction) in the liver that ultimately results in the formation of glucose through the Cori cycle. Subsequent metabolism of pyruvate will result in the generation of ATP. Generally, the production and consumption of lactate in the body are identical thus maintaining a steady low normal concentration in the serum 0.5-1.5 mmol/L. Lactate clearance occurs mostly in the liver (70%). Increased serum lactate occurs if the production of lactate surpasses its consumption. Lactate can exist in 2 isomeric forms. The L-isomer is predominant form in humans and is commonly measured by the point of care testing. The D-isomer of lactate is produced by some bacteria in the large intestine of humans and is pathologically insignificant except in short bowel syndrome in which carbohydrate fermenting bacteria produce D-lactate in the large intestine. The usual lactate detecting cartridge used in the pediatric intensive care units measure only the L-isomer and are unable to detect D-lactate. Recent evidence suggests that measurement of serum lactate in pediatric sepsis may have utility in early risk stratification and can predict mortality. Additionally, early elevations of lactate are shown to be associated with increased mortality following resuscitation from pediatric cardiac arrest. Clinicians need to be aware of conditions in which there is hyperlactatemia. Lactic acidosis is conventionally divided into 2 types. Type A lactic acidosis is seen in low flow states where tissues are deprived of oxygen delivery (eg, cardiac arrest, shock). Type B lactic acidosis is seen in conditions of normal oxygen delivery, such as medications impairing oxidative phosphorylation pathways, or in conditions that increase gluconeogenesis, such as use of high dose albuterol in asthma. Conditions that give rise to hyperlactatemia are shown in Table. Trending serum lactate is important for prognosis and to guide therapy especially in sepsis, low cardiac output states, cardiac failure, and acute ingestions. In low flow states the main problem is decreased delivery of oxygen (ie, product of oxygen content and cardiac output) to the tissues resulting in anaerobic glycolysis and generation of lactate (either from localized or global hypoxia). The lactate produced by anaerobic glycolysis would normally be consumed by the liver, but in low flow states decreased removal by the liver results in increased serum lactate level. Any associated acidemia or hypoxia can contribute to the decreased clearance by the liver. Additionally, in shock states, there is a massive release of epinephrine that stimulates the β-adrenoreceptor resulting in the generation of lactate. The hyperlactatemia in low cardiac output states can, therefore, result from anaerobic as well as aerobic glycolysis. In the case of acute ingestions due to alcohols such as ethylene glycol, salicylates, cyanide, or the use of drugs like propofol or metformin, the lactate is primarily generated due to interference with the oxidative phosphorylation pathways. High drug levels (of metformin), prolonged exposure to propofol infusions, the elevated osmolal gap (in ethylene glycol poisoning) could be clues to the cause of increased serum lactate. Additionally, propylene glycol used in medications such as lorazepam or pentobarbital can result in hyperlactatemia as the metabolism of these drugs can result in the production of L and D isomers of lactate without interference with oxidative phosphorylation. Diabetes, chronic liver disease, or cancers such as leukemias or lymphomas can also give rise to increased serum lactate. Many types of cancer cells are programmed to use aerobic glycolysis (ie, generating ATP from lactate) regardless of the presence or absence of oxygen. This is a condition called the Warburg effect. Prolonged total parenteral nutrition can lead to a deficiency of pyruvate dehydrogenase giving rise to decreased clearance of lactate by the liver. Lactic acidosis can exist without a change in the patient's acid-base balance or the anion gap. PREP Pearls Increased serum lactate occurs not only in low flow states such as shock, sepsis, and cardiac arrest, but also due to other causes such as seizures, shivering, and vigorous exercise. Increasing serum lactate level in patients with low flow states is associated with increased mortality. Lactic acidosis is pediatric sepsis occurs due to anaerobic as well as aerobic glycolysis. The latter is probably caused by epinephrine stimulation of the glycolytic flux. Lactate dehydrogenase exists as LDHA, which drives pyruvate to lactate, and LDHB, which drives the reconversion of lactate to pyruvate. ABP Content Specifications(s)/Content Area Understand the mechanisms of lactic acidosis Suggested Readings Fine-Goulden MR, Durward A. How to use lactate. Arch Dis Child Educ Pract Ed. 2014;99(1):17-22. doi: 10.1136/archdischild-2013-304338 Kraut JA, Madias NE. Lactic acidosis. N Engl J Med. 2015;372(11):1078-9. doi: 10.1056/NEJMc1500327 Tsang R. Hemodynamic monitoring in the cardiac intensive care unit. Congenit Heart Dis. 2013;8(6):568-75. doi: 10.1111/chd.12148

A previously healthy 13-year-old girl developed fulminant hepatic failure after an intentional acetaminophen overdose. She received a whole-liver liver transplant from a size matched donor and is being maintained on tacrolimus. Both donor and recipient had matched ABO blood groups, were cytomegalovirus positive, and human immunodeficiency virus negative. The recipient was up to date with her vaccinations, including measles-mumps-rubella and varicella. Her central venous catheter placed during surgery had been removed before discharge from the hospital. She had been receiving oral prophylaxis with trimethoprim-sulfamethoxazole. She now presents to the emergency department 3 months posttransplant with persistent fever above 38.5°C for 10 days, malaise, myalgia, and cough for the past 3 days. Physical examination is negative for any new findings. Her oxygen saturation is 98% in room air. Extensive investigations do not show any evidence of rejection, though she does have both leukopenia and thrombocytopenia. The respiratory viral panel is negative. Of the following, the most likely etiology for her condition is: A. cytomegalovirus B. herpes simplex virus C. Pneumocystis jiroveci D. staphylococcal bacteremia

A. cytomegalovirus Cytomegalovirus (CMV) infection can occur through activation of latent infection or acquired infection from a CMV positive donor. The risk is greatest for severe disease when the donor is CMV positive and the recipient CMV negative, but CMV infection can occur even in a CMV seropositive recipient. Cytomegalovirus infection usually present between 1 and 12 months after solid organ transplantation. As described in the patient in the vignette, CMV infection can manifest with nonspecific symptoms of a viral syndrome such as fever (ie, temperature greater than 38°C for at least 1 week), myalgia, and malaise. These are associated with leukopenia and thrombocytopenia. Atypical lymphocytes may be seen on the blood smear and CMV may be isolated from peripheral blood cells. It can also present with disseminated infection, or single organ involvement such as CMV retinitis, hepatitis, or pneumonitis. The lungs, liver, and gastrointestinal tract are frequent targets of CMV infection irrespective of the organ transplanted. Prophylaxis with antiviral agents such as ganciclovir can reduce the incidence of posttransplant CMV infection but does not prevent it entirely. The diagnosis of CMV infection can be confirmed by quantitative nucleic acid based or CMV pp65 antigenemia, viral load assays, histopathology, or culture. Treatment with IV ganciclovir in conjunction with reduction of immunosuppression can lead to a clinical response within 5-7 days. Duration of therapy is based on clearance of CMV viral load seen on quantitative assay. Due to an immunosuppressed state from anti-rejection medications, opportunistic infections can occur at any time in solid organ transplant recipients. There is a characteristic temporal pattern when certain categories of infection occur more commonly. Bacterial infections related to the surgery, or wound infections from bacteria or yeast typically present within the first month after transplantation. Opportunistic infections with latent pathogens present in the recipient or derived from the donor occur by 12 months, and community-acquired viral infections and other infections associated with chronic graft dysfunction mainly occur after 12 months (Figure). Risk factors that predispose the patient to infections can be divided into those are present before transplantation, those related to the transplantation, or occur posttransplant. Pre-transplant conditions such as a malnourished state, colonization with resistant organisms (such as pseudomonas in a cystic fibrosis patient), prolonged hospitalization, and antibiotic exposure, all put the patient at risk for infection after solid organ transplantation. Younger age predisposes recipients to primary infections with viral pathogens such as herpes simplex virus (HSV), CMV, Epstein-Barr virus (EBV), and varicella since they may not have been exposed to or vaccinated against these agents. Donor organs containing latent infections with CMV, EBV, and Toxoplasma can be passed to the recipients and may cause severe disease in a naive recipient. Operative factors such as a long cold ischemic time (from organ procurement to transplantation) predispose the recipient to infections. The type of surgery (renal, liver, cardiac transplantation) affects the location and type of infection in the immediate posttransplant period. For example, cardiac recipients are at risk for infection with Toxoplasmosis gondii. The most crucial postoperative factor is the type and degree of immunosuppression. For example, patients receiving antilymphocyte products are at greater risk for CMV infection. Seropositivity for viruses such as EBV, CMV, as well as the use of indwelling catheters, need for recurrent surgery and nosocomial infections also play a part in the occurrence of opportunistic infections in the posttransplant phase. As discussed above, bacteremia is unlikely after the first month following solid organ transplantation, particularly without a predisposing risk factor such as an indwelling central venous catheter. Herpes simplex virus can cause a primary infection or affect the patient through reactivation. The most common clinical presentation of HSV is orolabial, genital, or perianal disease with vesicular and/or ulcerative lesions. Disseminated HSV infection may present with esophagitis, hepatitis, or pneumonitis. The patient in the vignette does not exhibit manifestations consistent with HSV infection. Pneumocystis jiroveci infection presents with fever, hypoxia, and lower airway disease. While the patient described had cough for 3 days, hypoxemia was not present, essentially excluding the possibility of Pneumocystis jiroveci infection. Prophylaxis with trimethoprim-sulfamethoxazole has virtually eliminated Pneumocystis jiroveci infection in transplant recipients. PREP Pearls Bacterial and yeast infections related to the surgery and surgical wound occur in the first month after transplantation. Opportunistic infections with latent pathogens present in the recipient or derived from the donor (eg, cytomegalovirus, Epstein-Barr virus, herpes simplex virus) occur between 1 and 6 months. Community-acquired viral infections occur during the late posttransplant period after 6 months. ABP Content Specifications(s)/Content Area Recognize the solid organ transplant recipient at risk for opportunistic infection Know the natural history of opportunistic infection in a solid organ transplant recipient Plan appropriate evaluation for suspected opportunistic infection in a solid organ transplant recipient Suggested Readings Green M, Michaels MG. Infections in pediatric solid organ transplant recipients. J Pediatric Infect Dis Soc. 2012;1(2):144-151. doi: 10.1093/jpids/pir001 Keough WL, Michaels MG. Infectious complications in pediatric solid organ transplantation. Pediatr Clin North Am. 2003;50(6):1451-1469.

A 6-year-old boy is admitted to your pediatric intensive care unit after sustaining a closed head injury. This child was riding a bicycle and was struck by a vehicle traveling approximately 40 miles per hour. He was found unresponsive with no pulse noted. Closed chest compressions were initiated and he received 3 doses of epinephrine en route to the hospital. Continued resuscitation efforts ensued on arrival to the hospital. Return of spontaneous circulation occurred after multiple doses of epinephrine, sodium bicarbonate, and atropine. He is currently being maintained on an epinephrine infusion at 0.1 μg/kg/min. Computed axial tomography scan of his head shows significant cerebral edema with effacement of the ventricles. There is loss of gray white differentiation. Subdural blood is noted in the frontal and parietal regions. An intracranial pressure monitor is placed with an opening pressure of 40 mm Hg. Despite administration of 3% saline, intracranial pressures remain elevated and a pentobarbital infusion is started. On examination vital signs reveal a blood pressure of 94/45 mm Hg, heart rate is 130 beats/min, and temperature is 34.7?. Multiple contusions and abrasions are noted. The boy has a Glasgow Coma Scale Score of 3. Pupils are fixed and dilated. There is no spontaneous respiratory effort, cough, corneal, or gag reflexes noted. He is mechanically ventilated. Hemodynamics reveal palpable pulses with extremities that are cool to touch and capillary refill of 4 seconds. Therapeutic hypothermia protocol is initiated at the request of trauma surgery to keep this child's temperature at 32? for the next 24 hours. Fentanyl and midazolam infusions are initiated during the hypothermia protocol with continuation of pentobarbital. On hospital day 3 he is warmed to 37? and his neurologic status remains unchanged. Fentanyl, midazolam, and pentobarbital have been discontinued. The boy continues to require an epinephrine infusion for blood pressure support. Testing for neurologic death is initiated. The parents have expressed interest in organ donation. Of the following, which conditions must be fulfilled before organ donation can proceed: A. a single brain death examination and apnea test to declare death B. adequate time for drug metabolism following therapeutic hypothermia before conducting a brain death examination and apnea test C. transcranial Doppler ultrasonography and 2 brain death examinations D. two brain death examinations and a single apnea test

B. adequate time for drug metabolism following therapeutic hypothermia before conducting a brain death examination and apnea test The foundation of organ recovery for transplantation is based on the dead donor rule that states: 1) patients must be declared dead before removal of vital organs for transplantation can occur, and 2) recovery of organs for transplantation cannot result in the death of the donor. Accurate determination of neurologic death must occur prior to recovery of organs for transplantation. Determination of neurologic death allows the process of organ preservation and preparation to begin if donation is planned; and if donation is not planned, medical therapies can be discontinued allowing redistribution of intensive care unit resources for other critically ill patients. The 1987 Guidelines for the determination of brain death in children were revised in 2011 and provided more detailed guidance emphasizing neurologic death determination as a clinical process with specific criteria consistent across the age spectrum. This multi-society guideline specifically defined pediatric patients as infants greater than 37 weeks gestation to 18 years of age. No recommendations were made based on insufficient evidence for preterm infants less than 37 weeks gestation. An accurate clinical history is essential to determine cause of coma and ensure the neurologic injury is irreversible. The guideline recommended waiting at least 24 hours prior to initiating testing for neurologic death following cardiopulmonary arrest or after a catastrophic brain injury. Two separate neurologic examinations performed by 2 different attending physicians, and 2 apnea tests separated by an observation period were recommended in the revised guideline. Apnea testing could be performed by the same physician who should be skilled at resuscitation should the patient deteriorate during apnea testing. Physical examination criteria to determine neurologic death includes co-existence of apnea and a known cause of coma. Physical examination criteria consistent with absence of neurologic function includes: mid-position or fully dilated nonreactive pupils, absence of spontaneous eye movements when stimulated by oculocephalic or oculovestibular testing, absence of cough, corneal, gag, and rooting reflexes, and absence of respiratory effort when apnea testing is performed. Irreversible neurologic injury and death can be more difficult to determine in younger patients. The revised guidelines emphasized caution in this age group resulting in age-based recommendations with an extended observation period of 24 hours between examinations and apnea testing for the infant 37 weeks gestation to 30 days of age. An observation period between examinations and apnea testing of 12 hours was recommended for children older than 30 days to 18 years of age. The recommended period of observation could be reduced if an ancillary study was performed and consistent with neurologic death. Prerequisite criteria are required prior to initiating the neurologic examination or neurodiagnostic testing to avoid interference with confounding variables that could lead to diagnostic error. Normal physiologic parameters should be restored and maintained prior to and during the determination of neurologic death. The patient must be normothermic with a core body temperature > 35? and normotensive for age. Correction of hypothermia, hypotension, metabolic and electrolyte disturbances, and ensuring adequate clearance of sedative and neuromuscular blocking agents are essential to avoid diagnostic error when making a determination of neurologic death. Conditions that can interfere with the neurologic examination and are capable of imitating neurologic death, or may play a role in the clinical presentation of the comatose infant or child must be excluded. These include severe hepatic or renal dysfunction, inborn errors of metabolism, metabolic disturbances, or toxic ingestions. Delayed drug metabolism can occur in patients with hepatic or renal dysfunction and should be considered prior to initiating testing for neurologic death. Hypothermia protocols continue to be used as a treatment following cardiac arrest and traumatic brain injury despite lack of supportive evidence. Hypothermia can affect drug metabolism and the clinical examination to determine neurologic death of a critically ill patient. Adequate time following patient rewarming after therapeutic hypothermia has been induced is crucial before attempting to initiate testing for neurologic death. Current pediatric guidelines recommend a minimum core body temperature of 35°C or greater. Clinicians should be aware that therapeutic hypothermia depresses central nervous system function and can alter drug metabolism creating potential for diagnostic error. No specific recommendation was made regarding an observation period following rewarming in the revised pediatric guidelines. Many hospitals and organ procurement organizations have arbitrarily determined a 24-hour period of observation following rewarming after induced hypothermia. Reports in the literature suggest that a longer period of observation following rewarming may be warranted to determine neurologic status of patients treated with therapeutic hypothermia. An ancillary study such as radionuclide cerebral blood flow study can be helpful to the clinician attempting to determine neurologic death in this patient population. Radionuclide cerebral blood flow study can be used in patients treated with high-dose barbiturate therapy to demonstrate absence of cerebral blood flow. Electroencephalography is not recommended because hypothermia and high-dose sedative agents can directly affect study results. Therapeutic hypothermia is commonly used in newborns with neonatal encephalopathy and clinicians should be aware of altered drug metabolism and lack of sensitivity of ancillary studies in this age group when making a determination of neurologic death. Caution is advised in the neonate treated with therapeutic hypothermia. A minimum period of normothermia exceeding 24 hours may be required before initiating testing for neurologic death to avoid diagnostic error. Patients must be declared legally dead and appropriate documentation that the patient meets criteria for neurologic death must be completed before organ donation can proceed. The updated guidelines for determination of neurologic death in infants and children recommend use of the incorporated guidelines checklist. This checklist was developed to provide standardization and assist providers with the process to determine and document neurologic death in children. The learner is encouraged to become familiar with state and local policies and guidelines for determination of neurologic death in their respective institution. PREP Pearls Determination of neurologic death should be approached with caution when the patient has been treated with a therapeutic hypothermia protocol. Hypothermia and end organ dysfunction or failure can alter drug metabolism and affect neurologic prognostication resulting in diagnostic error. Current guidelines recommend minimum requirements for the determination of brain death in infants and children. These criteria must be met before organ donation can occur. ABP Content Specifications(s)/Content Area Know the relationship between the diagnosis of brain death and organ donation Suggested Readings Martin DE, Nakagawa TA, Siebelink M, et al; Transplantation Society. Pediatric deceased donation - a report of the Transplantation Society Meeting in Geneva. Transplantation. 2015;99(7):1403-1409. Mulder M, Gibbs HG, Smith SW, et al. Awakening and withdrawal of life-sustaining treatment in cardiac arrest survivors treated with therapeutic hypothermia. Crit Care Med. 2014;42(12):2493-2499. doi: 10.1097/CCM.0000000000000540. Nakagawa TA, Ashwal S, Mathur M, et al; Society of Critical Care Medicine; Section on Critical Care and Section on Neurology of the American Academy of Pediatrics; Child Neurology Society. Guidelines for the determination of brain death in infants and children.: an update of the 1987 task force recommendations. Criti Care Med. 2011;39(9):2139-2155. doi: 10.1097/CCM.0b013e31821f0d4f. Nakagawa TA, Shemie SD, Dryden-Palmer K, Parshuram CS, Brierley J. Donation following neurologic and circulatory determination of death. Death and dying supplement. Pediatr Crit Care Med. 2018. In Press. Nakagawa TA. The process of organ donation and pediatric donor management. In: Fuhrman BP, Zimmerman JJ. Fuhrman Textbook of Critical Care Medicine 5th ed. Philadelphia, PA: Elsevier; 2016..

A 14-year-old male patient weighing 45 kg was admitted to the hospital with a 6-month history of weight loss. After a comprehensive evaluation he received a diagnosis of an eating disorder, and nasogastric feedings were instituted. After 4 days, the patient was transferred to the pediatric intensive care unit with an acute change in his mental status and generalized weakness. The patient is able to answer your questions and indicates that he is oriented to place and person but not time. He complains of peripheral paresthesias. His physical findings are notable for a respiratory rate of 20 breaths/min, heart rate of 60 beats/min, normal sinus rhythm, blood pressure of 90/50 mm Hg, and pulse oximetry of 98% on room air. On auscultation you note good air entry with no adventitious sounds. Perfusion is good with strong distal pulses and normal heart sounds. Results of the abdominal examination are normal. The patient is diffusely weak in both large and small muscle groups without fasciculations. There are no sensory deficits noted, and deep tendon reflexes are intact. Of the following, the DEFINITIVE laboratory test is A. arterial pH B. serum calcium level C. serum phosphorus level D. serum potassium level

C. serum phosphorus level This patient has hypophosphatemia due to refeeding syndrome. Phosphorus is a prevalent intracellular anion and is crucial to cellular membrane structure, energy storage, and transport in all cells. Serum phosphorus levels are determined by the balance between intestinal absorption of ingested phosphorus, cellular redistribution, and urinary excretion. Decreased intestinal absorption, transcellular shifts from extracellular space to intracellular compartment, and increasing urinary losses all contribute to hypophosphatemia and have a wide variety of causes. Decreased intestinal absorption occurs primarily as a consequence of decreased intake or dietary supplementation, inhibition of phosphorus absorption, chronic diarrhea, or vitamin D deficiency. Obtaining a clear history regarding dietary intake, presence of excessive stool losses or steatorrhea, and coingestion of antacids or phosphate binders should provide clues. Iatrogenic causes include inadequate provision of multivitamins, low phosphate in parenteral fluid composition for parenteral nutrition-dependent patients, or excess removal of phosphorus during continuous renal replacement therapies. Internal redistribution occurs secondary to transcellular shifts of phosphorus from the extracellular compartment to the intracellular space. This is seen during hypersecretory insulin states, such as refeeding syndrome, or is associated with respiratory alkalosis. Increases in the intracellular pH stimulate glycolysis, producing phosphorylated glucose precursors, consuming intracellular phosphorus, and thereby shifting phosphorus intracellularly, resulting in hypophosphatemia. Any condition associated with respiratory alkalosis such as diabetic ketoacidosis, sepsis, pain, anxiety, encephalopathy, or drug ingestions will affect serum phosphorus levels. In anabolic states such as refeeding, patients experience severe hypophosphatemia because of insulin-stimulated intracellular movement of glucose and phosphorus in response to administration of carbohydrates. This is often superimposed on a background of severe phosphorus deficiency due to decreased dietary intake. Pathologic increased urinary excretion of phosphorus resulting in hypophosphatemia is associated with derangements in parathyroid hormone secretion, renal tubular defects, or medication effects. Primary hyperparathyroidism results in the triad of hypercalcemia, hypophosphatemia, and increased fractional excretion of phosphorus. If the serum calcium level is normal, one should consider vitamin D deficiency or resistance. In the presence of decreased calcium levels, the stimulation for hypersecretion of parathyroid hormone occurs in secondary hyperparathyroidism. In specific circumstances seen after kidney transplantation, with residual increased parathyroid levels, there is persistent hypophosphatemia associated with increased urinary phosphorus excretion. Renal tubular defects resulting in increased phosphorus excretion are either due to isolated problems with phosphate transporter (eg, hereditary hypophosphatemic rickets, tumor-induced osteomalacia) or due to generalized problems with reabsorption. An example of generalized problems with reabsorption, Fanconi syndrome, also manifests glucosuria, aminoaciduria, and bicarbonate wasting in the urine. A normal response to decreased serum phosphorus levels is to increase the absorption of filtered phosphorus via the sodium-phosphate transporter located in the luminal membrane of the proximal tubule. This occurs via induction of receptor gene expression, with the result of newly synthesized transporters. If the diagnosis is not readily available by history and physical examination, diagnostic evaluation includes measurement of urinary excretion of phosphorus by either a 24-hour urine collection or determination of fractional excretion of phosphorus based on a random urine sample. If the 24-hour urine phosphorus excreted is below 100 mg or if the fractional excretion of phosphorus is less than 5%, there is appropriate conservation of phosphorus by decreased urinary excretion, and the cause is either internal redistribution or intestinal malabsorption. If the 24-hour urine phosphorus excretion is more than 100 mg or the fractional excretion of phosphorus is greater than 5%, phosphate wasting through the kidney is occurring. Signs and symptoms of hypophosphatemia include CNS disturbances ranging from confusion and paresthesias to seizures and coma. Weakness is observed in skeletal muscles, initially in large muscle groups, and may be missed as a cause of mild respiratory embarrassment. Decreased 2,3-DPG in red blood cells may shift the oxygen dissociation curve to the left, resulting in tissue hypoxia with development of metabolic acidosis. Hypokalemia demonstrates muscle weakness and paresthesias; however, at these levels, there are also observed electrocardiographic changes such as U waves, T-wave flattening, and inversion as well as dysrhythmias and usually no accompanying changes in mental status. The observed dysrhythmias include supraventricular and ventricular ectopy, which can progress to supraventricular tachyarrhythmias such as atrial flutter and fibrillation and ventricular tachyarrhythmias such as ventricular tachycardia, ventricular fibrillation, and torsade de pointes. Hypocalcemia is associated with paresthesias and muscle cramps, which may progress to tetany, at which point there is also observed bronchospasm, dysphagia, and laryngospasm. Altered mental status changes include confusion, seizures, and hyperreflexia. Electrocardiographic findings include prolonged QT interval due to prolonged ST segment; however, dysrhythmias are uncommon. Treatment of hypophosphatemia should first and foremost target treating the underlying cause, in particular intestinal malabsorption, because this will affect the route of phosphorus replacement. Management of hypophosphatemia is predicated by the patient's signs and symptoms as well as the serum phosphorus level. Symptoms do not generally appear unless the level is less than or equal to 1 mg/dL (0.32 mmol/L); however, there may be some subclinical weakness at levels less than 2 mg/dL (0.64 mmol/L). Parenteral repletion is recommended for symptomatic patients with a serum phosphate level less than 1 mg/dL, and enteral repletion is recommended in other scenarios. Phosphorus formulations contain either sodium or potassium as the cation, and generally sodium-containing formulations are favored if the serum potassium value is normal to elevated. Parenteral formulations should be used with caution because they can be associated with the development of hypocalcemia and precipitation of calcium-phosphate renal calculi. PREP Pearls Hypophosphatemia results from decreased intestinal absorption and increased urinary excretion or transcellular shifts of phosphate from the extracellular space to the intracellular milieu. History and physical findings provide the cause in most cases, but measurement of urinary phosphate excretion will provide the diagnosis. Symptoms of hypophosphatemia are rarely evident unless the serum phosphorus level is less than 2 mg/dL (0.64 mmol/L). Enteral phosphorus repletion is preferred as long as an enteral route can be tolerated and the serum level exceeds 1 mg/dL (0.32 mmol/L). ABP Content Specifications(s)/Content Area Know the causes of hypophosphatemia Plan treatment for a patient with hypophosphatemia Suggested Readings Lynch R, Wood EG, Neumayr TM. Fluid and electrolyte issues in pediatric critical illness. In: Fuhrman BP, Zimmerman JJ, eds. Fuhrman and Zimmerman's Pediatric Critical Care. 5th ed. Philadelphia, PA: Elsevier; 2017:chap 73.

You admit a previously healthy 1-year-old boy from the emergency department who was found unresponsive in his parents' car. He had fallen asleep on the ride home, so his mother left him strapped in his car seat to let him sleep longer. The weather was comfortable but sunny so she did crack the window a bit. When she returned she found him breathing and flushed, but unresponsive. Upon arrival to the emergency department, his initial vital signs were temperature 42℃, heart rate 200 beats/min, respiratory rate 40 breaths/min, and blood pressure 80/30 mm Hg. During his initial assessment, he had a generalized tonic-clonic seizure. Of the following, the MOST effective treatment based on his clinical presentation is: A. acetaminophen B. dantrolene C. evaporative cooling D. ibuprofen

C. evaporative cooling The child in the vignette is suffering from heat stroke and central nervous system dysfunction associated with hyperthermia due to environmental heat exposure. Traditionally, there are 2 types of heat stroke: 1) exertional heat stroke, and 2) nonexertional, or classic, heat stroke. Exertional heat stroke is more likely to occur with young athletes who perform extreme exercise during hot humid weather with limited hydration and rest. Classic heat stroke, on the other hand, occurs with environmental heat exposure but without significant exertion (eg, during heat waves or in an enclosed car where temperatures can rise rapidly even with moderate ambient temperatures). Young children or those with disabilities, who are unable to escape from the hot situation, are more susceptible to classic heat stroke. Children with heat stroke present with elevated core temperatures (> 40℃), an inflammatory state and in severe cases, multi-organ failure. They have tachypnea and tachycardia, often with hypotension due to hypovolemia and vasodilation. The degree of dehydration can be severe with exertional heat stroke, milder with nonexertional heat stroke. Their skin is flushed but may be dry rather than diaphoretic with nonexertional heat stroke. Central nervous system changes, the hallmark of heat stroke, can range from mild (eg, dizziness) with less severe cases to severe (eg, seizures, cerebral edema, coma). Laboratory abnormalities can include metabolic acidosis, electrolyte abnormalities, evidence of disseminated intravascular coagulation, acute liver, and kidney injury. These children may also manifest rhabdomyolysis, with elevated creatinine kinase and myoglobinuria, and progressive anemia during the first 24 to 48 hours. Management of heat stroke includes rapid cooling and support of end-organ injury. Cooling should begin as early as possible to reduce the ongoing exposure to elevated temperature. External cooling methods include evaporative cooling (ie, spraying patients with water, covering with wet sheets and then fanning) and use of cooling blankets or ice packs. Cold water immersion is another option but may not be feasible during ongoing resuscitation and monitoring. Internal cooling methods include administration of room temperature or cooled intravenous fluids, lavage with cold fluids and if available, extracorporeal circulatory support, or cardiopulmonary bypass. However, internal cooling has not been demonstrated to be more effective than evaporative methods. Antipyretics such as acetaminophen and ibuprofen are not effective for reducing body temperatures with heat stroke and may exacerbate other organ injury. Dantrolene, used for treating malignant hyperthermia, is not routinely used for heat stroke. In addition to cooling, managing cardio-respiratory compromise and supportive care is essential for reducing the morbidity and mortality associated with heat stroke. Children with altered mental status or respiratory compromise may require intubation and mechanical ventilation. Fluid resuscitation, especially with exertional heat stroke, is important, and vasopressor medication may be necessary for maintaining adequate blood pressure. If clinically significant bleeding is present, correction of coagulation may be necessary. Benzodiazepines may be useful in treatment of seizures as well as shivering from cooling methods. Appropriate hydration and avoidance of unnecessary nephrotoxins are important for minimizing ongoing kidney injury, especially in the setting of rhabdomyolysis. PREP Pearls Rapid cooling is essential in the management of heat stroke, and can be external cooling (eg, evaporative methods) or internal cooling. Antipyretic medication (eg, acetaminophen, ibuprofen) are not useful for reducing body temperatures in heat stroke. ABP Content Specifications(s)/Content Area Understand the epidemiology of heat stroke and prostration Know the clinical and laboratory manifestations of heat illness Plan the treatment of a patient with heat stroke, including various cooling methods Suggested Readings Hifumi T, Kondo Y, Shimizu K, Miyake Y. Heat stroke. J Intens Care. 2018;6:30. doi: 10.1186/s40560-018-0298-4 Jardine DS. Heat illness and heat stroke. Pediatr Rev. 2007;28(7):249-258 doi: 10.1542/pir.28-7-249

A 13-year-old girl with chronic liver failure, hepatic cirrhosis, and ascites is hospitalized. While awaiting liver transplantation, her hospital course has been complicated by an episode of sepsis, intermittent gastrointestinal bleeding, and 2 therapeutic paracentesis procedures. Her only daily medication is furosemide that has been gradually reduced over the last 3 weeks to 20 mg intravenously twice daily. She remains in the pediatric intensive care unit. Serial abdominal ultrasonography reveals mildly dilated hepatic vessels, moderate ascitic fluid, and no renal parenchymal, obstructive, or renovascular disease. Over a period of a week, there are gradual changes in her laboratory data as follows: - 1 week prior: serum sodium 132 mEq/L serum creatinine of 0.7 mg/dL urine volume: 750 mL/d urine 24-hr protein: negative urine red blood cell: < 50 per hpf - Today: serum sodium 128 mEq/L serum creatinine 2.1 mg/dL urine volume: 400 mL/d urine 24-hr protein: negative urine red blood cell: < 50 per hpf She has not received any recent hepatotoxic or nephrotoxic agents and hemodynamics have remained stable without hypotension or evidence of inadequate tissue oxygen delivery. Despite discontinuing diuretics and applying generous volume resuscitation for 2 days, her creatinine continues to rise and she progresses to renal failure requiring renal replacement therapy. Of the following, her renal pathology is MOST likely the direct result of: A. impaired renal perfusion from intraabdominal hypertension B. postobstructive renal disease C. reduction in diuretic use D. splanchnic arterial vasodilation

D. splanchnic arterial vasodilation The patient described in the vignette developed progressive liver failure after acetaminophen intoxication. Her acute kidney injury, manifested by worsening creatinine and urine output without concurrent shock, nephrotoxic medication exposure, or obstructive/parenchymal renal disease, is characteristic of hepatorenal syndrome. The hepatorenal syndrome is a severe, but potentially reversible cause of renal failure in patients with cirrhosis that carries an associated 3-month survival of 15%. The hepatorenal syndrome occurs infrequently in children, but is an important cause of renal failure in children with liver failure. As chronic liver disease progresses, with increasing portal hypertension and the creation of porto-systemic collateral vessels, greater shear stress or tension on the splanchnic vessels results in up regulation and systemic release of endogenous nitric oxide, a potent vasodilator. This splanchnic vasodilation results in relative systemic arterial hypovolemia. The body compensates with vasoconstriction via sympathetic activation, the renin-angiotensin-aldosterone system, local endothelin production, and nonosmotic release of vasopressin. Renal hypoperfusion follows when the total vasoconstriction overcomes the effects of local vasodilators such as prostaglandins and kallikrein resulting in acute kidney injury and ultimately renal failure. Risk factors for developing hepatorenal syndrome include sepsis (particularly associated with spontaneous peritonitis), variceal bleeding, large volume paracentesis, excessive diuresis, and acute cholestasis. In addition to avoidance of nephrotoxic agents such as IV contrast, certain antibiotics, and nonsteroidal anti-inflammatory medications, clinicians should consider administering 5% albumin to maintain vascular volume when performing therapeutic paracentesis in patients with cirrhotic liver disease. The treatment of hepatorenal syndrome focuses on improving intravascular volume to enhance cardiocirculatory function using volume replacement, and applying vasoconstrictors such as norepinephrine, terlipressin, midodrine, and octreotide for uncontrolled splanchnic arterial vasodilation. Portal hypertension and renal function may be improved by creating a transjugular intrahepatic portosystemic shunt, however many patients may not be candidates for this procedure because of hepatic encephalopathy or liver dysfunction. Ultimately many of these patients require liver transplantation to resolve their hepatorenal syndrome. Shock, sepsis, parenchymal or obstructive renal disease, and nephrotoxic drug exposure must be ruled out as potential causes contributing to the hepatorenal syndrome. Laboratory data consistent with renal insufficiency or failure persist in hepatorenal syndrome even after discontinuation of diuretics and volume resuscitation. PREP Pearls Hepatorenal syndrome is a unique form of acute kidney injury that develops in patients with cirrhotic liver disease. Arterial vasodilation and release of vasoconstrictive agents results in renal vasoconstriction characteristic of hepatorenal syndrome. Different mechanisms contribute to renal vasoconstriction. Liver transplantation is the preferred treatment for hepatorenal syndrome, however other treatments such as renal replacement therapy can be utilized as a bridge to transplantation. ABP Content Specifications(s)/Content Area Recognize the clinical manifestations of hepatorenal syndrome Suggested Readings De Mattos AZ, de Mattos AA, Mendez-Sanchez N. Hepatorenal syndrome: current concepts related to diagnosis and management. Ann Hepatol. 2016;15(4):474-481. Martin-Llahi M, Guevara M, Torre A, et al. Prognostic importance of the cause of renal failure in patients with cirrhosis. Gastroenterology 2011;140(2):488-496. doi: 10.1053/j.gastro.2010.07.043. Piano S, Tonon M, Angeli P. Management of ascites and hepatorenal syndrome. Hepatol Int. 2018;12(Suppl 1):122-134. doi: 10.1007/s12072-017-9815-0. Shah N, Silva RG, Kowalski A, Desai C, Lerma E. Hepatorenal syndrome. Dis Mon. 2016;62(20):364-375. doi: 10.1016/j.disamonth.2016.05.009.

An 8-year-old girl develops severe abdominal pain after climbing a tree, falling, and hurting her abdomen on a tree branch. A radiograph taken in the emergency department reveals free air under the diaphragm. There are no other internal injuries discovered on the initial trauma evaluation. On exploratory laparotomy, a small bowel perforation is found and repaired, and the abdomen is closed. She is brought back to the pediatric intensive care unit still endotracheally intubated, with a central line, arterial line, and urinary catheter in place. Her weight is 30 kg on admission. In the immediate postoperative period, she develops hypotension requiring five 20 mL/kg fluid boluses as well as the addition of an epinephrine infusion at 0.1 μg/kg/min to maintain an adequate blood pressure. On rounds the next morning, the bedside nurse is concerned because the patient's urine output has declined to only 20 mL in the last 6 hours. In addition, the oxygen requirement is up to 70% to maintain oxygen saturation at >92% despite increasing the positive end-expiratory pressure to 10 cm H2O . The end-tidal CO2 is also elevated at 48 mm Hg. The central venous pressure, measured via an internal jugular catheter with a good venous waveform, is 18 mm Hg and current blood pressure is 100/50 mm Hg. On examination, the abdomen is distended, but it is not tense to your palpation. Of the following, the next BEST step is: A. administer additional 20 mL/kg crystalloid bolus B. check how distended the abdomen feels again in 2 hours C. decrease the positive end-expiratory pressure to 8 cm H2O D. measure the intraabdominal pressure

This patient is showing signs and symptoms of abdominal compartment syndrome with elevation in intraabdominal pressure causing adverse effects on hemodynamics, oxygenation, ventilation, and urine output. Measuring the intraabdominal pressure is a key element in the diagnosis of abdominal compartment syndrome since clinical findings on palpation do not correlate well with measured intraabdominal pressure. Intraabdominal pressure is the steady state pressure concealed within the abdominal cavity. It should be measured at end-expiration in mm Hg in the supine position after ensuring that abdominal muscle contractions are absent, and with the transducer zeroed at the level of the mid-axillary line. In mechanically ventilated children, normal intraabdominal pressure, typically measured as via an indwelling bladder catheter, is approximately 4-10 mm Hg, and even lower in newborns (Table). Intraabdominal hypertension is defined by a sustained or repeated pathological elevation in intraabdominal pressure > 10 mm Hg in children. In adults the threshold is ≥ 20 mm Hg. The abdominal compartment is bordered superiorly by the diaphragm, and inferiorly by the pelvic bones and the muscles of the pelvic floor. Posteriorly lie the lumbar vertebrae and back muscles and anterolaterally, the anterior abdominal wall composed of abdominal muscles and fascia. An increase in the volume of abdominal contents, whether it is through bowel wall edema or "third spacing," both of which are probably occurring in the patient in the vignette, leads to progressive rise in intraabdominal pressure and compromised organ function. When organ function is compromised, and attributable to intraabdominal hypertension, then abdominal compartment syndrome is said to have developed. In children, abdominal compartment syndrome is defined as a sustained elevation in intraabdominal pressure > 10 mm Hg associated with new or worsening organ dysfunction. In adults it is defined as a sustained intraabdominal pressure > 20 mm Hg that is associated with new organ dysfunction or failure. Abdominal perfusion pressure is the difference between the mean arterial pressure and intraabdominal pressure (APP = MAP - IAP). In adults, abdominal perfusion pressure is a better predictor of patient outcome than mean arterial pressure, intraabdominal pressure, or other traditional resuscitation endpoints, such as arterial pH, base deficit, arterial lactate, and hourly urinary output. Normative data for abdominal perfusion pressure are not available for children. Since mean arterial pressure values are lower in children, the threshold abdominal perfusion pressure below which organ dysfunction ensues is likely to be lower in children than the 60 mm Hg threshold value used in adults. Risk factors for developing intraabdominal hypertension and abdominal compartment syndrome in children include conditions with: Diminished abdominal wall compliance: gastroschisis, omphalocele, circumferential abdominal wall burns, and abdominal surgery Increase in volume of intraluminal contents: constipation, Hirschsprung disease Increase in contents in the abdominal cavity: massive liver/spleen enlargement, tumors, ascites, bleeding (intra or retroperitoneal) Capillary leak/fluid resuscitation: systemic inflammatory response syndrome, sepsis, massive transfusion The patient in the vignette may have further bowel compromise, bowel wall edema, or free peritoneal fluid related to capillary leak that will need to be dealt with accordingly. Bedside sonography, abdominal CT, or an exploratory laparotomy are all possible avenues to pursue to determine an effective treatment plan. There are 4 basic principles that guide management of abdominal compartment syndrome: Lowering intraabdominal pressure medically and/or surgically Optimizing perfusion Treating the underlying cause Supporting organ dysfunction Additional fluid administration is unlikely to help raise the blood pressure or urine output in the patient in the vignette since the central venous pressure is already high. Intraabdominal pressure increases chest wall elastance and elevates the diaphragm, with reduction in functional residual volume and resultant atelectasis. This increases ventilation-perfusion mismatch resulting in hypoxia and hypercarbia. Increasing the positive end-expiratory pressure (PEEP) may or may not be of help either. In fact, measuring intraabdominal pressure may be an additional tool in adjusting the PEEP. A pilot study in 15 adults compared PEEP at 3 levels applied in a random fashion: PEEP of 5 cm H2O (baseline), PEEP=50% of measured intraabdominal pressure, and PEEP=100% of measured intraabdominal pressure. The highest PEEP was not well-tolerated and only marginally improved oxygenation in ventilated patients with intraabdominal hypertension. In contrast, moderate intraabdominal pressure-adjusted PEEP, applied at 50% of IAP, was well-tolerated hemodynamically and improved respiratory system compliance, but not oxygenation. Abdominal compartment syndrome is associated with 90% to 100% mortality, if not recognized and treated promptly. A high index of suspicion for intraabdominal hypertension in patients with clinical diagnoses or risk factors for developing abdominal compartment syndrome should prompt serial intraabdominal pressure measurements, so that intraabdominal hypertension and abdominal compartment syndrome can be diagnosed and treated in a timely manner. PREP Pearls The intraabdominal pressure threshold at which organ dysfunction can develop is lower in the pediatric population. Physical examination alone is unreliable, and the detection of abdominal compartment syndrome depends on early and serial measurements of intraabdominal pressure in patients at risk for abdominal compartment syndrome. Abdominal surgery and fluid resuscitation are risk factors for development of abdominal compartment syndrome. ABP Content Specifications(s)/Content Area Recognize the signs and symptoms of abdominal compartment syndrome Suggested Readings Defontaine A, Tirel O, Costet N, et al. Transvesical intra-abdominal pressure measurement in newborn: what is the optimal saline volume instillation? Pediatr Crit Care Med. 2016;17(2):144-1449. doi: 10.1097/PCC.0000000000000580 Ejike JC, Bahjri K, Mathur M. What is the normal intra-abdominal pressure in critically ill children and how should we measure it? Crit Care Med. 2008;36(7):2157-2162. doi: 10.1097/CCM.0b013e31817b8c88 Kirkpatrick AW, Roberts DJ, De Waele J, et al. Intra-abdominal hypertension and the abdominal compartment syndrome: updated consensus definitions and clinical practice guidelines from the World Society of the Abdominal Compartment Syndrome. Intensive Care Med. 2013l;39(7):1190-1206. doi: 10.1007/s00134-013-2906-z Regli A, De Keulenaer BL, Palermo A, van Heerden PV. Positive end-expiratory pressure adjusted for intra-abdominal pressure - A pilot study. J Crit Care. 2018;43:390-394. doi: 10.1016/j.jcrc.2017.10.012

A 4-year-old fully immunized boy is brought to the emergency department in July for weakness in his bilateral upper extremities. The boy also complained of headache, neck pain, and difficulty swallowing for the 2 days before presenting to the emergency department. Further history reveals that the family was on a camping trip a week earlier, where the boy had upper respiratory symptoms for a few days, including a low-grade fever that has subsequently resolved. The patient did not have a rash or any ticks on his body. Patient's vital signs are normal except for a low-grade fever; room air oxygen saturation is 100%. Physical examination reveals an awake, alert, and normally-developed child with bilateral weakness of his upper extremities (strength ~ 0/5-1/5 in both), absent reflexes in arms, and normal strength in the lower extremities. Patient has no cerebellar signs. Cranial nerves are intact except for weakness of left face resembling Bell's palsy. The Kernig and Brudzinski signs were negative. The rest of the physical examination reveals no organomegaly, rash, or lymphadenopathy. After consultation with the neurology service, an urgent magnetic resonance imaging of the brain and spine is done (Figure). The emergency department physician requests a pediatric intensive care unit bed for this patient. Additionally, a lumbar puncture done in the emergency department showed colorless cerebrospinal fluid with an elevated white blood cell count of 56 cell/μL (with lymphocytes 21%, neutrophils 52%, and monocytes 26%); cerebrospinal fluid protein of 58 mg/dL and cerebrospinal fluid glucose 64 mg/dL. The cerebrospinal fluid gram stain was negative and appropriate cultures were sent. A complete blood cell count showed: white blood cell 15.13 X 1000/ μL, with 69% segmented neutrophils, 24% lymphocytes, and 6% monocytes. A comprehensive metabolic panel was unremarkable. Viral studies from cerebrospinal fluid and serum (Enterovirus, West Nile, Eastern and Western Equine encephalitis, St Louis and California Encephalitis) were sent. The human immunodeficiency viral studies were negative. Treponema pallidum was nonreactive in the cerebrospinal fluid and serum. Rocky Mountain spotted fever and Ehrlichia titers were negative. The patient was started on vancomycin, ceftriaxone, doxycycline, and acyclovir. Of the following, the MOST likely reason for this boy to require the pediatric intensive care unit is the development of: A. acute respiratory failure B. hypertensive crises C. intracranial hypertension D. status epilepticus

A. acute respiratory failure Acute flaccid myelitis (AFM) is a rare polio-like illness defined by the acute onset of flaccid paralysis in the setting of spinal magnetic resonance imaging (MRI) demonstrating a longitudinal lesion in the gray matter of the cord. Since 2014, the Centers for Disease Control and Prevention (CDC) has investigated and confirmed approximately 537 cases of AFM occurring mostly in children. No pathogen was implicated in any of the cases except for 4 patients who had Coxsackievirus A16, Enterovirus-A71, or Enterovirus-D68 in their cerebrospinal fluid. Most cases presented during August to October, a peak time for enteroviruses, which would increase the likelihood of the implication of these group of viruses with AFM. These cases have occurred in 48 states as well as in the District of Columbia. The majority of patients had a preceding fever or viral respiratory illness before the onset of weakness. Any patient with symptoms suggestive of AFM needs to be reported to the local health department, which in turn will send the information to the CDC. The Council of State and Territorial Epidemiologists adopted the CDC's standard case definition of AFM in 2015, which was subsequently updated in 2017. The case definition (https://www.cdc.gov/acute-flaccid-myelitis/hcp/case-definition.html) is used by neurologists and other experts to classify a case as "confirmed," "probable," or "not a case." The case definition should not be used for the decision whether to report a case to the health department, and any patient suspected of AFM must be promptly reported. A patient presenting with acute flaccid limb weakness along with MRI findings in spinal gray matter spanning one or more vertebral segments and a cerebrospinal fluid (CSF) pleocytosis (white blood cells > 5 cells/mm3) will be confirmed to have AFM. A negative or normal MRI in the first 72 hours after the onset of limb weakness does not rule out the diagnosis of AFM. Acute flaccid myelitis is an umbrella term that includes acute transverse myelitis, Guillain-Barre syndrome, and other muscle disorders. Acute transverse myelitis is an immune-mediated inflammatory lesion of the spinal cord (usually seen postinfection). The patient often presents with a distinct area on the skin (transverse band) overlying the spine that is tender/sensitive to touch. Below the sensory level, besides the loss of sensation (ie, temperature, touch), the affected muscles are spastic, and furthermore, patients can experience bowel and bladder dysfunction. In patients with AFM, there is no loss of sensation, and the bowel and bladder functions are usually intact (as AFM only affects the gray matter of the spinal cord and not the white matter). In Guillain-Barre syndrome, the patient presents with an ascending paralysis and a distinct albumin-cytological disproportion in the CSF, but with normal brain and spine MRI imaging. Although acute demyelinating disseminated encephalomyelitis (ADEM) may include spinal cord involvement, the patient usually presents with altered mental status in acute demyelinating disseminated encephalomyelitis as opposed to a normal mental status in AFM. Patients with neuromyelitis optica usually present with longitudinal cord edema in association with anti-aquaporin 4 antibodies in the serum or the CSF. Although features of acute spinal cord infarction may mimic AFM, patients with acute spinal cord infarction may have a history of trauma or risk factors for stroke (such as sickle cell disease), and usually do not have fever or a preceding viral infection. The CSF studies are normal and the MRI demonstrates positive diffusion-weighted imaging, and significant enhancement of the affected part of the spinal cord. The diagnosis of AFM requires a good history, physical examination, MRI (which should be repeated if negative or normal in the first 72 hours), lumbar puncture, and sometimes a nerve conduction study to evaluate for the presence of a lower motor neuron lesion. The CSF obtained should be sent for cytology, glucose and protein, gram stain, and viral studies. Serum, stool, and respiratory viral studies should be sent at the earliest time possible (Table). Consultation with infectious disease, as well as a neurologist, will be helpful to have a team approach to this disease process. There is no definitive therapy for AFM, and the CDC has not made any specific therapeutic recommendations. Plasma exchange, intravenous immune globulin, steroids, as well as other immunosuppressive medications can be tried on a case-by-case basis. Some centers have tried fluoxetine (for its anti-enterovirus D68 activity) although data on outcomes with this therapy are not currently available. Early attention to rehabilitation is recommended to preventing muscle wasting and atrophy. Some centers have tried nerve transfer surgery in patients who have unilateral limb involvement in which a nerve from a less essential muscle group is used to innervate the muscles of the affected extremity. There are no data on outcomes or recommendations as to the timing of this procedure. The patient described in the vignette meets the case definition of AFM. Patients with bulbar symptoms (ie, drooling, difficulty swallowing, slurred speech) are at the highest risk for sudden onset of acute respiratory failure due to the weakness of the respiratory muscles and hence must be observed in the pediatric intensive care unit. Intubation and mechanical ventilation may be necessary to protect airway and support ventilation. Negative inspiratory force can be used to trend the onset of weakness of the respiratory musculature and the need for intubation. Patients who are ventilated may need a tracheostomy to facilitate early mobility and rehabilitation. As the disease process does not affect the cerebral cortex, the patient is not at risk for status epilepticus. A patient who is alert and awake with no space-occupying lesion seen on the brain MRI makes the presence of intracranial hypertension less likely. Although the patient with transverse myelitis can present with autonomic dysfunction resulting in hypertensive crises and cardiac rhythm abnormalities, it is less likely in patients with AFM. In at least half of the patients with AFM the muscle weakness may be permanent. PREP Pearls Acute flaccid myelitis is an umbrella term that includes acute transverse myelitis, Guillain-Barre syndrome, and other muscle disorders. The magnetic resonance imaging that classically shows grey matter changes in the spinal cord may be absent in the first 72 hours after the onset of weakness in the patient. Patients with bulbar symptoms (ie, drooling, difficulty swallowing, slurred speech) are at the highest risk for sudden onset of acute respiratory failure due to the weakness of the respiratory muscles and hence must be observed in the pediatric intensive care unit. ABP Content Specifications(s)/Content Area Acute flaccid myelitis Suggested Readings Centers for Disease Control and Prevention. Acute flaccid myelitis in US children. https://www.cdc.gov/features/acute-flaccid-myelitis/index.html Centers for Disease Control and Prevention. AFM investigation. https://www.cdc.gov/acute-flaccid-myelitis/afm-surveillance.html Hopkins SE. Acute flaccid myelitis: etiologic challenges, diagnostic and management considerations. Curr Treat Options Neurol. 2017;19(12):48. doi: 10.1007/s11940-017-0480-3

You are caring for a 13-year-old adolescent boy who has been admitted with kidney transplant rejection. He has been on pulse steroids, but kidney function continues to worsen. His blood pressure has become progressively higher, despite his current oral antihypertensive medications. To better control his blood pressure, you order a different intravenous antihypertensive medication. Of the following, the endogenous factor MOST likely to help maintain appropriate coronary blood flow given prior antihypertensive therapy in this patient is: A. adenosine B. endothelin-1 C. nitric oxide D. prostacyclin

B. endothelin-1 The child in the clinical vignette has hypertension, so the appropriate physiologic response for autoregulation would be to vasoconstrict the coronary blood vessels. Of the choices provided, endothelin-1 was the factor with a vasoconstrictive effect. Although the heart receives only a small fraction of the overall cardiac output (~5%), its oxygen needs are quite high as one of the most metabolically active organs in the body. To account for this mismatch, the arterial oxygen extraction is significantly higher in the coronary vascular bed than in other parts of the body (70%-80% vs 20%-50%). Consequently, the heart has less ability to further increase oxygen extraction in the face of decreased oxygen delivery or increased demand, leaving it at particular risk of ischemic injury. To reduce this risk, there are a number of mechanisms to ensure normal coronary blood flow across a range of perfusion pressures, a phenomenon termed "autoregulation" (Figure 1). Over this range, the coronary arteries vasodilate or vasoconstrict to achieve constant flow by altering the coronary vascular resistance, applying Ohm's law (flow = Δpressure/resistance). Outside of this autoregulated range, the vessels are either maximally dilated or maximally constricted such that flow then become pressure-dependent. The complex mechanisms controlling coronary blood flow have been studied for decades, but remain incompletely understood. In addition to the myogenic response of coronary smooth muscle cells (stretch in the face of increased pressure, relaxation with decreased pressure), the coronary vascular resistance appears to be influenced by neural, hormonal, metabolic, and endothelial signaling (Figure 2). Neural control occurs via the numerous adrenergic and cholinergic receptors distributed along the coronary vasculature. Stimulation of ß-receptors leads to vasodilation, while stimulation of ɑ-receptors tends to result in vasoconstriction. In general, the net effect of sympathetic stimulation is believed to be vasodilation but may be variable depending upon vessel size. Parasympathetic stimulation primarily results in vasodilation. Hypoxic conditions also lead to coronary vasodilation, although it is difficult to separate the direct effects of hypoxia and the local metabolic reaction from the hypoxia. Hypercarbia has a synergistic vasodilatory effect with hypoxia, but alone does not have a significant contribution outside of the response to acidosis. Adenosine, a potent vasodilator, was long thought to play a primary role in controlling vasomotor tone. However, data now suggests adenosine has little influence under physiologic conditions but maintains a strong vasodilating effect under ischemic conditions. In addition to local metabolic influences, the vascular endothelium also influences the coronary vascular resistance by synthesizing and secreting vasoactive substances in response to shear stress and circulating or local factors such as histamine release. Important endothelial-derived vasodilating substances include nitric oxide, prostacyclin, prostaglandin H2, and hydrogen peroxide. Endothelial-derived vasoconstricting substances include endothelin-1, thromboxane A2, and serotonin. Finally, circulating substances can also affect coronary vascular resistance, although the ultimate impact on coronary blood flow depends upon the balance between direct and systemic effects. For example, angiotensin II is a strong vasoconstrictor when directly instilled into the coronary vessels but the overall coronary blood flow may be increased due to increased systemic vascular resistance. Similar effects may be seen with vasopressin. PREP Pearls In resting conditions, oxygen extraction from the coronary vessels is high, resulting in limited ability to further increase extraction when cardiac output decreases or oxygen demands increase. The coronary blood flow is maintained within a normal range across a wide range of pressures, also called autoregulation. Autoregulation is modulated through vasodilation or vasoconstriction controlled by multiple interacting factors including neural, hormonal, metabolic and endothelial. ABP Content Specifications(s)/Content Area Understand the effects of PCO2 and PO2 on myocardial blood flow regulation Understand autoregulation of myocardial blood flow Suggested Readings Goodwill AG, Dick GM, Kiel AM, Tune JD. Regulation of coronary blood flow. Compr Physiol. 2017;7(2):321-382. doi: 10.1002/cphy.c160016 Ramanathan T, Skinner H. Coronary blood flow. Contin Ed Anaesth, Crit Care Pain. 2005;5(2):61-64. https://doi.org/10.1093/bjaceaccp/mki012

An 8 kg, 10-month-old infant boy undergoes bilateral mandibular distraction to treat his micrognathia associated with Pierre-Robin sequence. The mandibular distraction was complicated by a significant amount of bleeding, although there is good postoperative hemostasis. The infant is intubated with a 3.5 mm cuffed endotracheal tube. On request of the plastic surgeon, the infant does not receive additional neuromuscular blockade, but is quite deeply sedated overnight to limit movement. Consequently, the infant requires nearly full mechanical ventilatory support overnight. Furthermore, the surgery team requests that systemic corticosteroids be avoided. On the first hospital day, the sedation is lifted and as the infant begins to breathe independently, the ventilator settings are rapidly reduced. The ventilator is switched to a pressure support/ continuous positive airway pressure mode. After 2 hours, the infant is noted to have a respiratory rate of 38 breaths/min, a tidal volume of approximately 40 mL, and to have a peripheral oxygen saturation of 95% on a fraction of inspired oxygen of 0.3. An audible air leak cannot be detected below an airway pressure of 20 cm of water. Of the following, the GREATEST risk for extubation failure in this patient is: A. failed airway leak test B. fraction of inspired oxygen of 0.3 C. history of upper airway obstruction D. rapid weaning of mechanical support

C. history of upper airway obstruction The need for respiratory support is one of the top few reasons for admission to a pediatric intensive care unit. Despite this fact and the large body of information on management strategies for patients with ongoing respiratory insufficiency, there is a marked lack of clear evidence guiding weaning of mechanical ventilatory support or accurate identification of patients who are ready to be extubated. It is important to avoid delays in weaning in order to reduce the risk of ventilator-associated pneumonia, ventilator-induced diaphragm dysfunction or lung injury, airway injury from the endotracheal tube, or dependence on sedation agents. On the other hand, precipitous weaning may result in extubation failure that has been reported to affect as many as 15% of children undergoing extubation and 40% of neonates. Failure of extubation by itself has been identified as carrying a 5-fold increase in the risk of mortality in children. Methods for assessing extubation readiness include the Rapid Shallow Breathing Index (RSBI), the Compliance, Rate, Oxygen and Pressure (CROP) index, or specific extubation readiness tests where a patient's ability to maintain certain physiologic parameters on a specified degree of support for a period of time is interpreted as signaling that the patient is likely to do well extubated. None of these mechanisms has been able to demonstrate widespread reliability and paradoxically, up to 50% of unintended extubations result in a successful, sustained extubation. An airway leak test is conducted by inflating the airway to a prescribed pressure, typically between 20 to 25 cm H2O. The ability to hear air "leaking" around the endotracheal tube by the unaided ear is assumed to be a favorable result. Many current-generation mechanical ventilators will calculate an inspiratory and an expiratory tidal volume, with the remainder supposed to have "leaked" around the endotracheal tube. This does not account for other potential sources of leak in the system. Previous investigators have demonstrated that the leak test is only reproducible in settings with the head in a neutral position and under neuromuscular blockade. Indeed, in children with upper airway anomalies or redundant tissue, an airway leak may be highly dependent on the position of the head and the endotracheal tube. Furthermore, individuals with copious secretions typically seen in viral respiratory infections may accumulate a seal of inspissated mucus around the endotracheal tube that can be expectorated after extubation. Consequently, while the presence of an audible air leak at modest airway pressures can be seen as reassuring, the absence of an air leak should not be taken as a contraindication to extubation readiness. Several different techniques for weaning mechanical ventilatory support have been described, some with gradual reduction in settings until the patient appears ready to extubate and others with modest reduction in settings but with daily extubation readiness tests (ERT). Although data strongly supporting daily ERTs exist in adults, data comparing weaning techniques in children are lacking. Many investigators have reported upper airway obstruction as a major cause of extubation failure, with some series attributing 40% of extubation failures to upper airway obstruction. Measuring the true incidence would require more invasive monitoring techniques capable of identifying a negative intrathoracic pressure (eg, esophageal manometry) and the absence of gas flow. A 2003 multicenter review of 1,459 intubated patients demonstrating a 6.2% extubation failure rate identified age less than 24 months, chronic respiratory or neurologic disease, the need to replace an endotracheal tube at the time of first intubation, and genetic or syndromic anomalies as being correlated with upper airway obstruction-related extubation failure. Objective data regarding the effectiveness of therapies such as racemic epinephrine, continuous positive airway pressure and helium-containing gas mixtures are lacking. An additional therapy commonly employed to limit postextubation stridor, upper airway obstruction, and extubation failure is the use of corticosteroids. Methylprednisolone and dexamethasone have been used in infants, children, and adults because of their theoretical effect on airway edema and the role edema may play in airway obstruction. A 2008 Cochrane Review of steroids for the prevention of postextubation stridor in infants, children, and adults failed to demonstrate effectiveness, though there was a trend towards benefit. PREP Pearls Less than 10% of patients who undergo scheduled extubation fail to successfully separate from mechanical ventilation. The need for reintubation carries significant potential morbidity and mortality. The principle cause of extubation failure in children is upper airway obstruction. A number of techniques have been demonstrated to predict successful extubation or limit the potential for failure after extubation. Objective data demonstrating specific benefit of any of these techniques and treatments is lacking. ABP Content Specifications(s)/Content Area Understand the principles of mechanical ventilation as a means to stabilize the patient after major or prolonged surgery Plan the postoperative approach to mechanical ventilation of a patient with abnormal preoperative lung function, upper airway abnormalities, or thoracic dystrophy Suggested Readings Newth CJ, Venkataraman S, Wilson DF, et al; Eunice Shriver Kennedy National Institute of Child Health and Human Development Collaborative Pediatric Critical Care Research Network. Weaning and extubation readiness in pediatric patients. Pediatr Crit Care Med. 2009;10(1):1-11. doi: 10.1097/PCC.0b013e318193724d

A 10-month-old infant was brought into the emergency department by his mother following a brief resolved unexplained event. The child was napping when the mother witnessed a brief episode of apnea, labored respirations, and limpness without color change. He was diagnosed with a viral respiratory infection several days ago and has had cough, congestion, and low-grade fever. Mom has been treating symptoms with pediatric acetaminophen, an antihistamine, and an adult cough syrup elixir. In the emergency department the infant is alert and appears well, but falls asleep easily when not stimulated. Rectal temperature is 36.8℃, heart rate is 132 beats/min, respiratory rate is 18 breaths/min, blood pressure is 80/50 mm Hg, and oxygen saturation by pulse oximetry is 100% on room air. The patient has slightly decreased muscle tone, pupils are equal and reactive, and motor reflexes are appropriate. The rest of the child's physical findings are unremarkable. Chest radiograph and brain CT are normal. Laboratory findings are as follows: Laboratory Test Result White blood cell count11,800/µL (11.8 x 109/L) Hemoglobin12.1 g/dL (121 g/L) Platelet count171 x 103/µL (171 x 109/L) Sodium (Na)140 mEq/L (140 mmol/L) Potassium4.6 mEq/L (4.6 mmol/L) Chloride105 mEq/L (105 mmol/L) Bicarbonate16 mEq/L (16 mmol/L) Blood urea nitrogen (BUN)8 mg/dL (2.86 mmol/L) Creatinine0.19 mg/dL (16.8 µmol/L) Glucose64 mg/dL (3.55 mmol/L) Anion gap19 mEq/L (19 mmol/L) Serum osmolality307 mOsm/kg (307 mmol/kg) Based on the clinical scenario and laboratory results, the infant's symptoms are most likely due to: A. acetaminophen toxicity B. antihistamine overdose C. ethanol ingestion D. viral meningitis

C. ethanol ingestion The infant described in the vignette has symptoms consistent with ethanol ingestion, as evidenced by respiratory and central nervous system (CNS) depression, moderate hypoglycemia, metabolic acidosis with increased anion gap, and high measured serum osmolality. While medications approved for infants do not contain ethanol, many adult medications do contain alcohol formulations and should be avoided in children. In this case, the adult cough syrup elixir is the likely culprit. Adult over-the-counter cough syrups can contain up to 25% ethanol and even small doses can cause toxic effects in infants. Patterns of ethanol poisoning in pediatric patients vary by age. While intoxication may be deliberate in teenagers and young adults, ingestion in infants and children is usually unintentional. Ethanol poisoning in infants often results from caregivers administering oral medications that contain ethanol, or can occur from overzealous use of topical products containing alcohol formulations. In young children, intoxication frequently stems from the child drinking adult beverages left within reach, or from ingesting household products that contain ethanol. Common household products with significant concentrations of alcohol include some mouthwashes, perfumes and colognes, aftershave lotions, and many cleaning solutions. Adult medications with alcohol formulations present a risk for young children as well. Parental education regarding proper storage of all dangerous medications and household products should be included as part of anticipatory guidance during pediatric office visits. Symptoms of ethanol ingestion in infants and children are similar to those seen in older patients. Acute intoxication at any age commonly produces CNS and respiratory depression, and symptoms generally manifest when serum ethanol levels exceed 50-100 mg/dL (0.05-0.1 g/dL). High doses may cause metabolic and electrolyte derangements, severe myocardial depression, hypotension, loss of muscle control, hypothermia, loss of airway reflexes, cardiovascular collapse, and sudden death. In addition, infants and children are far more likely to develop severe, symptomatic hypoglycemia compared to older patients. Due to their poor glycogen stores and the ability of ethanol to inhibit gluconeogenesis, infants are particularly prone to develop hypoglycemia following the ingestion of relatively small amounts of ethanol. Electrolyte disturbances may not be apparent initially in infants and children following acute ingestion, making the diagnosis of ethanol poisoning difficult. Health care providers should maintain a high index of suspicion in infants and children at risk for ingestion, particularly those presenting with altered mental status, respiratory depression, or hypothermia of uncertain etiology. Appropriate diagnostic testing will depend on the situation, but may include a toxicology work-up with electrolyte analysis, urine drug screening, and serum levels of testable substances, including ethanol. Hypoglycemia is a common finding following ingestion and may be severe. Other key findings may be subtle but often include increased anion gap metabolic acidosis; calculation of the anion gap (sodium- chloride- bicarbonate) should be performed in all patients with suspected ingestion. CAT MUDPILES is a mnemonic traditionally used to remember the differential diagnosis in patients with increased anion gap metabolic acidosis (see below). Patients with ethanol toxicity generally have a mild to moderately increased anion gap. The presence of severe metabolic acidosis with a large anion gap may suggest the existence of a comorbid condition or ingestion of another substance. Causes of Increased Anion Gap Metabolic Acidosis C Carbon monoxide, CyanideA Aminoglycosides, AcetaminophenT Theophylline, Toluene (glue sniffing) M MethanolU UremiaD Diabetic ketoacidosisP ParaldehydeI Iron, Inborn errors of metabolismL Lactic acidosisE Ethanol, Ethylene glycolS Salicylates In addition to an increased anion gap, patients with ethanol poisoning have an increased serum osmolal gap. The osmolal gap is the difference between the measured and expected osmolality; this difference should be less than 10 mOsm/kg (10 mmol/kg) under normal conditions. An elevated osmolal gap suggests the presence of an unmeasured substance contributing solutes to the serum. Such substances can include alcohols (eg, ethanol, methanol, ethylene glycol, isopropanol), acetone, sugars other than glucose, lipids, and certain proteins. In patients with suspected ingestion of any alcohol it is imperative to check a serum osmolality as well as calculate the expected osmolality. The osmolal gap can be calculated by the following equation: Osmolal gap = Measured osmolality - Expected osmolality, where expected osmolality is (2 X [Na concentration]) + (glucose concentration/18) + (BUN concentration/2.8). For the child in the vignette, the measured osmolality is 307 mOsm/kg (307 mmol/kg). The expected osmolality is (2 x 140) + (64/18) + (8/2.8) = 286 mOsm/kg (286 mmol/kg). Therefore, the patient's osmolal gap is 21, well above the normal value. Treatment in infants and children with ethanol ingestion is primarily supportive. Hypoglycemia and respiratory distress are the most immediate life-threatening conditions and should be addressed immediately. Endotracheal intubation and mechanical ventilation may be required in patients with inadequate oxygenation or ventilation, or in those with diminished airway reflexes. Significant or symptomatic hypoglycemia should be corrected. Cardiovascular support may include fluid resuscitation and ongoing infusion of fluid with dextrose. Electrolyte abnormalities are uncommon but should be corrected as needed. Metabolic acidosis and hyperosmolality will correct over time with isotonic fluid infusion, and as ethanol is metabolized and eliminated. Serial ethanol levels can be monitored but do not typically affect patient management once the diagnosis is established. While the patient in the vignette has been exposed to both acetaminophen and an antihistamine based on history, ethanol is the most likely cause of this patient's clinical presentation. Acetaminophen toxicity can cause hypoglycemia as well as an increased anion gap metabolic acidosis, but would not be expected to cause an increased osmolal gap. Antihistamine ingestion is not associated with either increased anion gap, metabolic acidosis or hyperosmolality. Viral meningitis is unlikely with lack of fever, normal white blood count, and the other laboratory findings seen in this infant. PREP Pearls While relatively uncommon in infants, ethanol intoxication poses an important risk and potential cause of apparent life-threatening event in this population. Health care providers should maintain a high index of suspicion in susceptible patients. Differences in metabolism and decreased glycogen stores predispose infants and children to severe symptoms and marked hypoglycemia with even small doses of ethanol compared to older patients. Key features of ethanol ingestion in infants and children include altered mental status, hypothermia, hypoglycemia, metabolic acidosis with increased anion gap, and increased osmolal gap. ABP Content Specifications(s)/Content Area Understand differential effects of ethanol ingestion in infants Suggested Readings Rayar P, Ratnapalan S. Pediatric ingestions of household products containing ethanol: a review. Clin Pediatr. 2013;52(3):203-209. doi:10.1177/0009922812470970 Toce MS, Burns MM. The poisoned pediatric patient. Pediatr Rev. 2017;38(5):207-220. doi: 10.1542/pir.2016-0130 Wolf AD. Principles of toxin assessment and screening. In: Fuhrman BP, Zimmerman JJ, eds. Fuhrman and Zimmerman's Pediatric Critical Care. 4th ed. Philadelphia, PA: Saunders Elsevier; 2011:1441-50. Comment on Question

A 7-month-old male infant, who was recently given a diagnosis of spinal muscular atrophy type 1, was admitted with acute gastrointestinal enteritis and dehydration. Given his acute illness, he is more hypotonic than at baseline. His chest radiograph reveals clear but low-volume lung fields. His usual pulmonary regimen is instituted with mechanical insufflator-exsufflator "cough assist." While volume resuscitation is ongoing, his pulse oximetry oxygen saturation is noted to have decreased from 99% to 92%. Recognizing that this is likely an indication of hypercarbia in a child with spinal muscular atrophy, a capillary blood gas analysis is performed, which confirms the suspicion. The use of biphasic cuirass ventilation is discussed with the parents. Of the following, the MOST accurate statement regarding the mode of ventilation proposed is A. alveolar expansion is determined by the transpulmonary gradient B. exhaled volume is inversely proportional to elastic recoil C. infection risk is increased because of secretion aspiration D. venous return is decreased

A. alveolar expansion is determined by the transpulmonary gradient Numerous methods for noninvasive support are available, including bilevel positive airway pressure (BiPAP or BPAP), continuous positive airway pressure (CPAP), high-flow nasal cannula, continuous negative extrathoracic pressure, negative pressure ventilation (NPV; "iron lung," "shell," "cuirass"), and biphasic cuirass ventilation. Each has its uses and limitations. Negative pressure ventilation typically uses a constant extrathoracic pressure of −20 to −40 cm H2O. Biphasic cuirass ventilation, in contrast to NPV, has an active (positive pressure) expiratory phase. First used in the United States during the 1930s polio epidemic, iron lungs encased the patient's body, leaving only the head and neck exposed. Patients typically were supported for days to a few weeks but not for prolonged periods, though there are reports of patients surviving more than 50 years in an iron lung. In today's environment, NPV is now provided via a cuirass (also called "turtle shell"). The device is placed around the child's thorax with the hard shell being anterior and sealed via soft, nonporous, pliable material around the edges (Figure). During inspiration, application of extrathoracic subatmospheric (negative) pressure decreases pleural pressure (Ppl); negative pressure applied to the body surface (Pbs) is transmitted to the pleural space and then to the alveoli (Palv), as shown in Figure 43-3 in Egan's Fundamentals of Respiratory Care. Given that the patient's airway is at atmospheric pressure and there is a continuous connection from the airway to alveoli (tracheobronchial tree), a transairway pressure gradient is created. Gas flows from the relatively higher pressure at the airway opening (mouth) to the lower pressure in the alveoli. As with spontaneous breathing, alveolar expansion during NPV is determined by the magnitude of the transpulmonary pressure gradient. Expiration passively occurs due to elastic recoil of the lungs and chest wall. As this recoil occurs, Palv becomes less negative, increasing above atmospheric pressure and thus reversing the transairway pressure gradient. As a result, gas flows from the lungs to the airway opening (exhalation). In biphasic cuirass ventilation, as in the vignette, the expiratory phase is augmented by application of positive pressure to the chest wall, which is transmitted to the alveolus. Because the ventilation provided by the cuirass is biphasic, it is possible to achieve high tidal volumes and a high respiratory rate. In human volunteers and animal models, NPV results in improvement in functional residual capacity, increased venous return, decreased right ventricular afterload, increased right ventricular output, decreased pulmonary vascular resistance, and increased cardiac output. Today its use is primarily for patients with neuromuscular disease, either for nocturnal or continuous use. Continuous negative pressure has been used in conditions with increased work of breathing, small airway disease, ventilation-perfusion (V/Q) mismatch, central hypoventilation syndrome, cystic fibrosis, and phrenic nerve injury, as well as after cardiac surgery. It is not a mode to use with upper airway obstruction because the inflow of gas would be impaired. A 2013 Cochrane review of continuous negative extrathoracic pressure and CPAP compared with conventional ventilation for acute hypoxemic respiratory failure in children failed to find sufficient evidence for adequate comparison. In cardiac repairs dependent on passive diastolic flow, such as Glenn and Fontan procedures, NPV has been demonstrated to increase pulmonary blood flow and thereby cardiac output. Its use has been reported as a means to support "failing Fontan" physiology. Benefits of NPV include no risk of ventilator-associated pneumonia, uniform lung expansion, and no risk of gastric distention. Patients with adequate oropharyngeal motor skills may talk, eat, and drink as they normally would. If the child is gastrostomy dependent, care must be taken to maintain a seal around the gastrostomy tubing as it exits the cuirass and to change the exit site frequently to avoid pressure injury and skin breakdown. Disadvantages of NPV include skin pressure injury and lack of easy mobility. PREP Pearls Negative pressure ventilation is primarily used for patients with neuromuscular disease, either for nocturnal or continuous use, and in cardiac repairs dependent on passive diastolic flow, such as the Glenn and Fontan procedures. Benefits of negative pressure ventilation include no risk of ventilator-associated pneumonia, uniform lung expansion, and no risk of gastric distention. Disadvantages include skin pressure injury and lack of easy mobility. ABP Content Specifications(s)/Content Area Know the principles underlying the use of negative-pressure mechanical ventilation Know the advantages and disadvantages of negative-pressure mechanical ventilation Suggested Readings Deep A, De Munter C, Desai A. Negative pressure ventilation in pediatric critical care setting. Indian J Pediatr. 2007;74(5):483-488. Deshpande SR, Mahre KO. Long term negative pressure ventilation: rescue for the failing fontan? World J Cardiol. 2014;6(8):861-864. doi:10.4330/wjc.v6.i8.861 Kacmarerk RM, Volsko TA. Physiology of ventilatory support. In: Kacmarerk RM, Stoller JK, Heuer A, eds. Egan's Fundamentals of Respiratory Care [ebook]. St Louis, MO: Elsevier Mosby; 2013:chap 43. https://clinicalgate.com/physiology-of-ventilatory-support Linton DM. Cuirass ventilation: a review and update. Crit Care Resusc. 2005;7(1):22-28. Shah PS, Ohlsson A, Shah JP. Continuous negative extrathoracic pressure or continuous positive airway pressure for acute hypoxemic respiratory failure in children. Cochrane Database Syst Rev. 2008;(1):CD003699. doi: 10.1002/14651858.CD003699.pub3

You added neuromuscular blockade to other therapies in an attempt to reduce the intracranial pressure in a 5-year-old with closed head injury and refractory intracranial hypertension. The bedside nurse starts a cisatracurium infusion at 0.1 mg/kg/h intravenously as prescribed but forgets to give the 0.1 mg/kg loading dose that you had also ordered. The nurse places electrodes on the patient's forearm along the course of the ulnar nerve and uses a peripheral nerve stimulator to assess a train-of-four response. The nurse reports that 15 minutes after starting the infusion, there are 4 equal muscle twitches elicited using a 20-mA current. This is exactly the same as the patient's baseline before the infusion was begun. Of the following, the most likely explanation for this train-of-four response is because the approximate percentage of the receptors occupied by the medication is A. 70% B. 80% C. 90% D. 100%

A. 70% Nondepolarizing muscle relaxants (eg, vecuronium, rocuronium, cisatracurium) act by binding to acetylcholine receptors in the neuromuscular junction and competitively blocking its action as a neurotransmitter. The degree of muscle relaxation seen depends on how many receptors are blocked. Muscle relaxation can be clinically monitored by using a peripheral nerve stimulator to deliver a succession of 4 electrical stimuli (train of four) and observing for muscle twitches. There is no change in the train-of-four response until 75% of the receptors are blocked. Once 75% of the receptors are blocked, stimulation of the target nerve produces only 3 twitches, not 4. As a general guide, Four equal twitches are seen if 0% to 75% of the receptors are blocked. Three twitches are seen if at least 75% of the receptors are blocked. Two twitches are seen if 80% of the receptors are blocked. One twitch is seen if 90% of the receptors are blocked. No twitches are seen if 100% of receptors become blocked. The ulnar nerve is commonly used to assess the train-of-four response in clinical practice. Two electrodes are placed on the extended forearm along the path of the ulnar nerve. The distal electrode is placed near the wrist flexor crease, and the second one, a few centimeters proximal to the first, parallel to the flexor carpi ulnaris tendon. The negative (black) lead wire is attached to the distal electrode, and the positive (red) lead wire is attached to the proximal electrode. On application of electrical stimulation (usually 10 mA to 50 mA, incrementally adjusted), the fingers are expected to twitch. It is important to note that clinical neuromuscular relaxation does not require 100% of the receptors to be blocked. Leaving 100% of the receptors blocked can lead to prolonged and severe muscle weakness during recovery in the intensive care unit, in some cases requiring long-term mechanical ventilation. Therefore, patients receiving nondepolarizing muscle relaxants should be monitored using a peripheral nerve stimulator, and their infusion rate should be adjusted to achieve 2 to 3 twitches on a train of four. The goal should be to use the lowest dose possible to limit adverse effects of prolonged neuromuscular blockade. This ensures adequate clinical muscle relaxation while minimizing the risks of prolonged neuromuscular blockade. The choice of agent may depend on user preference as well as the presence of organ dysfunction, which may affect elimination of the drug. In a randomized controlled trial, recovery of neuromuscular function after discontinuation of the neuromuscular blocking drug infusion in children was significantly faster with cisatracurium than vecuronium. PREP Pearls Clinical muscle relaxation can be achieved with blocking 75% to 80% of receptors with a nondepolarizing muscle relaxant. The goal should be to use the lowest dose of nondepolarizing muscle relaxant possible to limit adverse effects of prolonged neuromuscular blockade. ABP Content Specifications(s)/Content Area Understand that nondepolarizing neuromuscular blockade cannot be reversed until there is a twitch present on neuromuscular junction monitoring Know that the height of a single twitch is not altered until 75% of receptors are occupied during neuromuscular junction monitoring Suggested Readings Burmester M, Mok Q. Randomised controlled trial comparing cisatracurium and vecuronium infusions in a paediatric intensive care unit. Intensive Care Med. 2005;31(5):686-692. doi:10.1007/s00134-005-2615-3 Johnson PN, Miller J, Gormley AK. Continuous-infusion neuromuscular blocking agents in critically ill neonates and children. Pharmacotherapy. 2011;31(6):609-620. doi:10.1592/phco.31.6.609 Kudchadkar SR, Easley RB, Brady KM, et al. Pain and sedation management. In: Shaffner DH, Nichols DG, eds. Rogers' Textbook of Pediatric Intensive Care. 5th ed. Alphen aan den Rijn, Netherlands: Wolters Kluwer; 2015. Watling SM, Dasta JF. Prolonged paralysis in intensive care unit patients after the use of neuromuscular blocking agents: a review of the literature. Crit Care Med. 1994;22(5):884-893.

You admit a previously healthy 4-year-old boy with altered mental status. He and his parents had been staying in a friend's cabin for several days. He had had some mild complaints of headache and dizziness, but had been otherwise well. When they went to wake him in the morning, he was slower to arouse and appeared confused. He felt warm, but this was possibly due to the portable heater they had placed in the room the first night. Upon admission, the child is afebrile, awake, but does appear somewhat confused. His respiratory rate and heart rate are mildly elevated and his pulse oximetry is 96% on room air. The remainder of his examination is normal. You review the laboratory studies obtained in the emergency department. His chemistry panel and complete blood counts are normal, and his arterial blood gas had a pH 7.30, pCO2 34 mm Hg, pO2 60 mm Hg, and oxygen saturation 85%. Of the following, the test MOST likely to reveal the diagnosis is: A. computed tomography of head B. co-oximetry of arterial blood C. lumbar puncture D. respiratory viral studies

B. co-oximetry of arterial blood Based on the clinical presentation, the child in the vignette is likely to have carbon monoxide (CO) poisoning. Carbon monoxide is a colorless, odorless, and tasteless gas that forms during incomplete combustion of carbon-containing material. The most common sources of CO exposure are fires (particularly within enclosed spaces), engine exhaust, and heaters or generators used indoors. The diagnosis of carbon monoxide poisoning is confirmed with elevated levels of carboxyhemoglobin (COHb) levels. Carboxyhemoglobin can be measured from a blood sample using spectrophotometric co-oximetry, which is capable of separating out levels of oxyhemoglobin, COHb, and methemoglobin. The amount of COHb generated will depend upon the concentration of CO in the air, duration of exposure, and the level of alveolar ventilation. Thus, exposures that occur in closed spaces are much more likely to be associated with elevated COHb levels. Generally, levels above 2% to 3% would be consistent with CO poisoning, although tobacco smokers can have elevated levels chronically so a higher cut-off (eg, more than 10%) may be used for diagnosis. It is also important to note that patients can also be symptomatic from chronic exposure at lower COHb levels, and thus, the so-called "cut-off" should be placed in context of the clinical suspicion. The adverse effects of CO poisoning are multifold (Figure). Carbon monoxide can reduce oxygen delivery resulting in tissue hypoxia via 2 mechanisms. Carbon monoxide binds to the heme-component of hemoglobin with an affinity ~ 250 times greater than that of oxygen. As CO competes with oxygen for 1 of the 4 binding sites, its presence reduces the overall oxygen-carrying capacity of hemoglobin. Additionally, bound CO stabilizes the hemoglobin in its high-affinity state (R-state), shifting the oxyhemoglobin curve to the left. Therefore, hemoglobin is less likely to release any bound oxygen at the tissue level, further impairing oxygen delivery. Carbon monoxide can also bind to other heme-containing products such as myoglobin and cytochrome c oxidase. When bound to the latter, CO shuts down oxidative phosphorylation, decreasing adenosine triphosphate production and increased production of reactive oxygen species. Because the affinity of cytochrome c oxidase for CO is only 3 times that of oxygen, these effects are greatest under hypoxic conditions. Unbound carbon monoxide can also have deleterious systemic effects including platelet activation, neutrophil degranulation with release of myeloperoxidase or proteases, and reactive oxygen species with increased oxidative stress, stimulation of a systemic inflammatory response, and induction of further CO production within tissues. The clinical effects of CO poisoning can be variable, depending upon the degree of exposure. However, because the symptoms are nonspecific, often described as "flu-like," the clinician must have a high index of suspicion to make a timely diagnosis, implement appropriate therapy, and reduce the likelihood of future exposures, in the case of chronic exposure. Headaches and dizziness are common complaints at lower levels of exposure. More severe poisoning can result in further alterations in mental status, including coma and seizures. Cardiovascular effects include tachycardia, other arrhythmias, hypotension and myocardial ischemia. Because of the impairment in oxygen delivery and utilization, patients with CO poisoning can have lactic acidosis. They can have tachypnea, dyspnea, pulmonary edema, or respiratory depression in severe cases. Routine pulse oximetry using dual wavelength spectrophotometry can be falsely elevated with CO poisoning. Carboxyhemoglobin and oxyhemoglobin absorb at the red wavelength (660 nm) with a similar absorption, so dual wavelength pulse oximetry cannot distinguish between the two. Although there are pulse oximeters capable of measuring additional wavelengths, these are not as commonly used. Direct measurement of oxyhemoglobin using arterial blood gas sampling will demonstrate a "saturation gap" between the directly measured oxygen saturation and that obtained using dual-wavelength pulse oximetry. The presence of a gap should raise the suspicion of CO poisoning. The child in the vignette has several clinical signs of CO poisoning, including the saturation gap. The history of a portable heater in his room coinciding with the timeframe of symptoms further supports the clinical suspicion. The diagnosis should be made through measurement of COHb levels. PREP Pearls Carbon monoxide poisoning can have nonspecific symptoms, requiring a high index of suspicion. Carbon monoxide exerts its adverse effects primarily through binding to heme-containing products, including hemoglobin, resulting in impaired oxygen delivery and use and tissue hypoxia. The diagnosis of carbon monoxide poisoning is made from measuring carboxyhemoglobin levels using multi-wavelength co-oximetry. Elevated carboxyhemoglobin levels falsely elevate oxygen saturations measured by standard pulse oximetry, creating a "saturation gap" from directly measured oxygen saturation. ABP Content Specifications(s)/Content Area Recognize the clinical and laboratory manifestations of carbon monoxide poisoning Understand the pathogenesis and toxic effects of carbon monoxide Suggested Readings Guzman JA. Carbon monoxide poisoning. Crit Care Clin. 2012;28(4):537-548. doi: 10.1016/j.ccc.2012.07.007 Rose JJ, Wang L, Xu Q, et al. Carbon monoxide poisoning: pathogenesis, management and future directions of therapy. Am J Respir Crit Care Med. 2017;195(5):596-606. doi: 10.1164/rccm.201606-1275CI

While on service in the pediatric intensive care unit, you get called to the emergency department regarding a 2-year-old girl with suspected drug ingestion. The child was left unsupervised and was found unconscious near an open medicine cabinet by the mother, who immediately drove her to the emergency department. Upon arrival, the patient is lethargic, but becomes agitated upon stimulation. Her temperature is 38.8℃, heart rate 146 beats/min, respiratory rate 24 breaths/min, and blood pressure is 126/75 mm Hg. Her skin is flushed and dry, and pupils are equal, dilated, and sluggishly reactive. Complete blood count and serum electrolytes are unremarkable. Urine drug screen and acetaminophen levels are pending. Mother is not certain about all of the medications in the home, but she personally takes medications for hypertension and anxiety. She believes there are several over-the-counter cold and allergy remedies in the cabinet, as well as acetaminophen and ibuprofen. Of the following, this patient's symptoms are MOST likely due to: A. inhibition of ß-adrenergic receptors B. muscarinic receptor antagonism C. potentiation of gamma-aminobutyric acid (GABA) release D. presynaptic α2-adrenergic receptor stimulation

B. muscarinic receptor antagonism The child in the vignette has ingested an antihistamine and is suffering anticholinergic effects including lethargy, agitation, hyperthermia, tachycardia, hypertension, flushed skin, and mydriasis. The most likely culprit in this case is an over-the-counter cold or allergy medication containing a first-generation antihistamine such as diphenhydramine or brompheniramine. Poisonings from the ingestion of household medications occur in more than 1 million children per year, and 30% require admission to a health care facility. According to the American Association of Poison Control Centers, antihistamines account for 12% of such ingestions. In children less than 6 years of age, these exposures are usually unintentional; older children and especially adolescents may ingest such substances for recreational or suicidal purposes. Antihistamines comprise a group of drugs that oppose the activity of histamine receptors. Four histamine receptors have been described in humans; H1, H2, H3, and H4. Antihistamines are subclassified according to their targeted receptor. Currently, only medications that target H1 and H2 receptors are available for pharmacologic purposes. H1-receptor antagonists are used to treat cold and allergy symptoms, mast-cell related disorders, and insomnia. These include first-generation antihistamines (eg, diphenhydramine), which cross the blood brain barrier and cause CNS effects, and second-generation antihistamines (eg, loratadine), which minimally cross the blood brain barrier and thus are less sedating. H2-receptor antagonists (eg, cimetidine) primarily inhibit gastric secretions and are clinically used for gastrointestinal conditions including gastroesophageal reflux disease. While many antihistamines are available without a prescription, the first-generation antihistamines are the most frequent cause of overdose requiring hospital admission. This subclass is often used in over-the-counter cold and allergy preparations (Table), and thus is widely accessible in many households. First generation antihistamines also have poor receptor specificity and exert antagonistic effects on muscarinic receptors as well as H1-receptors. In fact, as histamine is not a major mediator in the pathogenesis of the common cold, the benefits of these drugs in relieving cold symptoms is secondary to their anticholinergic properties. The anticholinergic properties of first-generation antihistamines are also responsible for the majority of their toxic effects. Clinical features of this toxicity stem from competitive inhibition of acetylcholine at muscarinic receptors and include both peripheral and central manifestations. Altered mental status, hyperthermia, tachypnea, tachycardia, hypertension, dry skin and mucous membranes, mydriasis, ileus, and urinary retention are common. Agitation, psychosis, and seizures can occur. Arrhythmias are uncommon but prolonged QT and QRS widening can occur, particularly in patients with diphenhydramine exposure. Seizures and arrhythmias are thought to be related to sodium channel blockade. Prolonged seizure activity can result in rhabdomyolysis and severe metabolic acidosis in susceptible patients. The mnemonic "hot as a hare (fever), blind as a bat (mydriasis), dry as a bone (dry mucous membranes, decreased sweat), red as a beet (flushed skin), full as a flask (urinary and fecal retention), mad as a hatter (agitation, delirium), seizing like a squirrel (seizures, coma)" is sometimes used to describe the combination of peripheral and central symptoms in patients with anticholinergic syndrome. The diagnosis of antihistamine overdose relies on careful history and physical examination. No specific laboratory findings are diagnostic, and routine toxicologic screens do not detect these drugs. Diagnostic testing will depend on the situation, but basic toxicology work-up should be performed in any patient with suspected antihistamine ingestion to rule out the presence of other substances or comorbidities. Treatment for antihistamine toxicity is supportive. Patients with symptomatic overdose should undergo close cardiorespiratory monitoring until symptoms improve or resolve. Significant hyperthermia may require antipyretics or cooling measures. Benzodiazepines are indicated for agitation and seizures. Some patients with certain arrhythmias may improve with intravenous sodium bicarbonate administration. Cholinergic drugs such as physostigmine have been used in some cases for severe anticholinergic toxicity, particularly in the settings of tachydysrhythmias and significant CNS manifestations. However, this treatment is rarely required for anticholinergic symptoms caused by antihistamine overdose and should be used with caution. While the patient in the vignette likely had access to multiple medications, her symptoms are consistent with anticholinergic syndrome, and antihistamine toxicity is the most likely cause of this patient's presentation. Medications that inhibit ß-adrenergic receptors (ß-blockers), drugs that potentiate GABA release, such as benzodiazepines, and those that stimulate presynaptic α2-adrenergic receptors, such as clonidine, can all produce CNS effects but overdose would result in cardiorespiratory depression, not excitation, and would not be expected to cause other anticholinergic symptoms. PREP Pearls Toxicity in antihistamine overdose stems primarily from their ability to cause anticholinergic syndrome. First generation antihistamines, including diphenhydramine, are the most common antihistamines implicated in symptomatic overdose due to their widespread use and availability in over-the-counter medications, their CNS effects, and their propensity to cause anticholinergic symptoms. ABP Content Specifications(s)/Content Area Recognize the clinical and laboratory manifestations of antihistamine overdose Know the pathophysiology and toxic effects of antihistamine overdose Plan the diagnostic assessment and evaluation of suspected antihistamine overdose Plan appropriate therapy for a child with antihistamine overdose Suggested Readings Gosselin S, Hoegberg LG, Hoffman RS, et al. Evidenced-based recommendations on the use of intravenous lipid emulsion therapy in poisoning. Clin Toxicol. 2016;54(10):899-893. doi: http://dx.doi.org/10.1080/15563650.2016.1214275 Gummin DD, Mowry JB, Spyker DA, Brooks DE, Fraser MO, Banner W. 2013 Annual Report of the American Association of Poison Control Centers' National Poison Data System (NPDS): 34st annual report. Clin Toxicol. 2017;55(10):1072-1252. doi: 10.1080/15563650.2017.1388087 Lovegrove MC, Weidle NJ, Budnitz DS. Trends in emergency department visits for unsupervised pediatric medication exposures, 2004-2013. Pediatrics. 2015;136(4):e821-829. doi: 10.1542/peds.2015-2092 Lowry JA, Leeder JS. Over-the-counter medications: update on cough and cold preparations. Pediatr Rev. 2015;36(7):286-297. doi:10.1542/pir.36-7-286 Wolf AD. Principles of toxin assessment and screening. In: Fuhrman BP, Zimmerman JJ, eds. Fuhrman and Zimmerman's Pediatric Critical Care. 4th ed. Philadelphia, PA: Saunders Elsevier; 2011:1144-46.

A 2-year-old girl was admitted to the pediatric intensive care unit 7 days ago with respiratory failure from H3N2 influenza pneumonia, and has been mechanically ventilated since then. Her pneumonia has now improved on chest radiograph, and endotracheal secretions are minimal. An arterial blood gas was obtained and your calculations demonstrate an oxygenation index of 5. An extubation readiness test was performed by placing the patient on positive end-expiratory pressure of 5 cm of H2O. Her work of breathing increased significantly, though it improved after addition of pressure support at 5 cm H20 above positive end-expiratory pressure. Her FiO2 was maintained at her baseline setting of 0.5 during the extubation readiness test. Her oxygen saturation remained over 95% and exhaled tidal volumes were 7 mL/kg body weight. Of the following, which clinical parameters described above is most predictive of this patient's successful extubation? A. decrease in work of breathing on adding pressure support B. improved chest radiograph findings C. minimal endotracheal secretions D. stable O2 saturation and adequate tidal volume during pressure support

D. stable O2 saturation and adequate tidal volume during pressure support Identifying patients who are ready for extubation is important to reduce the morbidity and mortality associated with prolonged mechanical ventilation. Overall, extubation failure, and subsequent reintubation has been shown to occur in 14% to 24% of patients in various studies in children. Though several studies have attempted to identify predictors of extubation failure with varying success, the timing of extubation usually depends on clinical judgement by the clinical team. A study has recently shown the accuracy of an extubation readiness test in children receiving mechanical ventilation for lower respiratory tract disease. The Randomized Evaluation of Sedation Titration for Respiratory Failure (RESTORE) trial was a multicenter cluster randomized trial of sedation in children. A secondary analysis demonstrated the accuracy of extubation readiness test. In this study, patients with an oxygenation index (OI) or oxygen saturation index (OSI) ≤6 were eligible for extubation readiness test. The OI and OSI can be calculated as follows: Oxygenation index (OI) = mean airway pressure × FiO2 × 100 / PaO2Oxygen saturation index (OSI) = mean airway pressure × FiO2 × 100 / SpO2 In the extubation readiness test, the patients were placed sequentially on FiO2 of 0.50, positive end-expiratory pressure (PEEP) of 5 cm H2O, and pressure support applied above PEEP. If the patients were able to maintain oxygen saturations ≥95%, exhaled tidal volumes ≥5 mL/kg ideal body weight, and acceptable respiratory rate for age they were considered ready for extubation. In this population, the first extubation readiness test had a positive predictive value (PPV) of 92% if the patient was extubated within 10 hours of the extubation readiness test. The negative predictive value, sensitivity, and specificity were 3%, 90%, and 4%, respectively. The PPV falls to 80% if need for noninvasive ventilation such as bilevel positive airway pressure (BiPAP) is included in the definition of extubation failure. There can be a variety of reasons for extubation failure, including unresolved lung disease, increased resistive load due to upper airway obstruction, decreased respiratory drive (sedation, brain injury), neuromuscular weakness (malnutrition, severe electrolyte disturbances), and need for increased minute ventilation (such as sepsis, metabolic acidosis). These factors should be addressed by the clinicians before starting an extubation readiness test. In adults, respiratory therapist driven weaning protocols and spontaneous breathing trials have been successful as weaning strategies. When compared to physician directed weaning, respiratory therapist driven protocols do reduce the length of ventilation and weaning time in children, but spontaneous breathing trials has not proved to be as useful. For example, there is no difference in the proportion of patients who remain extubated at 48 hours whether pressure support of 10 cm H2O or a T piece are used for the spontaneous breathing trials. Decrease in work of breathing on the addition of pressure support probably reflects an increased level of support from the ventilator, and by itself is not predictive of readiness to extubate. Improvement in radiograph findings can lag behind the clinical picture, and while reassuring, may delay extubation unnecessarily. An improving chest radiograph also does not provide any insight regarding the other factors associated with extubation failure discussed above. The correlation between endotracheal secretions and successful extubation is not clear. PREP Pearls Oxygenation index or oxygen saturation index ≤6 signifies eligibility for extubation readiness test in patients with acute parenchymal lung disease. Patients who maintain oxygen saturations ≥95% and exhaled tidal volumes ≥5 mL/kg during extubation readiness test with FiO2 of 0.50, positive end-expiratory pressure of 5 cm H2O, and appropriate pressure support are highly likely to be extubated successfully (positive predictive value 92%). ABP Content Specifications(s)/Content Area Assess readiness for extubation in a patient recovering from acute lung disease Suggested Readings Faustino EV, Gedeit R, Schwarz AJ, et al. Accuracy of an extubation readiness test in Predicting successful extubation in children with acute respiratory failure from lower respiratory tract disease. Crit Care Med. 2017;45(1):94-102. doi: 10.1097/CCM.0000000000002024 Laham JL, Breheny PJ, Rush A. Do clinical parameters predict first planned extubation outcome in the pediatric intensive care unit? J Intensive Care Med. 2015;30(2):89-96. doi: 10.1177/0885066613494338 Schultz TR, Lin RJ, Watzman HM, et al. Weaning children from mechanical ventilation: a prospective randomized trial of protocol-directed versus physician-directed weaning. Respir Care. 2001;46(8):772-782.

A 17 year-old male is admitted to the pediatric intensive care unit after a motor vehicle collision in which he sustained multiple rib fractures, grade 3 liver laceration, and right femur fracture. Your primary survey reveals an alert but anxious patient with adequate airway, slight increased work of breathing, tachycardia, and pallor. Rectal temperature is 37.1°C, oxygen saturation is 96% on room air, blood pressure is 119/76 mm Hg, and capillary refill time is 3 to 4 seconds. You suspect compensated shock due to acute blood loss and initiate volume resuscitation with isotonic fluids as well as transfusion of 2 units of packed red blood cells. Blood typing and cross-matching reveal no antibodies or incompatibility with donor blood. During the transfusion the patient experiences fever and chills but has no other new signs or symptoms. Given the patient's clinical scenario, the MOST likely transfusion-related cause for fever involves which of the following? A. antibody-mediated complement fixation leading to red blood cell lysis B. attack of recipient cells by viable T lymphocytes in donor blood C. IgE-antibodies directed against antigens in donor plasma D. pyrogenic cytokines accumulated in blood product during storage

D. pyrogenic cytokines accumulated in blood product during storage The most likely transfusion-related cause for fever in this patient is a febrile nonhemolytic transfusion reaction (FNHTR). This reaction occurs during or immediately after a transfusion, requires at least a 1°C rise in temperature into the febrile range, and can be accompanied by chills but is not typically associated with other signs or symptoms. The pathogenesis of FNHTR is thought to be mediated by pyrogenic cytokines accumulated in blood product during storage, and by antibodies in the recipient's plasma reacting with leukocytes in the transfused product. The practice of prestorage leukoreduction of cellular blood components has greatly decreased the incidence of FNHTRs; however, this reaction remains the most reported transfusion reaction in adults and children. A diagnosis of exclusion, FNHTRs are generally benign and can be effectively treated with antipyretics in most cases. Blood product transfusion can be lifesaving for some patients, and up to 50% of all children admitted to the pediatric intensive care unit receive 1 or more transfusions during their hospital stay. Although advances in donor screening and blood component management have markedly decreased the risks, the potential hazards of transfusion-related adverse events remain a major concern. A prospective study from 2015 observed that transfusions in critically ill children were associated with increased morbidities including prolonged mechanical ventilation and hospital length of stay. Furthermore, randomized controlled trials such as the Transfusion Requirements in the Pediatric Intensive Care Unit (TRIPICU) study suggest that a conservative transfusion strategy should be implemented in critically ill children whenever possible. The potential hazards of transfusions can be divided into infectious and noninfectious categories. Although the risk of transfusion-transmitted infection is rare, such transmissions continue to occur. Infectious agents transmitted by blood products include viruses such as hepatitis A, B, C, and E; cytomegalovirus (CMV); human T-cell lymphotropic virus 1 and 2; Epstein-Barr virus; dengue; parvovirus; West Nile virus; and HIV, as well as certain parasites and variant Creutzfeldt-Jakob disease. Bacterial contamination is rare because of screening of all blood products but poses a life-threatening risk if it occurs. Advances in donor blood testing have led to increased sensitivity for many infectious agents and thus a marked reduction in donor transmission rates. The following is a list of infectious agents universally screened for before donor blood release: Hepatitis B virus HIV Hepatitis C virus Human T-cell lymphotropic virus Syphilis West Nile virus Chagas disease Zika virus Despite advances in screening, cases of transfusion-related infections are still reported in the United States each year. Testing failures, donation during the window between infection and detection, and clerical errors resulting in inappropriate quarantine assignment may all cause infectious units to be released for use. Furthermore, emerging infections continue to be a threat to blood product supply. Noninfectious risks of transfusions include a broad spectrum of adverse events ranging from true transfusion reactions to problems resulting from transfusion-related metabolic complications. True reactions are well described and can be divided into acute and delayed reactions, as well as categorized on the basis of presence or absence of fever. Acute reactions occur during transfusion or within 24 hours after transfusion. They include acute hemolytic, febrile nonhemolytic, transfusion-related acute lung injury, transfusion-associated circulatory overload, and allergic or anaphylactic reactions. Delayed reactions occur 24 hours to 6 weeks after transfusion and include delayed hemolytic, transfusion-associated graft-vs-host disease (GVHD), and posttransfusion purpura. The Table details all acute and delayed reactions with and without fever, and describes the incidence, timing, pathogenesis, signs and symptoms, and laboratory findings for each. After donation, blood products are anticoagulated for storage with citrate, which chelates calcium ions. Citrate toxicity and hypocalcemia can occur after transfusion of any product, but particularly with plasma and whole blood, which have the highest citrate levels. Other metabolic complications of transfusions include hypothermia and hyperkalemia. Patient size, clinical condition, and the amount and type of product all contribute to the frequency of these complications, and appropriate monitoring for such metabolic derangements is imperative. Noninfectious risks can be reduced by recipient pretransfusion testing, as well as blood product modification when indicated. Pretransfusion testing in children generally consists of ABO and Rh typing followed by screening for other red blood cell alloantibodies. These profiles are cross-matched with donor blood to avoid incompatibility. The use of modified products are indicated in certain clinical situations. Modifications include leukoreduction, irradiation, and washing. Leukoreduction involves filtering and reducing white blood cells from blood product to less than minimum requirements determined by the Food and Drug Administration. The near-universal practice of leukoreduction in the United States has significantly decreased the incidence of FNHTRs, as well as decreased the risk of CMV transmission. Irradiation of cellular blood products prevents lymphocytes from proliferating and renders them incapable of engraftment. Thus, this modification is currently indicated only for the prevention of transfusion-associated GVHD. The washing of cellular blood components removes extracellular fluid, most of the plasma proteins, anticoagulant, and storage media, and can be useful in patients with recurrent anaphylactic reactions to plasma-containing blood products or those susceptible to complications from hyperkalemia. The trauma patient in the vignette has many potential causes for fever, but the most likely transfusion-related cause is FNHTR, and pyrogenic cytokines accumulated in blood product during storage correctly describes the likely pathogenesis (Table). Antibody-mediated complement fixation leading to red blood cell lysis describes the pathogenesis for an acute hemolytic reaction. Although fever and chills may be present, this reaction would be accompanied by more severe symptoms and signs of hemolysis and increased bleeding. Attack of recipient cells by viable T lymphocytes in donor blood describes the mechanism associated with transfusion-associated GVHD. Fever may be prevalent, but this reaction is not acute, is associated with many other symptoms, and occurs only in certain susceptible patients. IgE-antibodies directed against antigens in donor plasma describe the pathogenesis of an allergic response, which would likely be nonfebrile and associated with other symptoms such as rash. Regardless of the cause or type, all suspected transfusion-related adverse events should be reported to the donor blood bank according to institution policy. Treatment depends on type and severity of the reaction, but usually includes immediate termination of the transfusion except in certain circumstances. Further workup and treatment depends on the clinical situation. All institutions that care for critically ill children and routinely employ transfusion therapy should institute standardized procedures to minimize transfusion requirements; recognize, report, and treat adverse events when they occur; and implement methods to ensure rigorous attention to detail to reduce clerical error in blood product management and delivery. PREP Pearls Advances in donor screening, pretransfusion testing, and blood product management have decreased transfusion risks, but transfusion-related adverse events and potential for increased morbidity remain a major concern in critically ill children. Specific indications for transfusion in children require further and ongoing investigations, but recent clinical trials recommend conservative restrictive transfusion strategies when possible. Efforts to minimize transfusion requirements; to recognize, report, and treat transfusion-related adverse events when they occur; and to implement methods to reduce clerical error in blood product management and delivery can further reduce potential hazards of transfusions in children. ABP Content Specifications(s)/Content Area Know what screening is routinely performed on donor blood prior to release from the blood bank Understand the potential hazards of blood component transfusions Suggested Readings Standards for Blood Banks and Transfusion Services. 29th ed. Bethesda, MD: AABB; 2014:31-32. American Society of Anesthesiologists Task Force on Perioperative Blood Management. Practice guidelines for perioperative blood management: an updated report by the American Society of Anesthesiologists Task Force on Perioperative Blood Management. Anesthesiology. 2015;122(2):241-275. doi:10.1097/ALN.0000000000000463 Lacroix J, Tucci M, Tinmouth A, Gauvin F, Karam O. Transfusion medicine. In: Fuhrman BP, Zimmerman JJ, eds. Pediatric Critical Care. 4th ed. Philadelphia, PA: Elsevier; 2011:1162-1176. Oakley FD, Woods M, Arnold S, Young PP. Transfusion reactions in pediatric compared with adult patients: a look at rate, reaction type, and associated products. Transfusion. 2015;55(3):563-570. doi:10.1111/trf.12827

A 16-year-old adolescent boy is admitted to the pediatric intensive care unit for severe epistaxis. The patient reports bleeding started the day prior to admission. He also reports hematemesis. Past medical history reveals no previous reports of bleeding. The patient is taking no medications. There is a history of marijuana use including synthetic cannabinoid, but he reports no other drug use. On examination, vital signs reveal a manual blood pressure of 114/72 mm Hg with no orthostatic changes, heart rate is 82 beats/min, and respirations are 16 breaths/min and nonlabored. The patient is afebrile. Room air saturations are 98%. The patient is awake and alert, able to answer questions, and has a normal neurologic examination. You note gingival bleeding on the oral examination. Lungs are clear to auscultation. Cardiovascular examination reveals normal heart sounds with good pulses and perfusion. Abdominal examination is unremarkable. A chest radiograph reveals no pulmonary pathology. Laboratory results include: Complete blood count White blood cell 10.6 x 103/uL Hgb 9 gram/dL Hct 28% Platelet 130,000 Electrolytes Na+ 137 mEq/L K+ 3.6 mEq/L Cl- 105 mEq/L CO2 21 mEq/L BUN 5 mg/dL Cr 0.72 mg/dL Glucose 94 mg/dL Liver function studies AST 15 units/L ALT 11 unit/L Albumin 3.8 g/L Total bilirubin 0.3 mg/dL Lactate1.8 mmol/L Urinalysis Slightly blood tinged Positive for red blood cells Specific Gravity 1.012 Protein: negative Ketones: negative Glucose: negative Urine drug screen Negative ETOH< 10 mg/dL Salicylates Undetectable Acetaminophen Undetectable Prothrombin time 70 s INR 4.2 Partial thromboplastin time 100s Fibrinogen 450 mg/dL Of the following, the MOST appropriate treatment is: A. factor IX concentrate B. packed red blood cells C. recombinant factor VIIa D. vitamin K

D. vitamin K Synthetic cannabinoids are psychoactive substances affecting the cannabinoid-1 (CB1) brain cell receptor. This same receptor causes the psychoactive effects of tetrahydrocannabinol (THC) found in marijuana. Synthetic cannabinoids are sprayed on dried or shredded plant material. Common street names for the synthetic cannabinoids or "fake weed" include "Green Giant," "K2," and "Spice." Synthetic cannabinoids elevate mood, cause a relaxed state, alter perception, and have psychotic effects including anxiety and confusion, paranoia, hallucinations. Suicidal ideations and violent behavior have also been reported with the synthetic cannabinoids. Additionally, synthetic cannabinoids have been associated with intracranial hemorrhage and immune thrombocytopenia. Synthetic cannabinoids can be contaminated with other substances and drugs, specifically, cathinones. Cathinones mimic the effects of cocaine and methamphetamines and can result in tachycardia and vomiting. Acute kidney injury, rhabdomyolysis, and death have been reported with cathinones. Brodifacoum is another substance that has recently been reported as a contaminant in the synthetic cannabinoids. This substance is thought to enhance the duration of effects from the synthetic cannabinoids. Brodifacoum is a vitamin K antagonist derived from warfarin that is commonly used in rat poison. Warfarin and its derivatives inhibit the enzyme, vitamin K 2,3-epoxide reductase preventing activation of vitamin K. Brodifacoum and other similar agents were developed because of rodent resistance to warfarin. Brodifacoum belongs to a class of agents called superwarfarins that have an extremely prolonged half-life of 16 to 36 days. Brodifacoum toxicity has been reported in accidental and intentional ingestions. Common presentation of patients with brodifacoum poisoning includes hematuria, epistaxis, mucosal, and gastrointestinal hemorrhage. Immediate treatment includes correction of the coagulopathy using intravenous vitamin K. There is a risk of anaphylaxis associated with intravenous vitamin K. This risk can be reduced with slower administration of vitamin K over 30 minutes. Coagulopathy associated with brodifacoum toxicity requires high doses of vitamin K to restore 2,3-epoxide reductase activity. In severe cases of bleeding, intravenous vitamin K and prothrombin complex can be administered. Fresh frozen plasma can also be administered to help increase vitamin K-dependent proteins. The coagulopathy associated with brodifacoum can last for 9 months or more. There is little supporting evidence for the use of recombinant factor VIIa in superwarfarin toxicity. Massive bleeding leading to hemorrhagic shock can develop from brodifacoum poisoning requiring significant amounts of blood products and reversal of the vitamin K antagonism with intravenous vitamin K. Reports in the literature of patients presenting with massive hemorrhage and death from brodifacoum toxicity associated with synthetic cannabinoid use have been reported. In cases of hemorrhagic shock, first line therapy includes aggressive administration of crystalloids to restore intravascular volume and optimize cardiac output. Excessive administration of saline can result in a hyperchloremic metabolic acidosis. Balanced crystalloids such as lactated Ringers solution have lower chloride content and can also be used for volume expansion. Volume expansion with colloids such as albumin is frequently used however agents such as hydroxyethyl starch should be avoided because they can induce or exacerbate a coagulopathy. A balanced transfusion of packed red blood cells, plasma, and platelets in a ratio range of 1:1:1 - 2:1:1 until bleeding is no longer life-threatening is recommended for treatment of hemorrhagic shock requiring transfusion of blood products. Once the coagulopathy is controlled, ongoing treatment with high dose oral vitamin K (phytonadione) should be started because of reversal resistance of the superwarfarins and the prolonged antagonist effects of these agents. Brodifacoum concentrates in liver cells and is lipid soluble with a large volume of distribution. Even with undetectable levels there can be ongoing pharmacologic activity of brodifacoum requiring prolonged treatment with oral vitamin K. The lack of previous history of bleeding and a use of synthetic cannabinoids increases the likelihood that this is not a primary bleeding or a hereditary condition such as hemophilia. Factor IX complex would not be indicated making that incorrect. There is little supporting evidence for the use of recombinant factor VIIa in superwarfarin toxicity and the patient is not anemic making transfusion incorrect. Errata: Content revised for clarification. 2/2020 PREP Pearls Contaminants such as superwarfarins have been identified in synthetic cannabinoids and can result in significant life-threatening coagulopathy and hemorrhage. Treatment for superwarfarin exposure may require months of treatment with oral vitamin K to restore 2-3 epoxide reductase activity. Massive bleeding leading to hemorrhagic shock should be treated with a balanced transfusion of packed red blood cells, plasma, and platelets in a ratio of 1:1:1 - 2:1:1 until bleeding is no longer life-threatening. ABP Content Specifications(s)/Content Area Formulate fluid and blood component management strategy for hemorrhagic shock Suggested Readings Arepally GM, Ortel TL. Bad weed: synthetic cannabinoid associated coagulopathy. Blood. 2019:133(9):902-905. doi: 10.1182/blood-2018-11-876839 Boyack I, Opsha O. Coagulopathy hemorrhage with use of synthetic cannabinoids. Am J Emerg Med. 2018;37(2):374.e3-374.e4. doi: 10.1016/j.ajem.2018.10.016 Karam O, Russell RT, Stricker P, et al. Recommendations on RBC transfusion in critically ill children with nonlife-threatening bleeding or hemorrhage shock from the pediatric critical care transfusion and anemia expertise initiative. Ped Crit Care Med. 2018;19 (9S Suppl 1):S127-S132. doi: 10.1097/PCC.0000000000001605 Lacroix J, Tucci M, Du Pont-Thibodeau G. Red blood cell transfusion decision making in children. Curr Opin Pediatr. 2015;27(3):286-291. doi: 10.1097/MOP.0000000000000221 Schulman S, Furie B. How I treat poisoning with vitamin K antagonists. Blood. 2015;125(3):438-442. doi: 10.1182/blood-2014-08-597781

A 12-year-old who was admitted to the pediatric ward for syncope was found cyanotic and unresponsive. A code blue alert was called and cardiopulmonary resuscitation was initiated followed by intubation of the airway and positive pressure ventilation. Return of spontaneous circulation was established after several doses of epinephrine for asystole. Now moved to the pediatric intensive care unit, he is comatose and in sinus rhythm with a heart rate of 130 beats/min. Rectal temperature is 36.7oC, blood pressure is 105/68 mm Hg, and arterial blood gas analysis reveals pH 7.19, PaO2 65 mm Hg on FiO2 0.45, PaCO2 45 mm Hg, and oxyhemoglobin saturation of 90% while receiving conventional mechanical ventilation. Of the following, the most appropriate intervention at this time is: A. consultation with the surgical team for initiation of extracorporeal life support B. immediate cooling to a temperature of 28oC and maintain cooling for 24 hours C. immediate cooling to a temperature of 33oC and maintain cooling for 2 days D. maintain normothermia

D. maintain normothermia Recent studies have demonstrated that prevention of fever and maintenance of a normal core body temperature is as effective as therapeutic hypothermia in patients who have sustained cardiac arrest. Currently the old nomenclature "therapeutic hypothermia" has been replaced by the new term "targeted temperature management" to indicate that the goal of the intervention is to achieve and maintain the core body temperature within a specific range, including prevention of fever, rather than lowering the body temperature following cardiac arrest. Therapeutic hypothermia refers to the controlled lowering of the core body temperature to a specific range (32-34oC). In early 2000, randomized controlled studies from Australia and Europe demonstrated that when adult patients who had sustained an out-of-hospital cardiac arrest (due to ventricular fibrillation) were cooled to a core body temperature of 33oC, they had better neurological outcome compared to patients who did not get cooled. As a result of these studies, therapeutic hypothermia (33oC for 12-24 hours) was recommended by the American Heart Association for the amelioration of the neurological injury following out-of-hospital cardiac arrest in adults when the initial cardiac rhythm was ventricular fibrillation. However, uncertainty continued regarding patients with other conditions, such as cardiac arrest due to rhythm other than ventricular fibrillation, hypoxic ischemic encephalopathy, submersion injuries, or other causes of cardiac arrest. In 2013, an international study compared induced hypothermia to normothermia in adult patients who had sustained cardiac arrest. The results of these studies demonstrated that there was no statistically significant difference in outcome between the group that underwent therapeutic hypothermia and the normothermia group. Neonatal literature demonstrates that induced hypothermia is associated with improved survival and more favorable neurological outcome following an hypoxic-ischemic event in the newborn. Beyond the neonatal period the data have not shown a clear benefit to induced hypothermia. A Canadian retrospective multicenter study published in 2009 did not show that lowering the body temperature was effective in improving outcomes in children. In fact, induced hypothermia was associated with increased morbidity and mortality. A single-center study from the United States showed similar results. These conflicting results served as the motivation behind initiation of the multicenter Therapeutic Hypothermia After Pediatric Cardiac Arrest (THAPCA) trials in order to evaluate the effectiveness of induced hypothermia in children following in-hospital and out-of-hospital cardiac arrest. The most recent of these trials was a prospective study conducted in 37 pediatric centers in the United States, Canada, and the United Kingdom, and included infants who were older than 48 hours and children up to 18 years of age who had sustained in-hospital cardiac arrest. One group underwent targeted temperature control to 33oC for 48 hours following return of spontaneous circulation. This group was compared to a control group which had temperature maintained at 36.8oC. In both groups active temperature control in the normothermia range were maintained for a total of 120 hours. In the group that underwent cooling, the temperature was then gradually increased towards normal over a period of 16 hours. The primary outcome was survival with a score of 70 or higher on the Vineland Adaptive Behavioral Scale, second edition (VABS-II). The VABS-II has a mean age-corrected score of 100 with a standard deviation of 15. A higher score indicates better overall function. In the in-hospital cardiac arrest THAPCA trial between September 2009 and February 2015, 329 out of 746 eligible patients were enrolled in the study. The trial was aborted on because the interim data analysis by the data safety monitoring board showed futility of the study and the investigators believed that it would be unethical to continue the study. Analysis of the data showed that there was no statistically significant difference between the group that underwent cooling and the group that did not undergo cooling following in-hospital cardiac arrest with regard to survival with good VABS-II scores one year following the cardiac arrest. Also, there was no statistically significant difference with regard to secondary outcomes between the 2 groups of children. The authors concluded that in comatose children who had sustained in-hospital cardiac arrest, therapeutic hypothermia was not associated with any beneficial effects compared to maintaining normothermia in this clinical setting. These findings are consistent with an earlier trial that also demonstrated that therapeutic hypothermia did not have any beneficial effects in children who sustained out-of-hospital cardiac arrest. Thus, the best course of action based on current evidence would suggest that it is best to maintain normothermia in the patient in the vignette and that therapeutic hypothermia would not confer any beneficial effects in the patient in the vignette who had sustained in-hospital cardiac arrest. The patient in the vignette has satisfactory oxygenation and ventilation based on the blood gas analysis while on mechanical ventilation and there are no indications for initiation of extracorporeal life support. While cooling of the core body temperature to 28oC is effective during cardiopulmonary bypass with total circulatory arrest for repair of certain complex congenital cardiac defects, there is no evidence that this approach is beneficial in patients who have sustained in-hospital cardiac arrest. PREP Pearls Therapeutic hypothermia has significant adverse effects if not performed properly. Maintaining normothermia is as effective as hypothermia in children who are victims of cardiac arrest. Deep hypothermia (temperature of 30°C or less) induces cardiac arrhythmias. ABP Content Specifications(s)/Content Area use of therapeutic hypothermia after cardiac arrest Suggested Readings Bernard SA, Gray TW, Buist MD, et al. Treatment of comatose survivors of out-of-hospital cardiac arrest with induced hypothermia. N Engl J Med. 2002;346(8):557-563. doi: 10.1056/NEJMoa003289 Hypothermia after Cardiac Arrest Study Group. Mild therapeutic hypothermia to improve the neurological outcome after cardiac arrest. N Engl J Med. 2002;346(8):549-556. doi: 10.1056/NEJMoa012689 Moler FW, Silverstein FS, Holubkov R, et al THAPCA Trial Investigators. Therapeutic hypothermia after in-hospital cardiac arrest in children. N Engl J Med. 2017;376(4):318-329. doi: 10.1056/NEJMoa1610493

An 8-year-old boy is admitted to the pediatric intensive care unit in direct transfer from his pediatrician's office. This previously healthy boy was seen by his pediatrician 5 days prior to admission for fever, malaise, and mild nausea. His examination findings were nonspecific, and he was treated at home with fluids and an antipyretic. Today, which is his eighth day of illness, he returned to his pediatrician's office. He was febrile, irritable, and lethargic, and he had developed a rash. His pediatrician was concerned for sepsis and gave a dose of ceftriaxone in the office prior to the ambulance arrival. The boy currently appears lethargic and is in moderate respiratory distress. He is febrile (39.8°C), tachycardic, and tachypneic with mild intercostal retractions. Oxygen saturation on 2 L/min of nasal cannula oxygen is 91%. He is intermittently lucid and unable to respond to questions, but he can squeeze a hand on request. His rash is shown (Figure ). Other notable physical findings include meningismus, palpable spleen tip, and some edema of the extremities. He is not icteric, no joints appear enlarged or erythematous, and there are no splinter hemorrhages. Laboratory data are shown: Laboratory TestResult White blood cell count 6,000/µL (6.0 × 109/L) Hemoglobin 11 g/dL (110 g/L) Hematocrit 33% Platelet count 56 × 103/µL (56 × 109/L) Sodium 128 mEq/L (128 mmol/L) Chloride 101 mEq/L (101 mmol/L) Potassium 4 mEq/L (4 mmol/L) Creatinine 1.6 mg/dL (141 µmol/L) BUN 36 mg/dL (12.9 mmol/L) AST 120 U/L ALT 110 U/L Total bilirubin 0.2 mg/dL (3.4 µmol/L) His parents are asked further questions regarding their son's history. Of the following, the piece of historical information that is MOST likely for this patient is that he: A. attended a camp in North Carolina 11 days ago B. had an atrial septal defect device closure as a young child C. repeatedly jumped into a Georgia lake during a swimming outing 3 weeks ago D. went fly fishing in Alaska 1 week ago

A. attended a camp in North Carolina 11 days ago The child in this vignette has the classic findings of Rocky Mountain spotted fever (RMSF). According to the National Notifiable Diseases Surveillance database, which tracks 16 vector-borne diseases, the number of reported tick-borne illnesses doubled from 2004 to 2016, with Lyme disease making up 82% of the reports. Spotted fever rickettsiosis increased 250% during that time period. These increases may be due to improved reporting or increased occurrence. The majority of cases of spotted fever rickettsiosis in the United States are reported in North Carolina, South Carolina, Tennessee, Oklahoma, and Arkansas, but cases have been reported from all states except Alaska and Hawaii. There are 20 known species of Rickettsia worldwide that cause spotted fever. In the United States, RMSF is caused primarily by Rickettsia rickettsii, which is transmitted via the American dog tick (Dermacentor variabilis), brown dog tick (Rhipicephalus sanguineus), and the Rocky Mountain wood tick (Dermacentor andersoni). The adult tick primarily feeds on mammals and transmits the bacteria to humans with 6 to 10 hours of feeding. Infection rates peak in the summer months, although the Centers for Disease Control and Prevention (CDC) notes up to 3% of cases being reported in December to February. Infections have been reported as far north as New England. Tick bites do not cause pain and may go unnoticed. The incubation period is typically 4 to 7 days but may be as long as 14 days. With approximately 3% mortality, RMSF is the most common fatal rickettsial disease. The mortality rate is much higher in patients with glucose-6-phosphate dehydrogenase deficiency or immunocompromise. The causative bacterium, R rickettsia, is not typically recovered in blood smears and does not grow in standard culture media. The obligate intracellular bacteria infect endothelial cells with a resultant diffuse vasculitis (eg, skin, brain, liver, lungs, heart, spleen) leading to increased vascular permeability in small vessels. Interestingly, the bacterium lacks genes to facilitate lipid synthesis or use glucose, thus it must scavenge ATP from host cells. The rash, initially seen as small, blanching, pink macules, appears on the ankles, wrists, and forearms 2 to 4 days after fever is noted. Rash is more common in children than adults. The rash develops into a more maculopapular rash, and by the fifth or sixth day the rash may appear more petechial and can cover the entire body, including palms and soles, although less intense on the face. About half of patients will have rash on the palms and soles of their feet. As with any petechial rash, other life-threatening conditions (eg, Neisseria meningitidis, bacterial endocarditis, immune thrombocytopenic purpura, enteroviral infection) must be included in the differential diagnosis. Children who progress to having a petechial rash are severely ill. Purpuric lesions and gangrene may result in loss of digits. In addition to a rash (which is absent in 20% of patients), patients may exhibit hyponatremia, thrombocytopenia, anemia, an abnormal coagulation profile, and elevated hepatic transaminase levels. Neither the triad of fever/rash/headache nor the triad of fever/rash/known tick bite is a sensitive indicator of RMSF. Coagulation may be abnormal, but disseminated intravascular coagulation is rare. The white blood cell count may be elevated or depressed. Hyponatremia (< 130 mEq/L [130 mm/L]) is common. Elevated transaminase levels are common, but hepatic failure and hyperbilirubinemia are not typical. Elevated levels of creatinine, blood urea nitrogen, and creatine kinase are seen in severe cases. Cerebral spinal fluid analysis may demonstrate a mild pleocytosis. Brain computed tomography may show cerebral edema in severe cases. Brain magnetic resonance imaging more specifically may demonstrate punctate hemorrhages on T2-weighted images. The following signs and symptoms are associated with RMSF: Severe headache Irritability or inconsolability Altered mental status Lethargy Meningismus Seizures Coma Pulmonary edema Myocarditis, heart block Vomiting, diarrhea, abdominal pain Malaise Myalgias Edema Hepatomegaly Splenomegaly Lymphadenopathy Diagnosis is typically empirical initially because testing takes time. Hospitals and public health departments may have the ability to perform immunofluorescence antibody testing on serum. The CDC can perform polymerase chain reaction testing on serum and histochemical examination of skin punch biopsy specimens. Public health departments are able to facilitate this testing at the CDC. Immunofluorescence antibody testing sensitivity improves after 7 to 14 days of illness, but the infection is not detectable in the first week; therefore, confirmation is likely to occur well after treatment has begun. The use of acute and convalescent serum specimens increases the likelihood of definitive diagnosis. Doxycycline is the treatment of choice in infants and children with presumed or confirmed RMSF or when any vector-borne disease is suspected. Chloramphenicol is less effective and associated with the potential for development of aplastic anemia. Doxycycline also will treat ehrlichiosis, which chloramphenicol does not. It is a common misconception that doxycycline stains tooth enamel. Delay in initiating appropriate antibiotic treatment, even by a few days, is associated with a higher mortality rate. Death is usually the result of multisystem organ failure or cerebral edema with elevated intracranial pressure. In survivors of severe disease, neurologic deficits (eg, ataxia, cortical blindness, dysarthria) are seen in 15% of patients, but a wide spectrum of neurologic impairments may be seen transiently. A history of jumping into a lake may lead to the consideration of an infection with Naegleria; however, the rash, laboratory findings, and timeline of clinical presentation in this vignette are not consistent with Naegleria infection. The patient's rash is not typical of endocarditis. Fishing in Alaska is not associated with any particular rash. PREP Pearls Treatment with doxycycline should never be delayed if a vector-borne illness is suspected. Treatment for Neisseria meningitidis with a third-generation cephalosporin should be administered until that diagnosis can be ruled out in a child with a petechial rash. A delay in treatment greatly increases the risk of severe or fatal Rocky Mountain spotted fever disease. ABP Content Specifications(s)/Content Area Understand the pathophysiology of Rocky Mountain spotted fever Know the clinical features and manifestations of Rocky Mountain spotted fever Plan treatment for a patient with Rocky Mountain spotted fever Suggested Readings Biggs HM, Behravesh CB, Bradley KK, et al. Diagnosis and management of tickborne rickettsial diseases: Rocky Mountain spotted fever and other spotted fever group rickettsioses, ehrlichioses, and anaplasmosis — United States. MMWR Recomm Rep. 2016;65(2):1-44. doi: http://dx.doi.org/10.15585/mmwr.rr6502a1 Buckingham SC. Tick-borne diseases of the USA: ten things clinicians should know. J Infect. 2015;71 Suppl 1):S88-S96. doi: 10.1016/j.jinf.2015.04.009 Gottlieb M, Long B, Koyfman A. The evaluation and management of Rocky Mountain spotted fever in the emergency department: a review of the literature. J Emerg Med. 2018;55(1):42-50. doi:10.1016/j.jemermed.2018.02.043 McFee RB. Tick borne illness—Rocky Mountain spotted fever. Dis Mon. 2018;64(5):185-194. doi: 10.1016/j.disamonth.2018.01.006 Woods CR. Rocky Mountain spotted fever in children. Pediatr Clin North Am. 2013;60(2):455-470. doi: 10.1016/j.pcl.2012.12.001

A 7-year-old boy is admitted to the intensive care unit from the oncology service. The oncology team describes a progressive clinical decline with fevers, tachypnea, increased work of breathing, and oxygen requirement despite broad-spectrum antibiotic therapy. The child received a hematopoietic stem cell transplant 7 days earlier, and his absolute neutrophil count was 20/µL (0.02 × 109/L) this morning. Because of concerns for infection, he underwent bronchoscopy and broncho-alveolar lavage earlier today, during which he had worsening oxygenation and lung compliance. His initial chest radiograph in the intensive care unit shows diffuse alveolar infiltrates without cardiomegaly. Of the following, the organism MOST likely to be the cause of this patient's respiratory failure is A. Candida species B. cytomegalovirus C. Epstein-Barr virus D. Pneumocystis jirovecii

A. Candida species The child in the vignette is in the pre-engraftment phase with severe neutropenia and is at high risk of an infection with Candida. Cytomegalovirus and Pneumocystis are more likely to emerge during the engraftment phase. Epstein-Barr virus-related posttransplant proliferative disorder tends to occur much later in the transplant course. Children who have undergone hematopoietic stem cell transplantation (HSCT) represent a unique population in the intensive care unit. They have a high risk of pulmonary complications requiring invasive mechanical ventilation, which, in turn, is associated with a high risk of death. While the mortality risk has improved significantly over the past few decades, it remains high at 60% to 70% in recent observational studies. The cause of respiratory failure after HSCT depends on multiple factors, including the underlying reason for HSCT, the conditioning required before transplant, and the timing after transplant. Given their immunocompromised state associated with HSCT and its therapies, children who have undergone transplantation are at increased risk of infections. The infections can involve the respiratory system (eg, pneumonia or pneumonitis, lung abscess) or be associated with an inflammatory state leading to respiratory failure (eg, acute respiratory distress syndrome). Infectious causes are commonly divided into 3 phases: (1) pre-engraftment, (2) engraftment (30-100 days after HSCT), and (3) late phase (more than 100 days after HSCT) Figure. Common infections associated with immunocompromised state after hematopoietic stem cell transplantation. EBV = Epstein-Barr virus; HHV = human herpesvirus; NK = natural killer; PTLD = posttransplant lymphoproliferative disease Reprinted with permission from Tomblyn M, Chiller T, Einsele H, et al. Guidelines for preventing infectious complications among hematopoietic cell transplantation recipients: a global perspective. Biol Blood Marrow Transplant. 2009;15(10):1152. During the pre-engraftment period, children have severe neutropenia and breakdown of their mucosal barriers from mucositis. These conditions increase the risk of both gram-positive and gram-negative bacterial infections as well as reactivation of herpes simplex virus. In addition, children are at high risk of opportunistic fungal infections, particularly from Candida and Aspergillus organisms. During the engraftment phase, patients have early immune recovery with resolving neutropenia. Therefore, their risk of bacterial infection remains elevated but is much lower than during pre-engraftment. However, the risk of invasive Aspergillus organisms remains elevated for several months. Additionally, although these patients are no longer neutropenic during the engraftment phase, their cell-mediated immunity remains impaired, making infection or reactivation of cytomegalovirus (CMV) common, especially among those who are CMV seropositive before transplant. Varicella infection can also occur with greater frequency during this time, as can Pneumocystis jirovecii. After the engraftment period, full immunologic recovery can take years, especially for those children who have graft-vs-host-disease. They remain at elevated risk of severe viral and fungal infections, and because full recovery of humoral immunity is also delayed, they are also at greater risk of severe infections with encapsulated organisms (eg, Streptococcus pneumoniae, Haemophilus influenzae, and Neisseria meningitides). There are also numerous noninfectious causes of respiratory failure in the HSCT population (Table). Early in the post-HSCT course, children can have severe mucositis leading to upper airway obstruction. They can also have severe pulmonary edema secondary to fluid overload associated with chemotherapy or kidney injury, or increased capillary permeability related to irradiation or pulmonary toxicity from chemotherapy. The term idiopathic pneumonia syndrome (IPS) has been used to describe a spectrum of lung injury after transplant that is noninfectious. A 2011 American Thoracic Society Research Statement updated the definition of IPS to include: (1) evidence of widespread alveolar injury; (2) absence of active infection; and (3) absence of other causes of pulmonary edema such as cardiac failure, acute kidney injury, and iatrogenic fluid overload. Under the spectrum of IPS are many clinical entities (Table). Diffuse alveolar hemorrhage occurs early in the post-HSCT period (less than 30 days). Diffuse alveolar hemorrhage is diagnosed with progressively bloody bronchoalveolar lavage fluid. Periengraftment respiratory distress syndrome occurs, by definition, within 5 days of engraftment, although it tends to be a diagnosis of exclusion. Additionally, IPS can present with an interstitial pneumonia pattern early in the posttransplant course. Bronchiolitis obliterans, cryptogenic organizing pneumonia (previously known as bronchiolitis obliterans organizing pneumonia), pulmonary veno-occlusive disease, posttransplant lymphoproliferative disorder (often related to Epstein-Barr virus), and pulmonary alveolar proteinosis are all late complications of HSCT. Bronchiolitis obliterans is characterized by chronic lower airway obstruction with areas of bronchiectasis and airway fibrosis, and is often associated with chronic graft-vs-host-disease. Cryptogenic organizing pneumonia occurs with lower frequency, is notable for plugs of tissue in the respiratory bronchioles and alveolar ducts, and is much more responsive to corticosteroid treatment than is bronchiolitis obliterans. PREP Pearls Children with acute respiratory failure after hematopoietic stem cell transplant have a high risk of death. The causes of respiratory failure depends on the underlying disease requiring hematopoietic stem cell transplant, the conditioning treatment required for transplant, and the time since transplant. Idiopathic pneumonia syndrome is a clinical spectrum of noninfectious lung injury. ABP Content Specifications(s)/Content Area Understand the prognosis of respiratory failure after hematopoietic stem cell transplantation Understand the etiologies of respiratory failure after hematopoietic stem cell transplantation and differences by timing of presentation Suggested Readings Haddad IY. Stem cell transplantation and lung dysfunction. Curr Opin Pediatr. 2013;25(3):350-356. doi:10.1097/MOP.0b013e328360c317 Michelson PH, Goyal R, Kurland G. Pulmonary complications of haematopoietic cell transplantation in children. Paediatr Resp Rev. 2007;8(1):46-61. doi:10.1016/j.prrv.2006.04.003 Panoskaltsis-Mortari A, Griese M, Madtes DK, et al; American Thoracic Society Committee on Idiopathic Pneumonia Syndrome. An official American Thoracic Society Research Statement: Noninfectious lung injury after hematopoietic stem cell transplantation: idiopathic pneumonia syndrome. Am J Respir Crit Care Med. 2011;183(9):1262-1279. doi:10.1164/rccm.2007-413ST Rowan CM, Gertz SJ, McArthur J, et al; Investigators of the Pediatric Acute Lung Injury and Sepsis Network. Invasive mechanical ventilation and mortality in pediatric hematopoietic stem cell transplantation: a multicenter study. Pediatr Crit Care Med. 2016;17(4):294-302. doi:10.1097/PCC.0000000000000673

A 4-month old boy is brought to the emergency department for respiratory distress. History reveals a normal pregnancy and birth, with the infant discharged from the nursery at 48 hours. The family has not seen their primary care physician since birth. The parents describe the infant as having always fed poorly. The infant's examination reveals diffuse hypotonia and patellar areflexia. A harsh systolic murmur is noted. No hepatosplenomegaly is detected. No skin findings are seen. Chest radiograph shows an enlarged cardiac silhouette (Figure 1). Electrocardiogram is shown (Figure 2). Laboratory analysis shows normal electrolytes, lactate, and ammonia levels; however, creatine kinase levels are significantly elevated. Of the following, the enzyme replacement therapy that should be considered in this clinical scenario is: A. acid-α glucosidase B. α-galactosidase C. α-L-iduronidase D. glucocerebrosidase

A. acid-α glucosidase Inborn errors of metabolism can frequently present as shock or respiratory distress in infants. The infant in this clinical vignette demonstrates typical findings for Pompe disease, a lysosomal storage disorder. Pompe disease is also known as Glycogen Storage Disease Type II or acid maltase deficiency. The enzymatic defect in Pompe disease is due to lack of acid-α glucosidase. This enzyme cleaves the α-1,4- and 1,6- bonds of glycogen, maltose, and isomaltose. In the absence of this enzyme, glycogen accumulates in lysosomes, particularly in muscle tissue and less commonly in liver tissue. To date, more than 300 mutations and polymorphisms to the genes encoding for acid-α glucosidase have been detected and are located in the GAA gene sequence on chromosome 17. The incidence of Pompe disease in the United States is 1 in 40,000 individuals. Among African-Americans, it can be as high as 1 in 14,000. Three variants of Pompe disease are described. The most common variant presents before age one year and is known as the infantile-onset. Pompe disease can also present in later childhood, and recently several cases have been described in adults (late-onset). All infantile-onset cases will demonstrate cardiomyopathy, often resulting in left ventricular outflow tract obstruction. Additionally, they will typically have a history of poor feeding, muscle weakness, hyporeflexia, and a history of repeated respiratory issues or upper airway infections. Prognosis for the infantile-onset form is poor. Most infantile onset patients succumb to respiratory insufficiency or cardiac failure within the first year of life. Late-onset Pompe disease presents in childhood or early adulthood with proximal muscle weakness and respiratory insufficiency. Severe scoliosis and lordosis are common findings. Many late-onset patients are wheelchair dependent and require assisted ventilation. Cardiac findings are much less common in late-onset Pompe disease. Lysosomal enzyme deficiencies are commonly categorized based on the substances stored within the cell due to lack of enzymatic degradation. In Pompe disease, excess carbohydrates are stored as glycogen within lysosomes in cardiac and skeletal muscle. Pompe disease was originally recognized in the 1930's by Johannes Pompe. He described an infant with generalized weakness who died of hypertrophic cardiomyopathy. On autopsy, he noted large amounts of glycogen in all tissues examined. While not diagnostic, serum creatine kinase levels are significantly elevated in all cases of infantile-onset Pompe disease, often greater than 200 times normal. Additionally, elevations of urine tetrasaccharide is another common laboratory finding in infantile-onset Pompe disease. However, the disease is confirmed only through detection of deficient acid-α glucosidase activity. In infantile-onset Pompe disease, acid-α glucosidase enzymatic activity levels are <1% normal. In late-onset disease, activity can be 2% to 40%. Glycogen accumulation and hypertrophy of cardiac muscle begins in utero and becomes clinically significant in the first few months of life. Cardiomegaly is noted on chest radiographs. Electrocardiogram abnormalities are common and typically show a short PR interval and significant elevation in QRS complexes across all cardiac leads. Bradycardia may be noted. Conduction abnormalities can also be present, producing tachyarrhythmia during times of stress such as dehydration or infection. Enzyme replacement therapy is currently considered the best treatment option for these patients. This form of therapy provides a recombinant acid-α glucosidase analog which may decrease glycogen deposition in muscle tissue. In 2007, an alglucosidase alfa drug became commercially available in the United States. It was hoped that all muscle tissue would show improved function. Interestingly, cardiac muscle demonstrated a far better response than skeletal muscle. This drug has significantly altered the natural outcome of the infantile-onset variant, causing marked improvement in cardiac function and lessening cardiac hypertrophy among recipients. Most infantile-onset patients now live beyond a year of age, but still suffer from gradual worsening of skeletal muscle myopathy. It is uniformly agreed that treatment with enzyme replacement therapy should begin soon after diagnosis. As such, Pompe disease was added to the Recommended Uniform Screening Panel for newborn screening in 2014. Cochrane database analysis suggests that infants started on enzyme replacement demonstrated improved mortality, cardiac function, and required less ventilation support. They could not however determine best dosing requirement or duration of therapy. The appropriate time to begin enzyme replacement therapy for late-onset disease is still under investigation. Recent advances in biochemical understanding and pharmacotherapy have resulted in enzyme replacement therapies for a number of genetic conditions including Gaucher disease, Fabry disease, and Hurler syndrome. One of the first clinical uses of enzyme replacement therapy for inborn errors of metabolism involved Gaucher disease. Gaucher disease is the most common lysosomal storage disease and is caused by a lack of glucocerebrosidase activity, resulting in an accumulation of glucocerebroside 1 in reticuloendothelial tissue. Gaucher disease tends to present with hepatosplenomegaly and skeletal lesions. Treatment with recombinant human glucocerebrosidase began in the 1990's and continues to be a mainstay of therapy. Fabry disease is an x-linked disorder of glycosphingolipid catabolism. It is caused by lack of α-galactosidase A activity and results in angiokeratomas in the skin, opacities in the cornea or lens, and worsening renal failure. Initial clinical presentation is usually in adolescence. Enzyme replacement therapies with α-galactosidase is now considered standard of care. Mucopolysaccharidosis type I (MPS I) or Hurler syndrome is an autosomal recessive disorder caused by lack of α-L-iduronidase activity. This results in accumulation of glycosaminoglycans within various tissues. Mucopolysaccharidosis type I infants appear normal at birth, but gradually develop coarsening of facial features, skeletal dysplasia, hearing loss, and intellectual decline. Treatments for Mucopolysaccharidosis type I include hematopoietic stem cell transplantation and enzyme replacement therapy with α-L-iduronidase. PREP Pearls Pompe disease is an autosomal recessive disorder causing varying degrees of deficiency in α 1,4 glucosidase; lack of α 1,4 activity inversely correlates with age of clinical onset of disease. Cardiac hypertrophy is a common finding in infantile-onset Pompe disease. Enzyme replacement therapy significantly improves cardiac pathophysiology, but has limited effect on skeletal muscular myopathy. ABP Content Specifications(s)/Content Area Recognize Pompe disease (glycogen storage disease type II) as a cause of cardiomyopathy and hypotonia Suggested Readings Chen M, Zhang L, Quan S. Enzyme replacement therapy for infantile-onset Pompe disease. Cochrane Database Syst Rev. 2017;11:CD011539. doi: 10.1002/14651858.CD011539.pub2 Kishnani PS, Hwu WL, Mandel H, Nicolino M, Yong F, Corzo; Infantile-Onset Pompe Disease Natural History Study Group. A retrospective, multinational, multicenter study on the natural history of infantile-onset Pompe disease. J Pediatr. 2006;148(5):671-676. doi: 10.1016/j.jpeds.2005.11.033 Lim JA, Li L, and Raben N. Pompe disease: from pathophysiology to therapy and back again. Front Aging Neurosci. 2014;6;177. doi: 10.3389/fnagi.2014.00177

A 3-year-old girl is brought to the emergency department following the ingestion of an unknown substance. The child lives with her parents and an older sibling who has multiple medical problems, including cerebral palsy, intractable seizures, and chronic respiratory insufficiency. The sibling is on levetiracetam, glycopyrrolate, baclofen, and diazepam. Today the 3-year-old was found drinking one of her sibling's medications. The mother rushed the child to the emergency department, but in her haste, she forgot to note which medication the toddler was ingesting. In the emergency department the patient is agitated and restless. Her temperature is 38.3℃, heart rate 131 beats/min, respiratory rate 35 breaths/min, and blood pressure 129/77 mm Hg. Oxygen saturation is 98% on room air. Her skin is flushed and dry, and pupils are dilated. Of the following, this patient's symptoms are MOST likely due to: A. antagonism of muscarinic receptors B. antagonism of muscle-type nicotinic receptors C. stimulation of α-adrenergic receptors D. stimulation of β-adrenergic receptors

A. antagonism of muscarinic receptors The child's symptoms are consistent with the ingestion of glycopyrrolate, an anticholinergic agent with muscarinic receptor selectivity, used to control airway secretions in patients with cerebral palsy. Adverse effects from glycopyrrolate toxicity stem from the drug's antimuscarinic effects and include hyperthermia, tachycardia, tachypnea, flushed, dry skin, and mydriasis. Acetylcholine receptors are classified as muscarinic or nicotinic, based on their affinity for muscarine and nicotine. Five subgroups of muscarinic receptors and 2 subgroups of nicotinic receptors have been described. Muscarinic receptors are located at target organs innervated by parasympathetic fibers, including heart, smooth muscle, and exocrine glands, as well as sweat glands innervated by sympathetic fibers. Muscarinic receptors are also scattered throughout the central nervous system, although their physiologic significance is not well-characterized. Nicotinic receptors are located at the neuromuscular junctions (NMJ) of skeletal muscles (muscle-type) and at all presynaptic ganglia of the autonomic nervous system (ganglion-type). Stimulation of muscarinic receptors leads primarily to parasympathetic responses, including decreased heart rate and blood pressure, increased glandular secretions, enhanced gastrointestinal motility, bronchoconstriction, and pupillary constriction. Stimulation of nicotinic receptors leads to contraction of skeletal muscles via muscle-type receptors, as well as a complex array of physiologic responses through ganglion-type receptors. The table provided summarizes the locations and physiologic effects of cholinergic receptors, and includes agonists and antagonists at these sites. Anticholinergic agents are divided into muscarinic and nicotinic subtypes based on their receptor affinities. Muscarinic antagonists competitively bind muscarinic receptors and prevent acetylcholine from activating the receptor. Examples of such agents include atropine, scopolamine, glycopyrrolate, and ipratropium. Clinical consequences are primarily due to their anti-parasympathetic effects, and include tachycardia, hypertension, decreased secretions, mydriasis, and diminished gastrointestinal motility. Pharmacologically, antimuscarinics can be used to control motion sickness and emesis (scopolamine), reduce airway secretions (glycopyrrolate), manage cycloplegia (atropine), cause bronchodilation (ipratropium), reduce urinary frequency (oxybutynin), and treat sinus bradycardia and atrioventricular heart block (atropine). Toxic effects of antimuscarinics can include cardiac arrhythmias, as well as the anticholinergic syndrome (ie, hyperthermia, tachycardia, delirium, dry skin, flushing, mydriasis, decreased bowel sounds, urinary retention). Nicotinic antagonists include ganglion blockers and NMJ blockers. Ganglion blockers competitively block acetylcholine at nicotinic receptors of both parasympathetic and sympathetic autonomic ganglia, and as such, block all autonomic outflow. Their lack of selectivity leads to multiple undesirable effects, and they have little clinical use. Examples of such agents include hexamethonium, mecamylamine, and tubocurarine. Physiologic effects are complex but can include sedation, tremor, cycloplegia, pupillary dilation, decreased vascular tone, hypotension, decreased cardiac contractility, tachycardia, impaired gastrointestinal motility, and urinary retention. Nicotinic antagonists selective for muscle-type receptors (NMJ blockers) include depolarizing agents (ie,succinylcholine) and nondepolarizing agents (ie, vecuronium, rocuronium, pancuronium, atracurium, cisatracurium). Depolarizing agents mimic acetylcholine and occupy receptor binding sites. Nondepolarizing agents compete with acetylcholine at receptor sites, preventing receptor activation. With the exception of the iris sphincter muscle, the result of both types of NMJ blockade is complete muscle paralysis. These agents can be useful in situations where pharmacologic paralysis may facilitate intubation, mechanical ventilation, procedures that require immobilization, or to ensure safety during transport of patients on mechanical ventilation. These agents should be used in conjunction with sedatives and analgesics whenever possible. Adverse effects of NMJ blockers include fasciculations (ie, succinylcholine) and prolonged muscle weakness. Certain metabolic derangements can also prolong the effects of NMJ blockers, such as hypothermia, hyponatremia, hypocalcemia, and hypokalemia. The patient in the vignette has signs and symptoms of anticholinergic toxicity caused by glycopyrrolate, a selective muscarinic antagonist. Levetiracetam, baclofen, and diazepam do not cause this complex of symptoms. Inhibition of muscle-type nicotinic receptors would cause paralysis. Neither the stimulation of α-adrenergic receptors or the stimulation of β-adrenergic receptors would lead to anticholinergic symptoms. PREP Pearls Muscarinic receptors are located primarily at target organs innervated by parasympathetic fibers; nicotinic receptors are located at the neuromuscular junction (muscle-type) and autonomic ganglia (ganglion-type). Blockade of muscarinic receptors can lead to tachycardia, arrhythmias, and anticholinergic syndrome. Nicotinic blockade leads to muscle paralysis at muscle-type receptors, and complete blockade of autonomic outflow at ganglion-type receptors. ABP Content Specifications(s)/Content Area Differentiate between the actions of nicotinic and muscarinic acetylcholine blockers and sites of action at autonomic ganglia, neuromuscular junction, and visceral organs Suggested Readings Kruse AC, Kobilka BK, Gautam D, et al. Muscarinic acetylcholine receptors: novel opportunities for drug development. Nat Rev Drug Discov. 2014;13(7):549-560. doi:10.1038/nrd4295 Pappano JA. Cholinoceptor-Blocking Drugs. In: Katzung BG, ed. Basic & Clinical Pharmacology. 14th ed. New York, NY: McGraw-Hill Education; 2018:124-135. Sine SM. End-plate acetylcholine receptor: structure, mechanism, pharmacology, and disease. Physiol Rev. 2012;92(3):1189-1234. doi:10.1152/physrev.00015.2011 Zuppa AF, Barrett JS. Pharmacology. In: Nichols DG, ed. Rogers' Textbook of Pediatric Intensive Care. 4th ed. Philadelphia, PA: Wolter Kluwers: Lippincott Williams & Wilkins; 2008:266-281.

A 6-year-old girl with trisomy 21 is admitted to the PICU for close respiratory and cardiovascular monitoring following tonsillectomy. She was extubated in the recovery room without difficulty, but demonstrates mild hypotension and bradycardia. Her medical history is significant for atrioventricular canal defect repair at 6 months of age, as well as reactive airway disease for which she requires daily inhaled corticosteroids and occasional treatments with aerosolized bronchodilators. Upon arrival to the PICU she is resting comfortably on room air. However, she remains hypotensive and bradycardic despite awakening from anesthesia, and a narrow pulse pressure on her arterial line pressure monitor is noted. She is mildly obese and has trisomy 21 features. She is afebrile, awakens easily, and is in no acute distress. There are no murmurs or gallops on cardiac auscultation. Pulmonary examination findings are normal. She has no focal neurologic deficits, but she does have general hyporeflexia and delayed relaxation of her patellar reflexes bilaterally. Her skin is dry, and she has mild nonpitting edema in her lower extremities. Of the following, the MOST likely finding in this patient is A. antithyroid peroxidase antibodies B. low plasma cortisol levels C. pericardial effusion D. pulmonary edema

A. antithyroid peroxidase antibodies The patient in this vignette has signs consistent with hypothyroidism, as evidenced by hypotension, bradycardia, narrow pulse pressure, hyporeflexia, dry skin, and nonpitting edema. The most likely cause of her hypothyroidism is autoimmune thyroiditis, which is commonly associated with trisomy 21 and marked by the presence of antithyroid antibodies on serum analysis. Hypothyroidism in children can be congenital or acquired and can be further divided into primary diseases of the thyroid and central causes, including disorders of the pituitary and hypothalamus. Causes of primary and central acquired hypothyroidism are listed in Table 1. The most common (and preventable) cause worldwide is iodine deficiency. In iodine-sufficient countries, the most common cause of acquired hypothyroidism is autoimmune thyroiditis, also known as Hashimoto disease or lymphocytic thyroiditis. There is an increased frequency of acquired hypothyroidism associated with specific chromosomal disorders, including trisomy 21, Turner syndrome, and Klinefelter syndrome. The most common cause in these conditions is autoimmune thyroiditis. Other causes of acquired hypothyroidism include exposure to goitrogenic agents, which include certain medications and environmental toxins (Table 1). The clinical presentation of hypothyroidism is often insidious, and early findings may be subtle. Signs and symptoms of acquired hypothyroidism are listed in Table 2. Clinical features vary by disease process, but common findings include lethargy, bradycardia, cutaneous changes, cold intolerance, and constipation. Neurologic changes, such as hyporeflexia and delayed relaxation of deep tendon reflexes, usually occur later. Delayed bone age or below-normal growth velocity should raise suspicion for hypothyroidism. Myxedematous features including nonpitting edema are characteristic of more long-standing disease. Physical examination may reveal the presence of goiter (depending on etiology), but absence of goiter does not exclude the condition. Diagnostic testing should reveal elevated thyrotropin levels in primary hypothyroidism, and this finding is often the earliest laboratory abnormality. The thyrotropin level may be low, normal, or elevated in cases of central hypothyroidism. Thyroxine concentrations are usually low, but may be normal early in the disease course. Autoimmune thyroiditis is associated with antithyroid antibodies (antithyroid peroxidase or antithyroglobulin) in most cases, and these markers should be measured in any patient suspected to have this condition. The most emergent manifestation of hypothyroidism is myxedema coma, a term used to describe critical hypothyroidism characterized by severe respiratory, cardiovascular, and neurologic compromise. This term is a misnomer, as most patients with critical hypothyroidism do not present with myxedematous features or in a comatose state. This condition is extremely rare in pediatrics and more commonly associated with congenital hypothyroidism rather than acquired. However, it can rarely occur as the initial presentation of acquired hypothyroidism, particularly if precipitated by stresses such as surgery, infection, trauma, and certain medications. Critical hypothyroidism is characterized by progressive organ dysfunction that, if untreated, leads to multisystem organ failure and death. Common findings include altered mental status, hypothermia, hypoventilation, bradycardia, and severe hypotension. Gastrointestinal, renal, and hematologic manifestations may occur. Early recognition is crucial, as without treatment the mortality rate is extremely high. Management involves supportive measures including respiratory and cardiovascular stabilization, correction of physiologic derangements, and immediate treatment with intravenous thyroid hormone. Hydrocortisone should be considered in such patients, as cortisol responses are blunted during critical hypothyroidism. The patient in this vignette likely had subtle signs and symptoms of hypothyroidism prior to her tonsillectomy, including cutaneous changes and nonpitting edema. More obvious and clinically significant features can be precipitated by stress, such as sedation or surgical procedure. As patients with trisomy 21 are at high risk for developing hypothyroidism, this condition should be suspected in any affected patient with hemodynamic instability following surgery. Her condition has not progressed to critical hypothyroidism, but recognition and treatment are imperative to prevent further decompensation. The American Thyroid Association Task Force recommends monotherapy with levothyroxine as primary management for children with hypothyroidism. Treatment should be initiated and further directed with the guidance of a pediatric endocrinologist. While this child has a history of congenital heart disease, a pericardial effusion would not be expected to occur several years following repair in this setting. Postobstructive (negative pressure) pulmonary edema can occur following tonsillectomy, but it would present with physical findings on lung examination and significant respiratory distress. Adrenal insufficiency should be considered in any critically ill patient with hemodynamic instability but is not associated with the neurologic changes or other symptoms described in this patient. PREP Pearls Autoimmune thyroiditis is the most common cause of acquired hypothyroidism in the United States, while iodine deficiency is the most common cause worldwide. Hypothyroidism is commonly associated with congenital conditions (eg, trisomy 21, Klinefelter syndrome, Turner syndrome) and can be caused by many medications. Signs and symptoms may be subtle, thus clinicians must have a low threshold for screening. Myxedema coma is rare in children but is a true endocrine emergency; immediate recognition and treatment are essential to avoid complications. ABP Content Specifications(s)/Content Area Understand the association between acquired hypothyroidism and the use of specific drugs (eg, amiodarone) or specific conditions (eg, trisomy 21) Recognize the manifestations of acquired hypothyroidism Suggested Readings Demirbilek H, Kandemir N, Gonc EN, Ozon A, Alikasifoglu A, Yordam N. Hashimoto's thyroiditis in children and adolescents: a retrospective study on clinical, epidemiological and laboratory properties of the disease. Pediatr Endocrinol Metab. 2011;20(11):1199-1205. doi:10.1515/JPEM.2007.20.11.1199 Jonklaas J, Bianco AC, Bauer AJ, et al. Guidelines for the treatment of hypothyroidism: prepared by the American Thyroid Association Task Force on Thyroid Hormone Replacement. Thyroid. 2014;24(12):1670-1751. doi:10.1089/thy.2014.0028 Ulate KP, Zimmerman JJ. Common endocrinopathies in the pediatric intensive care unit. In: Fuhrman BP, Zimmerman JJ, Carcillo JA, eds. Pediatric Critical Care. 4th ed. Philadelphia, PA: Saunders Elsevier; 2011:1121-1123. Zhu Y, Qiu W, Deng M, Zhu X. Myxedema coma: a case report of pediatric emergency care. Medicine. 2017;96(21):e6952. doi:10.1097/MD.0000000000006952

A 17-year-old previously healthy boy is transferred to the pediatric intensive care unit from the general pediatric floor for worsening generalized weakness. Five days before the transfer to the pediatric intensive care unit, the patient was brought to the emergency department for "passing out" after binge consumption of beer. The only significant initial laboratory finding at that time was a serum sodium of 115 mEq/L. Even though the patient had no other symptoms, a 3% hypertonic saline bolus was given, and the patient started on D5NS for maintenance IV fluid. The repeat serum sodium 2 hours later was 127 mEq/L. The patient was admitted to the general pediatric floor after improvement in his neurological status was noted. Over the next 2 days the patient's serum sodium was corrected to 145 mEq/L. On the day of transfer to the pediatric intensive care unit, the patient was more sleepy and demonstrated dysarthria and generalized muscle weakness. Physical examination reveals normal vital signs, along with normal heart and lung examination. He is delirious with spastic quadriparesis, but brisk deep tendon reflexes. The pupils are equal, round, and reactive to light. Brain CT performed en route to the pediatric intensive care unit was read as normal. The patient is afebrile with no meningeal signs. Of the following, the findings in this patient are most likely explained by: A. central pontine myelinolysis B. diabetic ketoacidosis C. neuropathy due to chronic alcohol abuse D. tick bite paralysis

A. central pontine myelinolysis The patient described above has signs suggestive of pseudobulbar palsy (dysarthria, dysphagia), which indicates bilateral involvement of corticobulbar tracts from their origin in the cerebral cortex to their termination in the brainstem. The pseudobulbar palsy along with the spastic tetraparesis (likely due to involvement of the corticospinal tracts) is highly suggestive of central pontine myelinolysis, which is a condition caused by the rapid correction of serum hyponatremia. Hyponatremia (serum sodium < 125 MEq/l) is classified based on patient's volume status is shown in the Table. Hyponatremia, which develops within a 48-hour period, is considered to be acute hyponatremia. The immediate effect of acute hyponatremia is water gain in the neurons of the brain. The neurons will adapt by rapidly extruding sodium, potassium, and chloride to decrease osmolality and cerebral edema. If hyponatremia develops over a period of time >48 hours (chronic) as in the patient described in the vignette, the neurons undergo a slow adaptation process that involves loss of organic osmolytes to reduce cerebral edema. This stage of slow adaptation that protects the brain is also the period when the brain is most vulnerable to injury, especially if aggressive rapid correction of chronic hyponatremia is provided to the patient. Rapid correction of chronic hyponatremia can cause neurons to shrink as there is neuronal water loss due to the gradient created by the increased serum sodium. The shrinkage of neurons along with demyelination within the pons is called central pontine myelinolysis. A patient with central pontine myelinolysis can demonstrate altered mental status, cranial nerve deficits, flaccid quadriplegia, irreversible neurologic damage, and even death. Extrapontine involvement can result in a locked in syndrome in which the patient is conscious but unable to produce voluntary response. Magnetic resonance imaging shows hyperintensity on T2-weighted sequences with corresponding hypointensity on T1-weighted sequences. There is usually no enhancement with gadolinium. The changes in T2-weighted sequences may not become evident in the first 2 weeks hence the more sensitive diffusion-weighted magnetic resonance imaging should be used within the first 2 weeks of the insult. Patients with chronic alcoholism, malnutrition, and burns have a higher likelihood of developing central pontine myelinolysis with rapid correction of chronic hyponatremia. As there is no therapy for central pontine myelinolysis, the best approach is prevention by early identification of at-risk patients, slow correction of chronic hyponatremia, and frequent evaluation of serum sodium levels. Central pontine myelinolysis patients who survive have severe neurological sequelae. The management of hyponatremia essentially involves: 1) treatment of the underlying cause, and 2) slow correction of hyponatremia. Most clinicians advocate correction by 12-15 mEq/L in a 24-hour period. The only exception to the slow correction of hyponatremia is the presence of acute neurologic symptoms such as seizures, headache, or hallucinations. In this case a rapid 3% hypertonic saline bolus should be given to correct the serum sodium to 125 mEq/L, which is considered as the seizure threshold for serum sodium. Hypovolemic hyponatremia is treated by replacing the sodium loss, and restoration of intravascular volume with a saline bolus. The saline bolus inhibits production of antidiuretic hormone allowing for diuresis of any excess water. In patients with diarrhea the ongoing losses need to be replaced in addition to providing for maintenance requirements. Therapy for euvolemic hyponatremia involves restriction of fluids in conditions of water retention states such as the syndrome of inappropriate antidiuresis. Management of hypervolemic hyponatremia involves judicious removal of the excess water without allowing for a drop in serum sodium. Diuretics (sometimes even preceded by 25% albumin) may be needed in hypervolemic states such as nephrotic syndrome. Therapy for heart failure can improve renal function in heart failure resulting in loss of fluid and increase in serum sodium. Sometimes fluid restriction becomes necessary especially in states of chronic renal insufficiency with volume overload. Tick paralysis classically presents with an ascending flaccid paralysis with absent deep tendon reflexes. The patient in the vignette is not a diabetic and has no history of polyuria, polydipsia or polyphagia. Additionally the initial laboratory findings in the emergency department were normal except for the hyponatremia, making diabetic ketoacidosis unlikely. Peripheral neuropathy from chronic alcohol abuse is usually seen in patients >40 years old who have a long history of alcohol abuse. Patients have tingling numbness, paresthesias, cramps, weakness, muscle atrophy, and pain usually in the upper and lower extremities. Bulbar involvement is less likely and so is the acute onset as seen in this patient. PREP Pearls Chronic asymptomatic hyponatremia should be corrected slowly in order to prevent cerebral pontine myelinolysis. Symptomatic hyponatremia should be treated using 3% hypertonic saline in order to prevent neurologic injury. Syndrome of inappropriate antidiuresis is a diagnosis of exclusion and is managed by fluid restriction. ABP Content Specifications(s)/Content Area Understand the complications of rapid correction of hyponatremia Plan treatment for a patient with hyponatremia Suggested Readings Androgue HJ, Madias NE. Hyponatremia. N Engl J Med. 2000;342(21):1581-1589. doi: 10.1056/NEJM200005253422107 Farrell C, Del Rio M. Hyponatremia. Pediatr Rev. 2007;28(11);426-428. doi: 10.1542/pir.28-11-426 Greenbaum LA. Sodium. In: Kliegman RM, Behrman RE, Jenson HB, Stanton BF, eds. C Nelson Textbook of Pediatrics. 20th ed. Philadelphia, PA: Elsevier; 2015:272-279. Zieg J. Pathophysiology of hyponatremia in children. Front Pediatr. 2017; 5: 213. doi: 10.3389/fped.2017.00213

A 3-month-old girl born at 34 weeks gestation is admitted to the pediatric intensive care unit with acute respiratory failure after being found limp at home by the mother. The paramedics who intubated the patient in the field also had to provide 5 minutes of cardiopulmonary resuscitation before return of spontaneous circulation. The mother was at work and had left the baby with her boyfriend. The boyfriend reported that he had just fed the baby before rocking her to sleep in her crib. The heart rate is 155 beats/min, blood pressure is 80/40 mm Hg, and respiratory rate is 22 breaths/min. The patient is afebrile and being ventilated using a tidal volume of 6 mL/kg, positive end-expiratory pressure 7 cm H2O, set rate of 20, and fractional inspired oxygenation at 75%. The patient's oxygen saturation is 100%. The physical examination of the heart, lungs, and abdomen are normal. A focused neurological examination reveals that the anterior fontanel is soft, the pupils are equally round and reactive to light, but there is decreased muscle tone. There are a few round to irregular shaped macular bluish lesions near the sacrum. There are no intra-oral lesions or tears. Laboratory studies such as complete blood count, comprehensive metabolic panel, and the coagulation profile are pending. Chest radiograph shows that the endotracheal tube is appropriately placed, with normal heart and lungs. Of the following, the BEST test to assist in this infant's diagnosis is: A. computed tomography of the brain B. fundoscopic examination C. magnetic resonance imaging of the brain D. skeletal survey

A. computed tomography of the brain Approximately 700,000 to 750,000 children are abused in the United States every year. While the majority of these are cases of general neglect, ~ 100,000 are due to physical abuse, and ~ 50,000 are due to sexual abuse. Most states have mandatory reporting of child abuse and neglect based on suspicion of abuse by the physician. Additionally, the pediatric intensivist may be subpoenaed to be an expert witness in cases of nonaccidental trauma. It is therefore imperative for physicians to have the skills to identify child abuse and differentiate abusive from unintentional injuries correctly. Most academic hospitals and large trauma centers have multidisciplinary teams made of social workers, child advocacy specialists, etc. who assist with the diagnosis and reporting of child abuse. The initial step in the diagnosis of child abuse is detailed history. Family members may have to be interviewed separately. Physicians should be aware of risk factors for child and parent-related risk factors for abuse, which are shown in Table 1 and Table 2. The physical examination seeks to differentiate nonaccidental trauma from accidental injuries and other normal developmental findings in children. Cutaneous manifestations such as rashes from infections and birthmarks such as Mongolian spots have to be differentiated from bruises or inflicted burns. Table 3 shows the differential diagnosis of symptoms of child abuse. Physical abuse takes 4 main forms in children: 1) abusive head trauma; 2) abdominal trauma; 3) fractures; and 4) cutaneous and oral findings. Abusive head trauma was previously called "shaken baby syndrome." A high index of suspicion is needed to make a diagnosis as findings in infants such as seizures, vomiting, decreased feeding, poor weight gain, apnea, and limpness may be blamed on other diagnoses. Physical examination should include a thorough examination of the scalp for swelling and bruising on the neck, face, or in the axilla. Any step offs in the skull (depressed skull fracture), or fullness of the fontanel in a child who does not have meningitis should alert the clinician to nonaccidental trauma. For the infant in the vignette, a computed tomography (CT) scan should be the first step to evaluate for fractures, subdural hematomas, and other intracranial pathology. Evaluation using a CT scan is fast, although there is exposure to ionizing radiation. Magnetic resonance imaging takes a longer time to acquire images, requires sedation (although the infant in this vignette is already intubated), and may not be possible in an unstable infant. Most centers will, therefore, perform an initial CT scan of the brain, and also the abdomen if there is also suspicion of abdominal injury. The newer 3-dimensional reconstruction images obtained from the CT scans can help delineate fractures from suture lines. Clinicians must be aware of an autosomal recessive disorder type 1 glutaric aciduria that can give rise to acute subdural hematomas even from minor trauma and may be mistaken for nonaccidental trauma. The subdural hematomas from glutaric aciduria are usually accompanied by other findings such as bilateral frontotemporal cerebral atrophy or even macrocephaly. The disease doesn't give rise to fractures of bones or retinal hemorrhages, and if a subdural hematoma is present along with other bone fractures or signs of abuse, then the diagnosis of glutaric aciduria is less likely. Ophthalmological evaluation for retinal hemorrhages (Figure) requires chemical dilatation of the pupils, which can compromise the ability to perform serial neurological examinations using pupillary reflex to light. Furthermore, conditions such as seizures, meningitis, leukemia, and hypertension can give rise to retinal hemorrhages. Only 85% of patients with abusive head trauma had retinal hemorrhages. Therefore, initial fundoscopic examination may not be helpful in assisting the evaluation of child abuse in the infant described in the vignette. The hemorrhagic retinopathy commonly seen with abusive head trauma tends to be multilayered and many times extends to the ora serrata. The infant in the vignette should also have a skeletal survey later to evaluate for posterior rib fractures (from thoracic compression) as well as to search for metaphyseal bucket handle fractures of the legs (due to jerking of the legs by the infant during the shaking event). A skeletal survey is not the first step in the diagnosis of child abuse in a comatose patient, but should be done later when the patient is more stable. Furthermore, the skeletal survey can also be done postmortem. A validated tool to estimate the probability of child abuse in a patient with head trauma are shown in Table 4. The presence of 3 or more features in a child less than 3 years of age with head trauma predicted child abuse with a specificity of more than 85%. Abdominal trauma, which can be isolated from abusive head injury usually presents as injuries to liver, spleen, pancreas, duodenal hematoma, and less commonly other parts of the intestines. Besides initial hemorrhagic shock from bleeding, a child can present with only laboratory abnormalities such as transaminitis, elevation of lipase, abnormal or coagulation profiles (from disseminated intravascular coagulation). Other tests such as complete blood counts (to evaluate hemoglobin and platelets), comprehensive metabolic panels to follow liver function tests and electrolyte issues, type and screen (for potential blood transfusion needs), as well as urine analysis for evaluation of hematuria may be needed. Appropriate imaging may be required to evaluate the abdominal injury further. Cutaneous findings such as bruising can be difficult to assess if it is from accidental injuries or abusive trauma. The adage "if you don't cruise, you rarely bruise" is helpful to remember in small infants who are not mobile compared to a toddler. Most accidental bruising is usually over the bony prominences. Any lesion on the face, neck, or inside the mouth, such as torn frenulum, should be highly suspicious for nonaccidental trauma. The infant in the vignette most likely has Mongolian spots on the back, which should be present at birth and usually disappear by 1 year of age. Fractures of skull and long bones posterior ribs especially in small infants should be evaluated closely for child abuse. Posterior rib fractures may not be picked up on chest radiographs, but can be diagnosed using the bone scan or repeating chest radiographs in 2 to 3 weeks to detect the callus formation. Metaphyseal fractures (bucket handle injuries of the long bones) are highly specific for child abuse. Consultation with a pediatric radiologist may be helpful. Spiral fractures of the tibia may be accidental in toddlers (ie, toddler fracture). Clinicians must be aware of bone disorders such as rickets, osteogenesis imperfecta, and Menkes Syndrome, which can predispose to fractures from minor accidental trauma. Appropriate laboratory evaluation such as vitamin D levels and genetic tests may be required for diagnosis. The role of the pediatric intensivist, besides managing the airway, maintaining hemodynamics, and cerebral protection, must also include accurate documentation of all injuries including using photographs to record bruising, limb deformities, and other injuries. Furthermore, protection of other children at home from harm must be also considered. PREP Pearls The term shaken baby syndrome is now replaced by the term abusive head trauma. Posterior rib fractures may not be seen on initial radiographs. Repeat radiographs in 2 to 3 weeks or bone scan may be required. Retinal hemorrhages are present in only 85% of children with abusive head trauma. Furthermore, retinal hemorrhages may be seen with seizures, leukemia, meningitis, and hypertension. ABP Content Specifications(s)/Content Area Recognize the important physical signs and symptoms diagnostic of child abuse Evaluate a suspected victim of child abuse Recognize the laboratory findings associated with child abuse Suggested Readings Asnes AG, Leventhal JM. Managing child abuse: general principles. Pediatr Rev. 2010;31(2):47-55. doi: 10.1542/pir.31-2-47 Berkowitz CD. Physical abuse of children. N Engl J Med. 2017;376(17):1659-1666. doi: 10.1056/NEJMcp1701446 Kellogg ND; American Academy of Pediatrics Committee on Child Abuse and Neglect. Evaluation of suspected child physical abuse. Pediatrics. 2007;119(6):1232-1241. doi: 10.1542/peds.2007-0883

A 15-year-old adolescent boy is in fulminant liver failure after an intentional overdose of acetaminophen. He is being evaluated for liver transplant. His initial laboratory findings are shown below. He has had significant nausea and vomiting refractory to antiemetics. On morning rounds, he begins vomiting copious amounts of bright red blood. Rapid transfusion of packed red blood cells is initiated immediately followed by transfusion of fresh frozen plasma. However, he continues to have large-volume hematemesis. Repeat laboratory studies are shown: Laboratory Test Initial Result Repeat Result INR 2.5 1.8 Fibrinogen, mg/dL (µmol/L) 110 (3.2) 50 (1.5) Hemoglobin, g/dL (g/L) 14 (140) 9 (90) Platelet count, /µL (/L) 250 × 103 (250 × 109) 190 × 103 (190 × 109) A gastroenterologist is consulted and is preparing for upper endoscopy. Of the following, the MOST appropriate blood product transfusion to correct his coagulopathy is A. cryoprecipitate B. fresh frozen plasma C. packed red blood cells D. platelets

A. cryoprecipitate The patient in this vignette most likely has Mallory-Weiss syndrome, which is defined by mucosal tears in the esophagus and proximal stomach due to repeated, forceful vomiting. He also has a significant coagulopathy caused by acute liver failure and diminished production of coagulation factors. Massive hemorrhage requires replacement of red blood cells (RBCs), coagulation factors, and platelets. Packed RBCs are depleted of coagulation factors and platelets, so these must be transfused separately. Fresh frozen plasma contains normal concentrations of all coagulation factors with the exception of factor V (65%) and factor VIII (40%) and is indicated for replacement of coagulation factors in acquired multifactor deficiency (ie, disseminated intravascular coagulation, massive hemorrhage, warfarin reversal, liver failure) and inherited single-factor deficiencies for which there are no single-factor replacement products. Unfortunately, single-factor assays, with the exception of fibrinogen levels, are not readily available in the acute setting. In patients who are bleeding without a history of single-factor deficiency, the prothrombin time (PT)/INR, the activated partial thromboplastin time (aPTT), and the fibrinogen level are most the most commonly used assays to assess coagulation status. Prolongations in the PT/INR and/or aPTT alone suggest the possibility of a multifactor deficiency for which fresh frozen plasma transfusion is warranted. When prolongations in PT/INR and aPTT are associated with decreased levels of fibrinogen, the coagulopathy may be due primarily to the inadequate levels of fibrinogen, the terminal factor in the clotting process. In this scenario, replacement of fibrinogen by administration of cryoprecipitate is indicated. Cryoprecipitate is made by removing the precipitate from thawed fresh frozen plasma that contains higher concentrations of factor VIII, factor XIII, von WIllebrand factor, and fibrinogen. Cryoprecipitate is indicated mainly for the replacement of fibrinogen due to its highly concentrated nature. One bag of cryoprecipitate contains the same amount of fibrinogen found in a unit of fresh frozen plasma in one-tenth the volume (10-20 mL vs approximately 250 mL). These small-volume, concentrated units allow for transfusion of more than one unit without volume overload. Factor VIII, factor XIII, and von WIllebrand factor are all available in single-factor replacement formulations, therefore, cryoprecipitate should only be used for these deficiencies when the other formulations are not available. The patient in this vignette has continued bleeding despite prior factor replacement with fresh frozen plasma. His most recent laboratory results indicate severe hypofibrinogenemia for which transfusion with cryoprecipitate is indicated. When fibrinogen is the only deficient factor, replacement with fresh frozen plasma is unlikely to provide sufficient levels of fibrinogen to correct the severe deficiency seen in this patient. If other factors are deficient then the PT/INR and aPTT will remain prolonged despite replacement of fibrinogen with cryoprecipitate. In this scenario, repeat administration of fresh frozen plasma is warranted. If the patient continues to bleed he is likely to need both platelet and packed RBC supplementation. However, both the hemoglobin and platelet levels are sufficient at this time; so, fibrinogen replacement should take priority in order to help achieve hemostasis, followed by diagnostic and therapeutic upper endoscopy. PREP Pearls Massive hemorrhage requires replacement of red blood cells, platelets, and plasma. Fresh frozen plasma is indicated for replacement of multifactor deficiencies. Cryoprecipitate contains concentrated levels of fibrinogen (up to 10 times that found in fresh frozen plasma) and should be used preferentially for isolated fibrinogen deficiency. ABP Content Specifications(s)/Content Area Know the indications for plasma and cryoprecipitate transfusion Suggested Readings Karafin MS, Hillyer CD, Shaz BH. Transfusion of plasma and plasma derivatives: plasma, cryoprecipitate, albumin, and immunoglobulins. In: Hoffman R, Benz EJ Jr, Silberstein LE, et al, eds. Hematology Basic Principles and Practice. 7th ed. Philadelphia, PA: Elsevier; 2018:1744-1758. Squires RH Jr. Acute liver failure in children. Semin Liver Dis. 2008;28(2):153-166. doi:10.1055/s-2008-1073115. Sundaram SS, Alonso EM, Narkewicz MR, Zhang S, Squires RH; Pediatric Acute Liver Failure Group. Characterization and outcomes of young infants with acute liver failure. J Pediatr. 2011;159(5):813-818.e1. doi:10.1016/j.jpeds.2011.04.016.

During rounds in the pediatric intensive care unit, the pediatric resident presents the case of a 10-year-old boy with acute respiratory failure following an appendectomy 4 days ago. The patient had an aspiration event immediately following surgery requiring reintubation and admission to the pediatric intensive care unit. The patient weighs 30 kg, and his trachea is orally intubated with a 6.0 cuffed endotracheal tube. Current vitals: heart rate 125 beats/min, respiratory rate 18 breaths/min, blood pressure 110/65 (55) mm Hg, and an oxygen saturation of 92% on 0.8 fraction of inspired oxygen (FiO2). The physical examination reveals an intubated, sedated, and chemically paralyzed patient. The patient's respiratory examination reveals symmetric chest wall expansion, and bilateral air entry with rhonchi. The patient is on a pressure regulated volume control mode of ventilation with a positive end-expiratory pressure of 6 cmH2O, a set tidal volume of 300 mL (10 mL/kg), a rate of 15, inspiratory time of 0.7 seconds, and an FiO2 of 0.8. The patient's chest radiograph is shown in Figure 1. An arterial blood gas reveals a pH of 7.28, PCO2 of 40 mm Hg, PaO2 62 mm Hg. The first year pediatric intensive care unit fellow is concerned about the patient's respiratory status, especially given that the peak pressures are in the mid-thirties and the mean airway pressure is 22. You show the pediatric intensive care unit fellow the patient's pressure-volume loop on the ventilator (Figure 2). Of the following, the next BEST ventilator change to protect the lungs from ventilator-induced lung injury is: A. decrease the set tidal volume to 6 mL/kg B. do a recruitment maneuver by increasing positive end-expiratory pressure to 40 C. increase the fraction of inspired oxygen to 1.0 D. increase the set ventilator rate to 25 per minute

A. decrease the set tidal volume to 6 mL/kg The patient in the vignette is on mechanical ventilation following an aspiration event. The oxygenation index (OI) is given by the formula: [(mean airway pressure X fractional inspired oxygen)/PaO2] * 100 [(22 * 0.8) / 62] * 100 = 28. An OI ≥ 16 is one of the criteria used by the Pediatric Acute Lung Injury Consensus Conference (PALICC) to define severe pediatric acute respiratory distress syndrome (ARDS). Recently there has been progress in our understanding of the diagnosis and management of pediatric ARDS. Due to lack of randomized control trials in pediatrics, lung protective ventilation strategies borrowed from adult clinical trials (ARDSnet) are utilized in pediatric ARDS. Studies have shown that ventilation of ARDS patients with high tidal volume/low positive end-expiratory pressure (PEEP) can cause ventilator-induced lung injury, a collective term used for barotrauma (alveolar injury due to due to pressure), volutrauma (alveolar injury due to overdistention), atelectrauma (mechanical injury from cyclical opening and closing of the alveoli of the nonaerated lung during mechanical ventilation), and as well as biotrauma (spreading of inflammatory changes seen in the lungs during mechanical ventilation). Studies have shown that ventilator-induced lung injury can worsen mortality in ARDS. Careful attention needs to be paid to the mode of ventilation, tidal volume per predicted body weight, the level of fraction of inspired oxygenation (FiO2), peak pressures, and the plateau pressure to decrease ventilator-induced lung injury. In this patient, the pressure-volume curve has the classic beaking pattern (Figure 2 ) suggesting alveolar overdistension and risk for volutrauma. The immediate ventilator change indicated, therefore, is a reduction in the tidal volume to 6 mL/kg. Although not given as a response choice, another logical change to augment oxygenation would be to increase the PEEP given the high FiO2 of 0.8. Increasing FiO2 may increase PaO2 in the short run but that will not change the potential for further injury to the lungs. Animal studies have shown that prolonged exposure to high FiO2 for an extended period can cause free radical damage and lung injury. Therefore, increasing FiO2 is not the next best ventilator change in this patient. In fact, the PALICC recommendation in severe pediatric ARDS is to maintain SpO2 92%, already achieved by the patient in the vignette. The patient's PaCO2 of 40 mm Hg is in the normal range, and therefore, an increase in the ventilator rate is not recommended. In fact, a higher PaCO2 should be tolerated (permissive hypercapnia) as long as the pH is 7.15-7.30 and the patient has no pulmonary hypertension or intracranial hypertension. Various recruitment maneuvers such as a staircase recruitment maneuver that includes sustained gradual increases in the PEEP to 30-40 cm H2O have been described in adult studies. At this time the PALICC guidelines do not recommend recruitment maneuvers in pediatric ARDS due to lack of pediatric studies. The term that is collectively used for lung protective ventilation is called as open lung approach. The open lung approach is comprised of: 1) Low tidal volume 2) Applied PEEP 3) Low plateau pressure 4) Recruitment maneuvers There are no pediatric randomized trials of the above four components of the open lung approach. Current PALICC recommendation based on consensus guidelines is to use a tidal volume of 3-6 mL/kg in severe pediatric ARDS. The use of PEEP allows for alveolar recruitment, optimizes functional residual capacity and also prevents cyclical alveolar opening and closing thereby decreasing atelectrauma. The PALICC recommendation is to use a PEEP of 10-15 cm H2O in mild to moderate pediatric ARDS and PEEP of ≥ 15 in severe pediatric ARDS. Careful attention must be paid to the plateau pressure as PEEP is increased. Current PALICC recommendation is to use a plateau pressure of less than 28 cm H2O but 29-32 cm H2O if lung compliance is decreased. Currently, PALICC doesn't recommend any recruitment maneuvers other than smaller incremental changes in the PEEP. If despite the best efforts, there is an increase in mean airway pressures or failure to oxygenate using the conventional ventilation modes, the high-frequency oscillatory ventilator can be used.. The use of high-frequency oscillatory ventilator requires sedation and paralysis and can cause hemodynamic compromise necessitating the need fluid or pressors. Another interesting modality that provides an open lung approach is the use of the airway pressure release ventilation. The airway pressure release ventilation is a mode of ventilation that uses an inverse ratio, pressure controlled, intermittent mandatory ventilation with unrestricted spontaneous breathing. The use of airway pressure release ventilation prevents complete alveolar collapse at the end of expiration and reduces atelectrauma. An advantage of airway pressure release ventilation is the need for less sedation and no neuromuscular blocking agent use, as well as less hemodynamic compromise. PREP Pearls Open lung approach in mechanical ventilation of patients with acute respiratory distress is a lung protective strategy involving low tidal volume, applied positive end-expiratory pressure, low plateau pressure, and recruitment maneuvers. The applied positive end-expiratory pressure allows for alveolar recruitment and protects the lungs by decreasing cyclical atelectasis. The low tidal volume protects the lungs against volutrauma by minimizing overdistention of the lung. ABP Content Specifications(s)/Content Area Understand the principles of open lung strategy for ventilation in patients with acute lung injury Suggested Readings Cheifetz IM. Pediatric ARDS. Respir Care. 2017;62(6):718-731. doi 10.4187/respcare.05591 Heidemann SM, Nair A, Bulut Y, Sapru A. Pathophysiology and management of acute respiratory distress syndrome in children. Pediatr Clin North Am. 2017;64(5):1017-1037. doi: 10.1016/j.pcl.2017.06.004 Rimensberger PC, Cheifetz IM, Kneyber MCJ. The top ten unknowns in paediatric mechanical ventilation. Intensive Care Med. 2018;44(3):366-370. doi: 10.1007/s00134-017-4847-4

A 17-year-old adolescent girl who was previously healthy is admitted to the pediatric intensive care unit after developing shortness of breath, nausea, and a 10 kg weight gain over the last 4 days. She reports having a mild upper respiratory infection a few weeks ago. Physical examination reveals both third and fourth heart sounds, coarse rales, and pulsus alternans. A chest radiograph reveals massive cardiomegaly, and an echocardiogram is significant for global hypokinesia. Intravenous furosemide and a milrinone infusion fail to produce significant improvement, and over several hours she develops worsening tachycardia, an oxygen requirement, and progressive dyspnea. You plan for intubation and mechanical ventilation. Of the following, the effect of positive pressure ventilation that is MOST likely to be beneficial in this scenario is: A. decreased left ventricular afterload B. decreased left ventricular preload C. increased right ventricular preload D. increased right ventricular contractility

A. decreased left ventricular afterload The transition from negative pressure (usually spontaneous) breathing to positive pressure (mechanical) breathing is associated with a number of hemodynamic changes. In the setting of acute dilated cardiomyopathy, decreased left ventricular (LV) afterload can significantly improve cardiac output. Positive airway pressure is transmitted to the intrathoracic contents, including the aorta and the LV. Increased intrathoracic pressure while holding LV systolic pressure constant results in a decreased pressure differential, or left ventricular transmural pressure. Transmural pressure is closely coupled to LV afterload. Therefore, positive pressure breathing results in decreased LV afterload and improved cardiac output. This can be life-saving in the acute dilated cardiomyopathy seen in viral myocarditis. The relationship between airway pressure and pulmonary vascular resistance is complex. At very low lung volumes, hypoxic vasoconstriction dominates, and pulmonary vascular resistance is high. When the lung is inflated but not overdistended and the pulmonary vessels are neither constricted nor impinged upon by nearby airway structures, pulmonary vascular resistance is at its lowest. This is classically described as happening at functional residual capacity. At higher lung volumes and airway pressures, overdistention of airway structures begins to impinge on pulmonary vessels and pulmonary vascular resistance increases again. While improving lung recruitment and eliminating hypoxia can decrease right ventricular (RV) afterload and improve LV preload, it is not clear in this situation that this would predominate over decreased capillary cross-sectional area. Right ventricular preload is closely coupled to systemic venous return. This, being a passive process, is heavily affected by mean intrathoracic pressure. In spontaneous, negative pressure breathing, intrathoracic pressure is generally low throughout the breathing cycle and there is little impedance to systemic venous return. By definition, intrathoracic pressure increases in positive pressure breathing. This inevitably acts to limit venous return and therefore RV preload. In contrast to the LV, RV coronary perfusion happens during systole. In positive pressure breathing, RV pressure increases while, as noted above, aortic pressure generally remains constant. This results in a decreased gradient between the aortic root and the RV, a decrease in RV coronary perfusion pressure, and decreased oxygen delivery to the RV. PREP Pearls Positive pressure breathing reduces left ventricular afterload, which can be helpful when cardiac output is impaired. Positive pressure breathing increases right ventricular afterload while decreasing right ventricular preload and oxygen delivery. These factors can be problematic in the setting of critical heart disease. ABP Content Specifications(s)/Content Area Understand the physiologic effects of positive-pressure breathing Suggested Readings Cheifitz I, Lynn D M, Meliones J, Wetzel R. Respiratory support for the child with critical heart disease. In: Nichols G, Ungerleider R, Spevak P, et al, eds. Critical Heart Disease in Infants and Children. 2nd ed. Philadelphia, PA: Mosby Elsevier; 2006:316-318. Gomez H, Pinsky R. Effect of mechanical ventilation on heart-lung interactions. In: Tobin M, ed. Principles and Practice of Mechanical Ventilation. 3rd ed. New York, NY. McGraw-Hill; 2013: 825-833.

A 10-year-old girl is rescued from a house fire that originated from a malfunctioning space heater. She is brought to your hospital's emergency department prior to being transferred to a burn center. She is being given oxygen via a nonrebreather mask, and has two 22G peripheral intravenous catheters in place. Fluid resuscitation has been started based on a calculation of burns covering approximately 28% of her body surface area, including full thickness facial burns (5%), and deep partial burns over her arms (10%) and torso (13%). The patient is awake, but appears confused and dazed. Her initial vital signs include a rectal temperature of 38.2℃, heart rate 128 beats/min, respiratory rate 24 breaths/min, blood pressure 118/74 mm Hg, O2 saturation 99%. She vomited en route to the hospital, but has no cough, stridor, or wheezing on your examination. Of the following, the NEXT best step is: A. endotracheal intubation B. nasogastric tube placement C. placement of a central venous catheter D. placement of a laryngeal mask airway

A. endotracheal intubation The patient in the vignette has sustained facial burns that places her at increased risk of upper airway edema. It is critical to secure and maintain the airway as a priority in such patients since upper airway edema can progress rapidly, rendering orotracheal intubation difficult or impossible. Initial fluid resuscitation can also exacerbate laryngeal edema over a period of just a few hours. If available, a fiberoptic laryngoscopy may be performed to clarify the extent of airway injury and edema, and the need for intubation. If fiberoptic endoscopy is not readily available or other indications that the patient may have sustained a severe inhalation injury are present, endotracheal intubation should be performed prior to transfer to a burn center. Significant inhalational injury may manifest as cough, stridor, hoarseness, or wheezing but may be initially asymptomatic with only subtle clues indicating the risk of developing airway edema. These clues include the presence of facial burns, singed nasal hair, or carbonaceous matter in the mouth, nose, or sputum. Injury after inhaling hot gases can manifest as oropharyngeal edema, blisters or airway edema above the vocal cords. However, respiratory distress, hypoxia, and hypercarbia may manifest only after further progression of airway injury. Chest radiographs may be normal until secondary complications of inflammation, infection, or atelectasis develop. Significant airway injury may also be associated with elevated carbon monoxide and/or cyanide levels. Patients with soot in the oral cavity, facial burns, and/or severe body burns are at a higher likelihood of laryngeal edema and need early intubation. Fiberoptic laryngoscopic findings of edema of either the true or false vocal cords are also considered as indications for intubation. A recent study from 2018 recommends that "Intubation should be considered for patients with full thickness facial burns, stridor, respiratory distress, swelling on laryngoscopy, upper airway trauma, altered mentation, hypoxia/hypercarbia, hemodynamic instability, suspected smoke inhalation, and singed facial hair. Patients lacking these criteria should not be intubated." PREP Pearls Respiratory distress, hypercarbia, and hypoxia may manifest late after airway burns. Early intubation should be considered for patients with full thickness facial burns. ABP Content Specifications(s)/Content Area Know that facial or airway burns may require early intubation prior to swelling Suggested Readings Badulak JH, Schurr M, Sauaia A, Ivashchenko A, Peltz E. Defining the criteria for intubation of the patient with thermal burns. Burns. 2018;44(3):531-538. doi: 10.1016/j.burns.2018.02.016. Bittner E, Shank E, Woodson L, Martyn JA. Acute and perioperative care of the burn-injured patient. Anesthesiology. 2015;122(2):448-464. doi: 10.1097/ALN.0000000000000559. Madnani DD, Steele NP, de Vries E. Factors that predict the need for intubation in patients with smoke inhalation injury. Ear Nose Throat J. 2006;85(4):278-280. PMID:16696366.

A 17-year-old girl is admitted to the pediatric intensive care unit (PICU) because of fever and altered mental status. The patient was well until 3 weeks ago when she began experiencing intermittent mild headaches accompanied by nausea. Four days before admission, her headaches were persistent and worsening in severity. During this period, the patient was unusually sleepy and complained of light sensitivity and neck pain with movement. On the day of admission, the patient had a 5-minute generalized tonic-clonic seizure for which she was rushed to the emergency department. The patient's medical history was remarkable for a diagnosis of pulmonary sarcoidosis 6 months ago for which she received high-dose corticosteroids initially, followed by a tapering dosage. Due to difficulty in weaning steroids, methotrexate was added to the steroid taper. She has remained asymptomatic since but continues taking methotrexate and corticosteroids. On initial examination, her vital signs are normal except for a temperature of 39°C. She is sleepy but responds to verbal stimuli. She has photophobia, but her pupils are equal, round, and reactive to light. She has rigidity of the neck. In addition, her sensations are intact, she has no cerebellar signs, and she has normal tone and intact reflexes. An initial computed tomography scan performed before her transfer to the PICU showed no mass lesions or hemorrhage. Results of the basic metabolic panel and a complete blood cell count are unremarkable. In the PICU, a lumbar puncture is performed. Cerebrospinal fluid is obtained, with an opening pressure of 45 cm H2O. Cerebrospinal fluid findings are shown below and in the Figure. Cerebrospinal Fluid (CSF) Test (Reference)ResultComment CSF white blood cell count (normal < 5/µL [0.005 × 109/L])54/µL (0.05 × 109/L) Differential count: neutrophils <15% (0.15), lymphocytes 40% (0.4), monocytes 40% (0.4)CSF protein (normal CSF protein, 15-45 mg/dL [1.5-4.5 g/dL])60 mg/dL (0.6 g/L)CSF glucose (normal CSF glucose in two-thirds the range of serum)26 mg/dL (1.4 mmol/L) Patient's serum glucose = 100 mg/dL (5.6 mmol/L) Of the following, the MOST accurate statement regarding the organism detected in the patient's cerebrospinal fluid is A. the cerebrospinal glucose level, protein level, and cells can be normal in immunocompromised patients who have fungal meningitis B. echinocandins have excellent cerebrospinal penetration and are the drugs of choice to treat the meningitis in this patient C. frontal lobe involvement with development of acute hydrocephalus is characteristic of this organism D. the organism detected in the patient's cerebrospinal fluid is acquired by feco-oral transmission

A. the cerebrospinal glucose level, protein level, and cells can be normal in immunocompromised patients who have fungal meningitis The 17-year-old girl described in the vignette who presents with a headache, new-onset seizure, altered mental status, and fever should be presumed to have meningitis unless proven otherwise. Her prolonged corticosteroid and methotrexate use increases her risk for such infections because of immunosuppression. From the patient's cerebrospinal fluid (CSF) findings, it is clear that she has fungal meningitis caused by Cryptococcus neoformans (elevated protein level, low glucose level, predominantly lymphocytic pleocytosis, and detection of budding yeast with characteristic finding on India ink stain). Other fungi that may cause infection of the central nervous system (CNS) include Candida albicans (spread from the blood to the brain, especially in premature infants) and Aspergillus species (spread from the lungs or direct extension from the sinuses and most commonly seen in patients with hematopoietic stem cell transplant). Other less common fungal causes of meningitis include Histoplasma capsulatum or agents causing mucormycosis, including Mucorales, Fusarium, Scedosporium (seen in neutropenic patients), and Coccidioides immitis (seen rarely in patients receiving tumor necrosis factor [TNF] α antagonists). A high index of suspicion for C neoformans infection is required because only 50% of patients have decreased glucose levels with lymphocytic pleocytosis. In patients with acquired immunodeficiency syndrome (AIDS), the CSF findings can be normal, especially CSF glucose, protein, and cells because the patient is unable to mount an immune response. However, patients with AIDS have a higher burden of the fungus, and it is easier to isolate on direct microscopic examination or in cultures. Any patient with suspected C neoformans infection should have cultures of the urine, blood, and bronchoalveolar lavage fluid (if possible, such as in an intubated patient), because detection of C neoformans in these fluids provides non-culture-based supporting evidence of infection with C neoformans. The enzyme immunoassay to detect cryptococcal antigen is positive in more than 90% of patients, especially with CNS disease. A baseline CSF cryptococcal antigen test should be performed before the start of therapy, and levels can be used subsequently to assess the effectiveness of antifungal drug therapy. Repeated lumbar punctures to monitor the antigen levels are unnecessary unless the clinical response is suboptimal. In patients without AIDS, high-volume CSF may be required for growth of C neoformans in cultures, especially if suspicion for C neoformans is high, as in the patient described in the vignette. Central nervous system aspergillosis must be suspected in any immunocompromised host, especially if there is involvement of the sinuses or lungs. The detection of galactomannan antigen and 1,3- β-D-glucan in CSF or serum specimens of patients, although not specific for Aspergillus species and can be seen with other molds, is supportive of a diagnosis of CNS infection with Aspergillus species under the right circumstances. Cryptococcus is a pervasive yeast and is usually acquired by inhalation and not by the feco-oral route. Most immunocompetent patients with C neoformans infections are asymptomatic or have a very mild respiratory tract infection that is usually self-limited. The cellular immunity is vital for elimination of the organism, but the latent asymptomatic form can persist and can get activated with a decrease in cellular immunity. In patients with immune suppression, the fungus can disseminate from the lungs to other sites such as the CNS. Risk factors for fungal meningitis include immunosuppressive states such as human immunodeficiency virus, lymphoma, sarcoidosis, lymphoproliferative diseases, prolonged use of corticosteroids, methotrexate, newer anti-TNF-inhibiting agents such infliximab, and other immunosuppressive agents used in bone marrow or solid organ transplantation. The polysaccharide capsule of C neoformans obstructs CSF absorption by the arachnoid villi in the basilar (but not frontal) areas of the brain, resulting in an acute hydrocephalus and high opening pressures during lumbar puncture. Treatment with antifungal agents must be started as soon as fungal meningitis is suspected based on the patient's history, physical findings, or CSF findings, even if cultures are negative. Without treatment, fungal meningitis is fatal. Usually, a combination of intravenous amphotericin B and flucytosine for the first 2 weeks, followed by 8 weeks of fluconazole therapy, is used in patients with C neoformans meningitis. Treatment is continued until cultures are negative. Amphotericin B is nephrotoxic, hepatotoxic, and neurotoxic and can cause anemia and renal wasting of potassium. Lipid formulations, although costly, can be used to decrease the toxicity of amphotericin B. Voriconazole and the echinocandins such as caspofungin have no activity against C neoformans and are not recommended. In addition, the penetration of the blood-brain barrier by echinocandins is poor compared with that of amphotericin B. The CNS involvement with Aspergillus species is usually fatal, even though therapy can be given with voriconazole (as a first-line agent), or with liposomal amphotericin B as an alternative, if the patient cannot tolerate voriconazole. Candidal meningitis is usually treated with fluconazole, especially for C albicans and Candida parapsilosis. Candida glabrata and Candida krusei have shown resistance to fluconazole but are susceptible to agents such as voriconazole and micafungin. PREP Pearls Cryptococcus neoformans meningitis should be highly suspected in any immunocompromised patient, especially when presentation of the meningitis is indolent with asymptomatic periods. Amphotericin B is the drug of choice in the treatment of cryptococcal meningitis. Antigens to C neoformans are present in the cerebrospinal fluid of more than 90% of patients with untreated cryptococcal meningitis. ABP Content Specifications(s)/Content Area Recognize the clinical signs and symptoms and laboratory findings characteristic of fungal central nervous system infections Plan the treatment for a patient with a fungal central nervous system infection Know the factors associated with an increased risk of fungal central nervous system infection Suggested Readings Long SS, Pickering LK, Prober CG. Principles and Practice of Pediatric Infectious Disease. St Louis, MO: Saunders; 2017. McCarthy M, Rosengart A, Schuetz AN, Kontoyiannis DP, Walsh TJ. Mold infections of the central nervous system. N Engl J Med. 2014;371(2):150-160. doi:10.1056/NEJMra1216008 Segal HB. Aspergillosis. N Engl J Med. 2009;360(18):1870-1884. doi:10.1056/NEJMra0808853

A 12-month-old, previously healthy boy was admitted to the pediatric intensive care unit with acute respiratory failure due to pneumonia requiring invasive mechanical ventilation. Tracheal aspirates that were submitted for analysis 48 hours ago are negative for bacterial pathogens. Reverse transcriptase-polymerase chain reaction (PCR) tests are negative for Mycoplasma and Chlamydia. You suspect a viral cause for the underlying disease in this child. You order viral studies, and the results are pending. In your discussion with the resident physician regarding the pathophysiology of this child's disease, the MOST appropriate statement is A. the earliest pathologic feature of the underlying disease in this child is damage to the cilia B. if the chest radiograph shows a large pleural effusion, the pneumonia is most likely due to parainfluenza virus C. intense inflammation in the lungs with neutrophilic infiltration into the alveoli will be observed if the underlying disease is caused by the influenza virus D. the reverse transcription-polymerase chain reaction test that was positive for respiratory syncytial virus 1 week ago will become negative within 1 week of hospitalization

A. the earliest pathologic feature of the underlying disease in this child is damage to the cilia Community-acquired pneumonia is responsible for substantial morbidity and mortality in the pediatric population and accounts for up to 20% of all deaths in children younger than 5 years worldwide. Nearly all cases of community-acquired pneumonia are caused by viruses, with respiratory syncytial virus (RSV), parainfluenza, and influenza being the dominant etiologic agents. Other viruses that commonly cause community-acquired pneumonia in children include rhinovirus, adenovirus, human metapneumovirus, enterovirus, and coronavirus. Human metapneumovirus can cause a clinical picture similar to RSV bronchiolitis and pneumonia. Viruses that cause pneumonia mostly in immunocompromised hosts include herpes simplex virus types 1 and 2, varicella, measles, cytomegalovirus, and Epstein-Barr virus. Viruses that have caused epidemics of lower respiratory tract infection with pneumonia include influenza, Hantavirus, various serotypes of coronavirus (severe acute respiratory syndrome), and enterovirus D68. There appears to be an increased incidence of viral pneumonia in children over the past several years. This increase may reflect greater awareness about the diagnosis and the availability of new viral diagnostic tools, particularly reverse transcription-polymerase chain reaction (RT-PCR), which can be used to detect viruses in respiratory secretions. The advent of this new diagnostic modality and its widespread availability in clinical practice may also lead to changes in the epidemiology of viral pneumonia in children as newer data become available. However, it is important to recognize that in the United States, mortality due to pneumonia from all causes has declined by 97% since the 1930s. This decrease is believed to be related to better access to health care and the availability of effective vaccines and antibiotics. Based on the currently available data, in the pediatric population the most common cause of lower respiratory tract infection with pneumonia is RSV, followed by parainfluenza and influenza viral infections. Whereas RSV is transmitted via contact with animate or inanimate objects, the other 2 viruses are transmitted via respiratory droplets. Clinical manifestations of viral pneumonia include coryza and cough with progression to acute respiratory distress with tachypnea, nasal flaring, retractions, and increased work of breathing that may progress to acute respiratory failure with hypoxia, necessitating the need for ventilatory support, as is the case in the vignette. It may be difficult to differentiate between acute bronchiolitis and pneumonia caused by RSV in infants and young children because both may present with acute respiratory distress and hypoxia. It is also often difficult to differentiate between influenza and RSV pneumonia in children on clinical grounds alone. However, RSV is more often associated with rhinorrhea, production of sputum, and wheezing and is less likely to be associated with fever and gastrointestinal symptoms compared with influenza. Nearly all cases of pneumonia caused by parainfluenza virus infection are caused by type 3. Parainfluenza virus can cause giant cell pneumonia, particularly in immunocompromised patients in whom the mortality rate approaches 100%. The presence of upper respiratory tract infection before the onset of pneumonia, the presence of wheezing on physical examination, and the documentation of sinusitis (in older children) on radiographic evaluation suggest parainfluenza virus as the viral etiologic agent for pneumonia, rather than cytomegalovirus or Epstein-Barr virus in an immunocompromised child. Furthermore, as much as 30% of children with parainfluenza virus pneumonia can have secondary bacterial pneumonia. The pathophysiology of viral pneumonia is complex and has not been fully elucidated because of limited access to tissue samples. It does involve the interaction between the multiple arms of the immune system. After arrival into the upper respiratory tract, the virus begins to multiply in the epithelium of the upper respiratory tract, where it produces symptoms including nasal congestion and nasal drainage. The precise mechanism by which the virus invades the lower respiratory tract is not clear; however, it is believed that the virus gains access into the lower respiratory tract via the respiratory epithelium, via hematogenous dissemination, or both. Most viruses that cause pneumonia have surface fusion proteins that allow virus-cell fusion and cell-cell fusion.The influenza and parainfluenza viruses have a hemagglutinin protein that facilitates virus attachment to ciliated epithelial cells. The RSV has a glycosylated protein (RSV G protein) that facilitates attachment of the RSV to the respiratory epithelial cells. Destruction of the cilia appears to be a critical and early step in the pathogenesis of viral pneumonia. This destruction induces impaired mucociliary clearance, leading to pooling of secretions that facilitates progression of the virus into the lower respiratory tract by microaspiration or via cell-to-cell spread. Subsequent histopathological changes depend on the type of the virus causing the pneumonia. Some viruses have direct cytopathic effects and destroy the respiratory cells, whereas others cause intense inflammatory response that is responsible for the tissue damage. Influenza tends to have direct cytopathic effects on cells and causes cell death and destruction with minimal inflammatory response, although there may be hemorrhage seen at the sites of infection. Hemorrhagic pleural effusion can also been seen with influenzal pneumonia. On the other hand, RSV infection is more likely to induce inflammatory response that is characterized by substantial interstitial infiltration by mononuclear cells. Infiltration of lymphocytes into the peribronchial walls is also seen. The lung parenchyma thus becomes edematous, with foci of necrosis as well as alveolar flooding with inflammatory cells and inflammatory debris with consolidation interspersed with areas of atelectasis. Areas of compensatory hyperinflation can also be seen. This inflammatory response appears to be mediated via a number of cytokines that are released in response to the viral infection. Respiratory syncytial virus tends to induce release of several cytokines, including interleukins 6 and 8, that appears to be regulated by nuclear factor kB. Both influenza and RSV infections damage the cilia of the columnar epithelial cells in the respiratory tract. This damage impairs the mucociliary clearance that can pave the pathway for bacteria to invade the lower respiratory tract and cause secondary bacterial pneumonia. This process may explain the phenomenon that during the influenza season up to 50% of cases of bacterial pneumonia have associated influenza infection when appropriate laboratory tests are performed. Influenza A virus can infect livestock, which can remain a reservoir for the infection and subsequently lead to epidemics of influenza A infection. Influenza B infection is usually seen in the setting of overcrowding, such as military camps and college dormitories. The diagnosis of viral pneumonia is based on the clinical features discussed earlier. Bacterial pneumonia requiring hospitalization tends to produce higher fever (as high as 40°C) than in viral pneumonia. Radiographic evaluation may also be helpful to differentiate between viral and bacterial pneumonia. A lobar pneumonia involving a specific lobe of the lungs, a well-rounded pneumonia, or the presence of large pleural effusion or empyema are more likely with bacterial pneumonia than viral pneumonia. Currently, RT-PCR can be used to identify the specific viral pathogen. However, viral particles, including viral RNA, may persist in the respiratory secretions for weeks after an acute infection, even when the virus can no longer can be cultured. This phenomenon is important to recognize in children who require multiple admissions to the hospital for respiratory tract infections. The natural history of viral pneumonia in most children is slow resolution, and the prognosis for most previously healthy children is good. Complications such as pleural effusion, empyema, and lung abscess formation are rare with viral pneumonias. Most cases of viral pneumonia resolve without sequelae. However, when viral pneumonia occurs in neonates and young infants, it can be complicated by pulmonary hypertension, which can rarely be associated with pulmonary hemorrhage because of the pulmonary vascular damage. Furthermore, neonates and young infants who require high ventilatory support, high fraction of inspired oxygen, or both are at risk of chronic lung disease developing later in life. Pulmonary interstitial emphysema and other forms of air leak syndrome can also develop in infants with viral pneumonia who require ventilatory support. Some cases of severe adenovirus pneumonia in immunocompromised children, particularly due to serotypes 2, 3, 7, and 21, may progress to bronchiolitis obliterans and may also cause unilateral hyperlucent lung syndrome. Bronchiolitis obliterans has been reported in association with parainfluenza virus, influenza, RSV, and measles infections with pneumonia; however, it is most commonly associated with adenovirus infection (odds ratio of 49). Histopathological analysis usually shows that the bronchioles are surrounding by an inflammatory process with concentric narrowing of the lumen. The long-term effect of this process is permanent narrowing or complete obliteration of the small airways, with substantial morbidity and mortality. Other uncommon life-threatening conditions that may be associated with viral pneumonia include myocarditis, encephalitis, and multiple organ dysfunction syndrome. PREP Pearls The mortality rate from pneumonia in children in the United States has declined by 97% since the 1930s. Secondary bacterial pneumonia occurs in up to 50% of cases of influenza pneumonia. After a respiratory tract infection, viral particles such as viral RNA may persist in the respiratory secretions for weeks after the infection, even when the virus cannot be cultured from the respiratory tract. Adenovirus infection is most commonly associated with the development of bronchiolitis obliterans. ABP Content Specifications(s)/Content Area Understand the pathophysiology and pathogenesis of viral pneumonia Know the clinical course of viral pneumonia Recognize the life-threatening complications of viral pneumonia Suggested Readings Byington CL, Bradley JS. Pediatric community-acquired pneumonia. In: Cherry J, Demmler-Harrison GJ, Kaplan SL, Steinbach WJ, Hotez P, eds. Feigin and Cherry's Textbook of Pediatric Infectious Disease. 7th ed. Philadelphia, PA: Elsevier Saunders; 2014:283-306. Colom Al, Teper AM, Vollmer WM, Diette GB. Risk factors for the development of bronchiolitis obliterans in children with bronchiolitis. Thorax. 2006;61(6):503-506. doi:10.1136/thx.2005.044909. Crowe JE. Viral pneumonia. In: Wilmot RW, Boat TF, Bush A, Chernick V, Deterding RR, Ratjen F, eds. Kendig and Chernick's Disorders of the Respiratory Tract in Children. 8th ed. Philadelphia, PA: Elsevier Saunders; 2012:453-460. Fischer GB, Sarria EE, Mattiello R, Mocelin HT, Castro-Rodriguez JA. Post infectious bronchiolitis obliterans in children. Paediatr Resp Rev. 2010;11(4):233-239. doi:10.1016/j.prrv.2010.07.005.

An 18-month-old boy admitted for septic shock underwent percutaneous placement of a 4 French 8 cm double lumen central venous catheter at the right internal jugular vein site after multiple attempts. Maintenance IV fluids and an epinephrine drip have been running through this line for about 2 hours. He becomes progressively tachycardic and then has a sudden deterioration in his blood pressure to 40/20 mm Hg. He has decreased peripheral pulses and perfusion. His oxygen saturation also drops suddenly to 65%. An emergent chest radiograph is obtained showing pulmonary edema and a large cardiac silhouette. Bedside sonography is shown the (pericardial fluid) . While preparing for a definitive intervention, the BEST next step to improve the blood pressure in this patient is: A. administer furosemide through a peripheral IV line B. administer normal saline bolus through a peripheral IV line C. increase the positive end-expiratory pressure on the ventilator from 7 cm H2O to 10 cm H2O D. start a continuous infusion of milrinone

B. administer normal saline bolus through a peripheral IV line The scenario describes a patient with pericardial tamponade resulting from fluid administration through a central venous catheter that was malpositioned or subsequently eroded into the pericardial space. Rapid accumulation of fluid in the relatively nonelastic pericardial space causes external pressure on the heart, thereby compromising atrial filling and cardiac output. Administration of 20 mL/kg fluid peripherally will provide increased preload, thereby improving atrial filling. An increase in the central venous pressure raises the transmural filling pressure and offsets external pressure from the pericardial fluid that is causing right atrial and ventricular collapse. Once atrial filling is restored, cardiac output and blood pressure improve, at least transiently until the pericardial fluid can be drained as the definitive therapy. Cardiac tamponade is a medical emergency resulting from the accumulation of pericardial fluid, blood or air, leading to impaired cardiac filling due to external compression of the the heart. This reduction in preload precipitously drops the systemic blood pressure. Definitive treatment includes evacuation of the fluid, blood, or air either through a percutaneous tube or surgically. Pericardial tamponade can be seen in the first 24-48 hours following cardiac surgery or if fluids are infused through a malpositioned central venous catheter. Tamponade physiology also results from other processes that create large increases in intrathoracic pressure and thus reduce venous return, such as tension pneumothorax. Clinical features include tachycardia, decreased peripheral perfusion, distension of neck veins, rising measured central venous pressure, and pulsus paradoxus (exaggerated inspiratory decrease in systolic blood pressure). Point-of-care ultrasound or echocardiography can be used to diagnose pericardial tamponade quickly and provide definitive therapy (Video). Early detection improves survival from pericardial tamponade caused by central venous catheters. Tamponade physiology is based on the relationship between the right atrial and external pressures. Transmural pressure (Ptm) is the difference between the right atrial (PRA) and external pressure (Pexternal), which could be intrathoracic or pericardial pressure. Ptm= PRA - Pexternal For example, in a normal patient with a central venous pressure/right atrial pressure of 5 cm H2O, and a negligible external pressure during negative pressure breathing, the transmural filling pressure is also 5 cm H2O. However, with accumulation of pericardial fluid, as in the patient in the vignette, the transmural pressure decreases. For example, if the pericardial pressure rises to 15 cm of H2O, and the right atrial pressure remains at 10 cm H2O, the transmural pressure would effectively become minus 5 cm H2O, which would lead to the diastolic collapse of the right atrial wall described in the vignette. Administering a fluid bolus would increase PRA and restore transmural filling pressure, thereby providing forward flow and improve the cardiac output. Administration of furosemide would be contraindicated in this patient, as it would decrease the preload and right atrial pressure further, worsening tamponade physiology. Similarly, though increasing the positive end-expiratory pressure (PEEP) can decrease left ventricular afterload and improve cardiac output in patients with heart failure, with pericardial tamponade the primary issue is inadequate filling. In this case, increasing PEEP would be counterproductive because it would raise intrathoracic pressure and lower venous return, thereby reducing the right atrial and transmural filling pressure further. Milrinone is a phosphodiesterase inhibitor that improves cardiac contractility and lowers systemic vascular resistance. In this case, milrinone is not useful as cardiac contractility is not the primary issue in tamponade physiology. Once transmural filling pressure is restored by either increased preload or evacuation of pericardial fluid, cardiac output improves quickly. PREP Pearls External compression by pericardial fluid, blood, or air reduces atrial transmural pressure and cardiac output resulting in pericardial tamponade, which is a medical emergency. Fluid administration increases right atrial filling and can improve the systemic blood pressure until pericardial drainage is performed. Bedside sonography can aid in rapid diagnosis and treatment of tamponade physiology. ABP Content Specifications(s)/Content Area Understand the pathophysiology of tamponade Suggested Readings Hoit BD. Pathophysiology of the pericardium. Prog Cardiovasc Dis. 2017;59(4):341-348. doi: 10.1016/j.pcad.2016.11.001. Kayashima K. Factors affecting survival in pediatric cardiac tamponade caused by central venous catheters. J Anesth. 2015 Dec;29(6):944-952. doi: 10.1007/s00540-015-2045-5. Smith AT, Watnick C, Ferre RM. Cardiac tamponade diagnosed by point-of-care ultrasound. Pediatr Emerg Care. 2017;33(2):132-134.

A 14-year-old, previously healthy female patient is transferred to the pediatric intensive care unit with bacterial sepsis. On arrival, she is lethargic, tachycardic, and hypotensive and is in severe respiratory distress. She requires immediate intubation and mechanical ventilation as well as fluid resuscitation. After initial support measures, her respiratory status improves, but she remains tachycardic and hypotensive with widened pulse pressure. Her random cortisol level is 2.3 μg/dL (63.5 nmol/L), with minimal response to a cosyntropin stimulation test. She is placed on a regimen of norepinephrine for treatment of fluid-refractory shock and IV hydrocortisone for treatment of adrenal insufficiency. Over the next 2 days, the patient demonstrates some improvement despite need for ongoing support. However, you note that for the past 24 hours the patient's glucose levels have been persistently above 200 mg/dL (11.1 mmol/L). Of the following, hyperglycemia in this patient is MOST likely caused by A. cortisol-induced glycogen synthesis B. cytokine-mediated insulin resistance C. downregulation of growth hormone secretion D. stress-induced inhibition of glucagon release

B. cytokine-mediated insulin resistance The patient in the vignette has stress-induced hyperglycemia due to dysregulation of glucose homeostasis. This dysregulation is common during critical illness and stems from a complex interaction of stress-related events, including changes in hormone production, surges in catecholamines and proinflammatory mediators, and disruption of end-target signaling and response. Cytokine-mediated insulin resistance describes one of the multiple mechanisms responsible for this dysregulation and resultant hyperglycemia during severe illness. Under normal circumstances, glucose homeostasis is tightly regulated by neurohormonal control, and insulin and glucagon play the most important roles in this balance. Increased blood glucose levels trigger insulin secretion, which promotes the uptake and utilization of glucose, amino acids, and fatty acids in target-sensitive tissues. The overall effect on glucose homeostasis is a reduction of circulating glucose levels. In contrast, glucagon opposes insulin by promoting glucose release in response to low circulating glucose levels. The target effects of these opposing hormones are summarized in the Table. During the initial phase of critical illness, hyperglycemia develops through a combination of factors that ultimately elevates circulating blood glucose levels to provide a ready fuel for increased metabolic demands. Surges in glucagon, catecholamines, cortisol, and growth hormone (GH) lead to hyperglycemia via upregulation of glycogenolysis and gluconeogenesis. Despite normal levels of insulin, these counterregulatory hormones limit glucose uptake and provide a continued stimulus for glucose production and release into the circulation. After the acute phase of illness, glycogen stores are quickly depleted. However, catecholamines and other stress hormones continue to directly stimulate glucagon secretion. The presence of elevated glucagon levels leads to persistent hepatic and renal gluconeogenesis, whereas other counterregulatory hormones such as GH facilitate muscle breakdown to provide continued substrate for glucose synthesis. Furthermore, although insulin levels may be normal or even elevated, changes in insulin sensitivity perpetuate hyperglycemia during stress. During prolonged critical illness, counterregulatory hormones and inflammatory cytokines alter insulin signaling in liver, muscle, and adipose tissue and lead to peripheral resistance. Inflammatory mediators have also been shown to directly inhibit insulin secretion in some clinical settings. Although the mechanisms of glucose dysregulation in critical illness are complex and incompletely understood, hyperglycemia is well recognized and frequent in critically ill adults and children, and is associated with poor outcomes. As such, many expert providers have explored the safety, efficacy, and effectiveness of treating hyperglycemia in affected patients by limiting glucose intake or delivering IV insulin therapy, or both. Several early trials in adults found that treatment of hyperglycemia with insulin did indeed lead to improved outcomes in certain populations. However, the ideal therapeutic goals of this treatment are difficult to establish, and many replicated studies have not demonstrated similar results. Furthermore, while many health care providers recognize the high frequency of hyperglycemia in children and its association with increased morbidity and mortality, the effects of glycemic control with insulin in this population have not been thoroughly researched. A recent 35-center trial to evaluate strict glycemic control in children was terminated early because of the low likelihood of benefit and the potential harm from this therapy. The results of this landmark trial were published in the New England Journal of Medicine in 2017, and this study serves as a reminder that further research to evaluate glycemic control in this population is necessary before definitive treatment recommendations can be made. The patient described in the vignette has septic shock with multisystem organ involvement and is at high risk for stress-induced glucose dysregulation with resultant hyperglycemia. Cytokine-mediated insulin resistance is one of several contributing factors. Hydrocortisone therapy may play a role, but cortisol induces glycogenolysis, not glycogen synthesis. Growth hormone and glucagon are increased during the acute phase of critical illness and potentiate hyperglycemia by promoting glycogenolysis and gluconeogenesis; downregulation or inhibition of either would lead to decreased glucose levels, not hyperglycemia. PREP Pearls The endocrine response to stress is well recognized and includes severe derangements in glucose homeostasis. Alterations in insulin and glucagon production and end-target sensitivity are common during critical illness and often culminate in hyperglycemia. Hyperglycemia is common and associated with increased morbidity and mortality in critically ill children. However, the effectiveness of insulin treatment in this population has not been established, and further studies are warranted. ABP Content Specifications(s)/Content Area Know the alterations in insulin and glucagon secretion caused by critical illness Suggested Readings Agus M, Faustino E, Rigby MR. Hyperglycemia, dysglycemia, and glycemic control in pediatric critical care. In: Wheeler DS, Wong HR, Shanley TP, eds. Pediatric Critical Care Medicine. 2nd ed. New York, NY: Springer; 2014:93-102. Gastroenterological, Endocrine, Renal, Hematologic, Oncologic, and Immune Systems.; vol 3. Agus MS, Wypij D, Hirshberg EL, et al, for the HALF-PINT Study Investigators and the PALISI Network. Tight glycemic control in critically ill children. N Engl J Med. 2017;376(2):729-741. doi:10.1056/NEJMoa1612348. Ulate KP, Zimmerman JJ. Common endocrinopathies in the pediatric intensive care unit. In: Fuhrman BP, Zimmerman JJ, eds. Pediatric Critical Care. 4th ed. Philadelphia, PA: Elsevier; 2011:1112-1116.

A 15- year-old male patient is admitted to the pediatric intensive care unit after a motor vehicle crash, wherein he sustained injuries primarily about the lower part of the thorax and upper aspect of the abdomen. His physical findings reveal an alert young man complaining of back pain primarily at vertebral levels C7 and T1. His vital signs on admission reveal a heart rate of 60 beats/min, blood pressure of 80/40 mm Hg, respiratory rate of 18 breaths/min, and pulse oximetry of 96% on room air. His lungs are clear to auscultation, there are easily heard heart tones with no murmurs appreciated, and peripheral pulses are strong with well-perfused extremities. There is bruising noted across the anterior aspect of the chest and upper abdominal aspect consistent with a seat belt, and the patient has some tenderness on palpation of the anterior chest wall bilaterally as well as the right upper quadrant. The patient has no deformities of his extremities; however, he cannot move his lower extremities and has absent deep tendon reflexes in both legs. He received 60 mL/kg of crystalloid fluids in the emergency department because of an initial blood pressure of 70/30 mm Hg. Of the following, the cause of his hypotension is BEST described by A. cardiogenic shock due to myocardial ischemia B. distributive shock due to vasoplegia C. hypovolemic shock due to hemorrhage from splenic laceration D. obstructive shock due to cardiac tamponade

B. distributive shock due to vasoplegia The patient described in the vignette has shock associated with spinal cord injury, a prototypical example of distributive shock. Shock occurs when oxygen and nutrient delivery are inadequate to meet the metabolic demands of the tissues. Prompt recognition of shock is crucial to successful management and improved outcomes regardless of cause. Having a high index of suspicion in at-risk patients as well as knowledge of signs and symptoms of shock in children will facilitate recognition. Shock states can be classified into functional categories such as hypovolemic, cardiogenic, obstructive, distributive, and septic shock; however, patients may display features of multiple categories of shock at the time of diagnosis or during treatment. Regardless of cause, the progression of symptoms in the untreated child with shock occurs similarly, moving from compensated shock to uncompensated state and culminating in irreversible shock. Initially, compensatory mechanisms involving autonomic, cardiopulmonary, renal, and immunologic systems are activated to maintain vital organ function. Patients may display confusion, irritability, tachypnea, tachycardia, mild decreased capillary refill, and oliguria with preserved blood pressure. As compensatory mechanisms begin to fail and cellular function deteriorates, oxygen delivery becomes wholly inadequate and cellular death is observed. At this juncture, patients are lethargic or comatose, hypotensive, and anuric and display metabolic anomalies such as hypoglycemia, metabolic acidosis, and elevated serum lactate levels. Irreversible shock occurs when cellular death has transpired to a degree that regardless of restoration of cardiopulmonary performance to age-normative values, key organs have been irreversibly damaged and death is inevitable. These patients lack peripheral pulses and are comatose, apneic, hypotensive, anuric, and profoundly acidemic, with laboratory manifestations of end-organ damage including hepatic, renal, and bone marrow failure. Initial assessment of the patient includes targeted history and physical examination, provision of oxygen, and prompt vascular access. Laboratory profile includes complete blood cell count, basic electrolytes, hepatic function, coagulation profile, and arterial blood gas with ionized calcium. Targeted imaging studies are obtained as clinically indicated. Distributive shock is characterized by a maldistribution of blood flow to the tissues as a consequence of decreased systemic vascular resistance or by the inability at the tissue cellular level to use delivered oxygen. Maldistribution of blood flow clinically mimics hypovolemia and is generally a consequence of abnormal vasomotor tone. Despite normal or even elevated cardiac output, this maldistribution of blood flow results in regional inadequacies in tissue oxygen delivery. Distributive shock occurs in anaphylaxis, spinal anesthesia, medication exposure (nitrates), and spinal cord injuries. Anaphylaxis may occur in response to envenomation; exposure to allergens such as latex, food, or medication; or administration of blood products. The ensuing mast cell degranulation results in histamine release with subsequent vasodilation. Pertinent physical findings include urticarial rash, local edema, and low systemic vascular resistance. Histamine antagonists and corticosteroids to blunt the histamine response in addition to fluid resuscitation and vasoactive agents to improve vasomotor tone are the mainstays of treatment. Spinal cord injury involving the cervical cord above vertebral level T1 is associated with disruption of the sympathetic chain, allowing for unopposed parasympathetic tone. Normal peripheral vascular tone is impaired, leading to arterial and venous vasodilation. This leads to relative hypovolemia as vasodilation results in increased venous capacitance and decreased preload. However, lack of sympathetic tone hampers the compensatory reflex tachycardia. These patients commonly present with hypotension, relative bradycardia, and neurologic findings of spinal cord injury, notably absent deep tendon reflexes, loss of sensation, and motor deficits. Fluid resuscitation is indicated initially; however, these patients commonly require vasoactive infusions to manage vasoplegia, as well as avoidance of medication known to decrease systemic vascular resistance and avoidance of infections and fluid losses. Therapy for shock focuses on improving oxygen delivery and minimizing the metabolic demands of the body or oxygen consumption. Rapid delivery of crystalloid solution to achieve age-normal values of heart rate and blood pressure as well as restoring perfusion and urine output should be accomplished within the first hour. If this is insufficient, one must consider use of a vasoactive agent. The primary defect in distributive shock is vasoplegia; therefore, selecting a vasoactive agent that can restore vascular tone is recommended (Table). The available agents are phenylephrine, norepinephrine, and vasopressin. Both phenylephrine and norepinephrine exert their vasoconstrictive effects by their interaction with the α-receptors found on blood vessels. However, vasopressin acts on the V1 vasopressin receptors located on blood vessels throughout the body. All these vasoactive agents are administered as infusions and are promptly titratable to clinical effect. These infusions are titrated to achieve normal-for-age blood pressure values as well as to restore perfusion and improve end-organ function such as urine output. Minimizing metabolic demands may include fever control, provision of analgesia, anxiolysis, and sedation as well as more aggressive modalities such as intubation and mechanical ventilation. PREP Pearls Shock is the sequela of mismatched tissue oxygen delivery and metabolic demands. Untreated shock progresses from compensated shock to uncompensated shock to irreversible shock. This progression is characterized by progressive cellular death. Distributive shock is characterized by maldistribution of blood flow to tissues and by abnormal decrease in vasomotor tone. ABP Content Specifications(s)/Content Area Know how to investigate causes of distributive shock Understand the pathophysiology of distributive shock Know the prognosis associated with distributive shock Suggested Readings Davis AL, Carcillo JA, Aneja RK, et al. American College of Critical Care Medicine clinical practice parameters for hemodynamic support of pediatric and neonatal septic shock. Crit Care Med. 2017;45(6):1061-1093. doi:10.1097/CCM.0000000000002425 Smith LS, Badugu S, Hernan LJ. Shock states. In: Fuhrman and Zimmerman's Pediatric Critical Care. 5th ed. Philadelphia, PA: Elsevier; 2017:chap 36. Yager P, Noviski N. Shock. Pediatr Rev. 2010;31(8):311-319. doi:10.1542/pir.31-8-311

A 17-year-old adolescent girl is admitted to the pediatric ICU with lesions on her lips, buccal mucosa, chest, and arms (Figure). Her parents report that she has no significant past medical or surgical history. She has no known drug allergies and has been taking trimethoprim-sulfamethoxazole for the past 3 weeks for facial acne. A Gram stain of fluid from one of the bullae shows no organisms and few cells. Of the following, the substance MOST likely to be elevated in the bullae of this patient is A. C5a B. granulysin C. histamine D. tryptase

B. granulysin The patient in this vignette has toxic epidermal necrolysis (TEN), a variant of Stevens-Johnson syndrome (SJS). These are life-threatening dermatologic disorders frequently associated with concomitant drug use, particularly antibiotics. Toxic epidermal necrolysis results in rapidly progressing blistering lesions with widespread epidermal and mucosal sloughing. Stevens-Johnson syndrome is diagnosed when less than 10% of body surface area is affected, whereas TEN requires more than 30% involvement. In severe cases, TEN is associated with renal failure, liver failure, corneal injury, and septic shock. A 3-year epidemiologic study in the United States sponsored by the Agency for Healthcare Research and Quality showed a TEN frequency of 0.7/1,000,000 pediatric hospital admissions with a mortality rate of 16%. Antibiotics and antiepileptics are the most commonly cited causative factors for SJS/TEN. A multicenter study demonstrated that more than 50% of SJS/TEN patients had prior exposure to antibiotics and 31% had prior exposure to antiepileptics. Adverse drug reactions to antibiotics are reported in up to 10% of hospitalized children and 2% of outpatients. One study demonstrated that systemic antibiotics were most frequently implicated in adverse drug reactions (23%), followed by general anesthesia (15%). Common adverse drug reactions to antibiotics can include hypersensitivity reactions, maculopapular manifestations, and gastrointestinal problems. Less common adverse events include severe skin reactions such as SJS, bone marrow suppression, and liver failure. Hypersensitivity reactions usually appear immediately after antibiotic exposure and are IgE mediated. More severe adverse reactions typically present after patients have been on the antibiotic for days to weeks and are usually T-cell mediated. Among antibiotics, β-lactams are the most frequent cause of adverse drug reactions, accounting for a majority of cases. Amoxicillin alone or in combination with clavulanic acid is the antibiotic most frequently associated with adverse drug reactions. Non-β-lactam antibiotics, such as quinolones, sulphonamides, and macrolides, are reported to cause approximately 20% of adverse drug reactions. Sulphonamides, such as sulfamethoxazole-trimethoprim, most commonly cause a delayed hypersensitivity reaction and are strongly implicated in SJS and drug reaction with eosinophilia and systemic symptoms (DRESS) syndrome. Allergic reactions to vancomycin, aminoglycosides, and tetracycline are rare. The mechanisms responsible for adverse drug reactions to antibiotics are increasingly understood. In 1968, Gell and Coombs developed a classification system for allergic reactions. They identified the following 4 types of drug reactions: Type I, rapid in onset and mediated through IgE Type II, delayed in onset and classically caused by IgG-mediated cell destruction Type III, caused by immunoglobulin-drug complex deposition and subsequent activation of the complement system Type IV, delayed in onset and mediated through T-cell activation Most antibiotic reactions are type I, occurring almost twice as frequently as nonimmediate responses. In a typical type I reaction, antibiotics are initially analyzed by dendritic cells, then bound to endogenous proteins and presented to naive T cells. Activated T cells interact with B cells by 2 pathways (CD40-CD40L and the Th2 cytokines). This results in IgE activation and subsequent production of drug-specific IgE. These antibodies then bind to specific high-affinity Fc receptors on the surface of basophils and mast cells. Future exposure to this antibiotic/protein conjugate results in crosslinking of 2 or more IgE-bound receptors, which triggers release of inflammatory mediators, such as histamine, prostaglandins, leukotrienes, or tryptase. Delayed drug reactions (type IV) are mediated through CD4+ T cells and typically take several days to weeks to manifest. An example of this type of reaction is SJS/TEN. With this type of reaction, activated T cells demonstrate increased expression of cutaneous lymphocyte antigen and increased production of certain cytokines such as interferon, tumor necrosis factor α, granulocyte-macrophage colony-stimulating factor, and interleukin 2. These immune mediators regulate trafficking, proliferation, and activation of cellular components of the immune system. The activated T cells can further activate macrophages (type IVa), eosinophils (type IVb), or neutrophils (type IVd). Research suggests that genetics may play a role in type IV responses. Evidence suggests an association between certain drug classes and specific HLA phenotypes that may result in a higher binding affinity. For example, among those of European descent, HLA alleles A29, B12, and DR7 demonstrate a higher incidence of TEN. In SJS and TEN, activated T or natural killer cells produce cytotoxic proteins that result in diffuse apoptosis among keratinocytes and resultant loss of epithelial integrity. Granulysin is a recently discovered cytolytic protein that appears to play a key role in keratinocyte apoptosis in TEN. In a recent study, granulysin levels in blister fluid from TEN patients were significantly higher than other cytotoxic proteins such as perforin, granzyme B, or FasL. Depletion of granulysin decreased cytotoxicity and lesion progression. PREP Pearls Amoxicillin is the antibiotic most frequently associated with adverse drug reactions. Sulfonamide antibiotics and anticonvulsants are most commonly implicated in Stevens-Johnson syndrome and toxic epidermal necrolysis. The T cell- or natural killer cell-derived cytotoxin granulysin plays a key role in toxic epidermal necrolysis though initiation of keratinocyte apoptosis. ABP Content Specifications(s)/Content Area Know the adverse effects associated with commonly used antibiotics Suggested Readings Andrade PHS, Lobo IMF, da Silva WB. Risk factors for adverse drug reactions in pediatric inpatients: a cohort study. PLoS One. 2017;12(8):e0182327. doi:10.1371/journal.pone.0182327 Chung WH, Hung SI, Yang JY, et al. Granulysin is a key mediator for disseminated keratinocyte death in Stevens- Johnson syndrome and toxic epidermal necrolysis. Nat Med. 2008;14(12):1343-1350. doi:10.1038/nm.1884 Dibek Misirlioglu E, Guvenir H, Bahceci S, et al. Severe cutaneous adverse drug reactions in pediatric patients: a multicenter study. J Allergy Clin Immunol Prac. 2017;5(3):757-763. doi:10.1016/j.jaip.2017.02.013 Eaddy Norton A, Konvinse K, Phillips EJ, Dioun Broyles A. Antibiotic allergy in pediatrics. Pediatrics. 2018;141(5):e20172497. doi:10.1542/peds.2017-2497 Hsu DY, Brieva J, Silverberg NB, Paller AS, Silverberg JI. Pediatric Stevens-Johnson syndrome and toxic epidermal necrolysis in the United States. J Am Acad Dermatol. 2017;76(5):811-817. doi:10.1016/j.jaad.2016.12.024

A pediatric intensive care unit physician receives a call from the emergency department regarding a 14-year-old adolescent with a history of Graves disease who now presents with fever, tachycardia, hypertension, bounding pulses, dyspnea, and diaphoresis. There are no murmurs or gallops on cardiac auscultation. A complete blood cell count, electrolyte analysis, thyrotropin level, and serum thyroid hormone levels are pending. A 12-lead electrocardiogram demonstrates shortened QT interval, ST-segment elevation, and notching of the terminal QRS (J waves) in the inferior leads. Of the following, the MOST likely cause of this patient's electrocardiogram findings is A. high cardiac output syndrome B. hypercalcemia C. hyperthermia D. myocarditis

B. hypercalcemia The patient in the vignette likely has thyroid storm and has electrocardiogram (ECG) findings consistent with hypercalcemia, an infrequent but well-known complication of thyrotoxicosis. ST-segment elevation and QT-interval shortening are early changes that appear in the setting of mild to moderate hypercalcemia. J waves (Figure), also called Osborn waves, are positive deflections at the QRS and ST junction and become prominent when hypercalcemia is severe. While J waves are less common than other ECG changes during hypercalcemia, they are more specific for this electrolyte abnormality. Other causes of J waves include hypothermia, ventricular fibrillation, and brain injury. Hypercalcemia, defined as total calcium greater than 11 mg/dL (2.75 mmol/L) or ionized calcium greater than 5.4 mg/dL (1.35 mmol/L), is uncommon in children, and the physical effects are often subtle and nonspecific. However, severe hypercalcemia is life threatening, and thus this infrequent electrolyte disorder must be recognized and treated promptly in affected patients. The regulation of plasma calcium levels involves a complex interaction between modulatory hormones and multiple organ systems. Primary sources for plasma calcium include bone turnover, intestinal absorption, and renal filtration and reabsorption. Greater than 99% of calcium is stored in bone; only a small percentage is found in plasma. Half of plasma calcium exists in the physiologically active form, and the remainder is protein- or anion-bound. The major regulators of plasma calcium include parathyroid hormone (PTH), 1,25 (OH)2 vitamin D (calcitriol, active vitamin D), and calcitonin. Parathyroid hormone and 1,25 (OH)2 vitamin D target bone, kidneys, and intestines to increase bone turnover, upregulate renal calcium reabsorption, and stimulate intestinal absorption to cumulatively increase plasma calcium levels. High calcium levels in turn provide negative feedback for PTH and 1,25 (OH)2 vitamin D production. Calcitonin provides minor regulation by inhibiting calcium release from bone and decreasing renal reabsorption. Hypercalcemia occurs as a consequence of increased intake, increased bone resorption, increased intestinal absorption, or decreased renal excretion. Examples of conditions associated with increased intake include milk-alkali syndrome, parenteral nutrition, and overzealous oral calcium supplementation. Disorders of increased bone resorption include hyperparathyroidism, certain malignancies, thyrotoxicosis, and immobilization. Sarcoidosis, Williams syndrome, and vitamin D intoxication can lead to increased intestinal absorption. Familial hypocalciuric hypercalcemia and thiazide diuretic use both lead to decreased renal excretion. The causes of hypercalcemia in children vary by age, but the most common disorders include hyperparathyroidism from any cause (ie, parathyroid adenoma, transient neonatal hyperparathyroidism, chronic renal failure), certain malignancies, and conditions associated with prolonged immobilization. Causes of hypercalcemia and associated laboratory findings are summarized by mechanism in the Table. Signs and symptoms in affected patients correlate with the degree and rapidity of hypercalcemia, and they can range from subtle to life-threatening. Nonspecific findings include fatigue, weakness, vomiting, diarrhea or constipation, poor feeding, polyuria, and dehydration. More severe symptoms can include hypertension, confusion, psychosis, seizures, coma, and renal failure. Severe hypercalcemia leads to hyperpolarization across myocardial membranes and can result in life-threatening dysrhythmias. The diagnostic evaluation in patients with known or suspected hypercalcemia depends on the history, comorbid conditions, and clinical symptoms. Laboratory investigations should include total and ionized serum calcium, as well as serum phosphorus levels and renal function tests. Serum PTH, 1,25 (OH)2 vitamin D, and 25-hydroxyvitamin D levels, as well as urine calcium, phosphorus, and creatinine levels, may provide further information regarding cause (Table). Imaging may be indicated if malignancy is suspected. An ECG should be performed in all patients with hypercalcemia, and ECG changes may be the only clinical finding in mild to moderate cases. Common ECG findings in patients with hypercalcemia include shortened QT interval, ST-segment elevation, and T-wave flattening. Severe hypercalcemia is associated with more ominous ECG abnormalities including J waves and ventricular dysrhythmias. The treatment for hypercalcemia depends on severity and underlying cause. The initial goal of therapy is to correct dehydration and increase renal excretion of calcium with isotonic fluids. If the patient is well hydrated and calcium levels remain high, loop diuretics can be administered to enhance renal excretion. Severe hypercalcemia may require adjunct therapies that directly lower calcium, such as bisphosphonates or calcitonin. A total serum level greater than 15 mg/dL (3.75 mmol/L) is a medical emergency and necessitates immediate treatment with direct calcium-reducing agents. Surgery may be indicated in the settings of malignancy or primary hyperparathyroidism. Other treatments for hypercalcemia should be tailored to the underlying condition. The patient in the vignette is at risk for complications associated with thyrotoxicosis, including hypercalcemia, high-output cardiac failure, and hyperpyrexia. However, she has no gallop on cardiac auscultation, thus heart failure is unlikely and would not cause her specific ECG abnormalities. Hypothermia (not hyperthermia) is associated with J waves. Myocarditis would not be expected in this patient. PREP Pearls Hypercalcemia is uncommon in children, but severe hypercalcemia is life-threatening; thus, this electrolyte disorder must be recognized and treated promptly. Physical findings in patients with hypercalcemia are subtle and nonspecific. Electrocardiogram changes may be the only clinical findings and should prompt further evaluation. A serum calcium level greater than 15 mg/dL (3.75 mmol/L) is a medical emergency and necessitates immediate treatment. ABP Content Specifications(s)/Content Area Understand the various causes of hypercalcemia Know the clinical, laboratory, and electrocardiographic manifestations of hypercalcemia Suggested Readings Çullas Ilarslan NE, Siklar Z, Berberoglu M. Childhood sustained hypercalcemia: a diagnostic challenge. J Clin Res Pediatr Endocrinol. 2017;9(4):315-322. doi:10.4274/jcrpe.4247 Lietman SA, Germain-Lee EL, Levine MA. Hypercalcemia in children and adolescents. Curr Opin Pediatr. 2010;22(4):508-514. doi:10.1097/MOP.0b013e32833b7c23 Lynth RE, Wood EG. Fluid and electrolyte issues in pediatric critical illness. In: Fuhrman BP, Zimmerman JJ, Carcillo JA, eds. Pediatric Critical Care. 4th ed. Philadelphia, PA: Saunders Elsevier; 2011:960-962.

A 2-year-old boy is brought the emergency department after being found next to an open container of dishwasher soap pods. The parents report debris consistent with the pod capsule as well as detergent granules on his face and lips at the time of discovery. He is alert, awake, drooling slightly, and crying with a slightly hoarse voice. His lungs are clear to auscultation and he does not have any increased work of breathing. He is tachycardic with a heart rate of 160 beats/min. All other vital signs are normal. Of the following, the BEST plan of care for this child includes A. hemodialysis for the removal of absorbed toxins B. intubation with flexible upper endoscopy in 12 to 48 hours C. nasogastric tube placement followed by gastric lavage and charcoal administration D. no treatment necessary; discharge to home

B. intubation with flexible upper endoscopy in 12 to 48 hours The dishwasher pod ingested by the patient in this vignette is likely to be highly alkaline and falls in the category of a strong base. Strong bases are found in dish and laundry detergent, bleach, oven cleaners, and drain cleaners, as well as disk batteries. They are caustic to oral, esophageal, gastric, and pulmonary tissue and are a common cause of accidental ingestion in small children and intentional ingestion in teenagers and adults. Ingestion of strong bases, especially those with a pH of greater than 11, leads to liquefaction necrosis. Contact with mucosal tissue rapidly leads to fat saponification, protein denaturation, and cell destruction. The products of this reaction lead to liquefaction of the mucosal surface and allow for further injury via absorbance of the caustic substance into even deeper layers. Upper airway and esophageal injury are most common with ingestion of alkaline substances, although gastric injury may also occur. Significant esophageal injury almost always leads to subsequent stricture formation and the associated sequelae. Signs and symptoms associated with ingestion of alkaline substances include coughing, choking, drooling, dysphagia, wheezing, stridor, chest pain, nausea, and vomiting. Although symptoms are likely to occur immediately, delayed symptoms are possible, especially with substances such as dishwasher pods with granules that may take time to dissolve. Absence of initial symptoms in children is not predictive of severity of ingestion, therefore, any ingestion of a caustic substance must be evaluated thoroughly and managed cautiously. In severe cases, esophageal and/or gastric rupture, mediastinitis, acute respiratory distress syndrome, and death have been reported. Patients who are asymptomatic and have ingested a small volume of a substance known to be a weak base (such as household bleach) may be observed for several hours and then discharged home. Patients who are experiencing any symptoms or who are known to have ingested a substance with a very high pH should be managed aggressively. The patient in this vignette is symptomatic and has evidence of ingestion of a strong base, therefore, immediate discharge home is not appropriate. Providers should consider immediate intubation for airway protection in patients exhibiting symptoms suggesting upper airway involvement such as stridor, hoarseness, coughing, and drooling. Patients with high risk of esophageal injury, including those who ingest substances with a pH greater than 11, who ingest large volumes, and/or who exhibit oropharyngeal burns, should undergo flexible upper endoscopy in the first 12 to 48 hours for direct visualization of injury. Endoscopy earlier than 12 hours may not reveal the full injury. After 48 hours in extensive injury, the inflammation and cell death lead to tissue weakness and increased risk of perforation. Flexible endoscopy is preferred over rigid endoscopy to minimize risk of perforation. Fever, vital sign instability, and severe chest or abdominal pain may indicate esophageal or gastric perforation, which requires surgical intervention. The presence of oropharyngeal burns is a predictor for more significant esophageal injury. Injury can be divided into 4 levels based on endoscopic appearance (Table). Patients with injuries classified as grade 2b or greater should have a nasogastric tube placed under direct visualization during endoscopy to ensure accurate placement and avoid perforation. The nasogastric tube serves as a stable feeding route during the following weeks as well as a stent during the time of maximal stricture formation. Stricture formation due to fibroblast infiltration and scar formation is common after alkali ingestion and occurs within the first 6 weeks. Close to 100% of patients with grade 2b or greater injury go on to develop strictures that will need ongoing dilatation. Incidence of stricture formation in less severe injury is variable. Patients with injuries that are grade 2a or greater should have a barium esophagram to assess for strictures 2 to 4 weeks after injury followed by esophageal dilatation of narrowed areas. The timing for dilatation is controversial with some advocating that early intervention (1-4 weeks after ingestion) leads to more successful long-term results. Others maintain that the risk of perforation with early dilatation makes later intervention more desirable. Patients with severe strictures almost always require repeated intervention despite the timing. Gastric lavage, administration of ipecac syrup or charcoal, and dilution with water may lead to vomiting and re-exposure of the esophagus, oropharynx, and respiratory tract to the offending agent and are contraindicated. Ingested bases do not lead to absorption of toxins that would require removal with hemodialysis. Treatment with steroids in the acute period is controversial. Antibiotic therapy is indicated only when perforation is suspected. Chronic strictures may lead to long-term malnutrition, increased risk of malignancy, and significant emotional distress. Early evidence suggests that treatment of strictures refractory to dilatation with mitomycin C, an anti-fibroblast agent, applied topically at the time of dilatation can help decrease recurrence. In the most refractory cases, surgical palliation with esophagectomy and colon interposition may be attempted. Success rates with this procedure remain disappointing. PREP Pearls Ingestion of strong bases (pH > 11) can cause immediate and severe injury to the oropharynx, upper respiratory tract, esophagus, and stomach. Symptom severity after ingestion of alkaline substances does not always correlate with injury severity in pediatric patients. The most common morbidity experienced is esophageal stricture, found in 100% of patients with transmural injury. ABP Content Specifications(s)/Content Area Recognize the clinical and laboratory manifestations of acute alkali ingestion Understand the pathogenesis and toxic effects of ingested alkali Plan the diagnostic assessment and evaluation of a child suspected of alkali ingestion Plan appropriate therapy for a child with acute alkali ingestion Recognize the potential airway compromise associated with caustic ingestion Suggested Readings Betalli P, Fachetti D, Giuliani S, et al; Caustic Ingestion Italian Study Group. Caustic ingestion in children: is endoscopy always indicated? The results of an Italian multicenter observational study. Gastrointest Endosc. 2008;68(3):434-439. doi:10.1016/j.gie.2008.02.016. Schoem SR, Rosbe KW, Bearelly S. Aerodigestive foreign bodies and caustic ingestions. In: Lesperance MM, Flint PW, eds. Cummings Pediatric Otolaryngology. Philadelphia, PA: Elsevier Saunders; 2015:374-384.

An 8-year-old child is admitted to your pediatric intensive care unit for a 3-day history of vomiting and poor oral intake. Past medical history reveals a successful heart transplant was performed at the age of 2 years for a dilated cardiomyopathy. Since that time, this child has been maintained on cyclosporine and prednisone without episodes of acute rejection. The patient developed hypertension and has been treated with metoprolol. Vital signs reveal an oral temperature of 37.1℃, heart rate 118 beats/min, respiratory rate 15 breaths/min, blood pressure 94/72 mm Hg. Room air oxygen saturation is 99%. This child is awake and alert. Mucous membranes are dry. The child has a nonfocal neurologic examination. He has clear breath sounds with good spontaneous respiratory effort. Cardiovascular examination reveals good pulses and perfusion. He has a soft II/VI systolic ejection murmur at the left sternal border. No gallop or rub is appreciated. Abdominal examination is unremarkable. There has been no urine output over the past 7 hours. A fluid bolus had been administered and intravenous fluids with normal saline were started. Laboratory results Na+144 mEq/L K+3.0 mEq/L Cl-109 mEq/L CO2-19 mEq/L BUN 15 mg/dL CrCr 1.2 mg/dL WBC6.3 x 109 per liter (L) Hgb10.2 gram/dL Hct 34.6% Cyclosporine 620 ng/ml (normal 100-400 ng/ml) Of the following responses, which is correct regarding the drug causing nephrotoxicity in this patient? A. it blocks release of renin causing hypotension B. it causes vasoconstriction of the afferent and efferent glomerular arterioles C. it causes renal vein stenosis D. the duration of therapy, not dose is responsible

B. it causes vasoconstriction of the afferent and efferent glomerular arterioles Calcineurin inhibitors revolutionized transplantation with improved short and long-term outcomes following an organ transplant. Calcineurin inhibitors such as cyclosporine and tacrolimus are routinely used as immunosuppressive agents for patients that have received an organ transplant. These agents are also used in a number of different autoimmune diseases including psoriasis, rheumatoid arthritis, and atopic dermatitis. Although cyclosporine and tacrolimus are not structurally related, these agents suppress the immune system by inhibiting the production of interleukin 2 in T cells. Their side effects, specifically nephrotoxicity, are thought to be due to similar mechanisms. Adverse effects of the calcineurin inhibitors include hypertension, nephrotoxicity, hepatotoxicity, tremor, and headache. The nephrotoxicity associated with these agents causes acute azotemia and a dose reduction is needed to prevent further renal injury. This can be especially problematic for patients who have had an organ transplant because rejection can occur from lower dose immunosuppression. Calcineurin inhibitor renal toxicity is the result of a dose dependent reduction in glomerular filtration rate. Nephrotoxicity can be minimized using the lowest dose possible. Dose reduction can reverse the nephrotoxicity encountered with calcineurin inhibitors in some cases. Chronic calcineurin inhibitor nephrotoxicity results in progressive renal failure with irreversible deterioration of renal function, progressive tubulointerstitial injury, and glomerulosclerosis as a result of direct toxic effects of the drug and induced hemodynamic changes. Calcineurin inhibitors have a narrow therapeutic index and are directly cytotoxic to tubular cells at high concentrations. Drug levels should be monitored closely. Increased dose and duration both contribute to the nephrotoxicity of this agent. Nephrotoxicity due to calcineurin inhibitors is multifactorial. These agents activate the renin-angiotensin system by directly affecting the release of renin from juxtaglomerular cells causing vasoconstriction of the afferent and efferent glomerular arterioles resulting in renal toxicity. Calcineurin inhibitor vasoconstriction also occurs because of reduced production of vasodilators, such as prostaglandin and nitric oxide, and increased release of vasoconstrictors, such as endothelin and thromboxane, causing a reduction of renal blood flow and glomerular filtration rate. Hemodynamic changes are a result of arteriolar vasoconstriction secondary to decreased vasodilator factors and increased endothelin resulting in systemic hypertension. Calcineurin inhibitors can result in neurotoxicity, affecting the central and peripheral nervous system. Peripheral tremors and headache are common. Hypertension associated with hemodynamic changes from calcineurin inhibitor use can result in seizures and encephalopathy. Posterior reversible encephalopathy syndrome (PRES) is a reversible condition consisting of headache, altered mental status, seizures, and cortical blindness. The syndrome is associated with bilateral white matter abnormalities seen on the MRI of the brain. In the case scenario described, calcineurin inhibitors cause vasoconstriction of the afferent and efferent glomerular arterioles and activate the renin-angiotensin system affecting release of renin from juxtaglomerular cells. Nephrotoxicity is related to both dose and duration. Hemodynamic effects of the calcineurin inhibitors result in systemic hypertension due to vasoconstriction and decreased production of vasodilators such as prostaglandins and nitric oxide. PREP Pearls Calcineurin inhibitors have a narrow therapeutic index and require close monitoring of drug levels. Nephrotoxicity of calcineurin inhibitors are related to dose and duration of therapy. ABP Content Specifications(s)/Content Area Recognize renal toxicity associated with calcineurin inhibitors Suggested Readings Azzi JR, Sayegh MH, Mallat SG. Calcineurin inhibitors: 40 years later, can't live without. J Immunol. 2013;191(12):5785-5791. doi: 10.4049/jimmunol.1390055 Bechstein WO. Neurotoxicity of calcineurin inhibitors: impact and clinical management. Transpl Int. 2000.13(5):313-326. PMID:11052266 Hoskova L, Malek I, Kopkan L, Kautzner J. Pathophysiologic mechanisms of calcineurin inhibitor-induced nephrotoxicity and arterial hypertension. Physiol Res. 2017;66(2):167-180. PMID: 27982677 Williams D, Haragsim L. Calcineurin nephrotoxicity. Adv Chronic Kidney Dis. 2006;13(1):47-55. doi: 10.1053/j.ackd.2005.11.001

A 7-year-old girl is admitted to the pediatric intensive care unit in late summer for altered mental status. Her parents noted mouth ulcers as well as lesions on her palms and soles of her feet for the previous 2 days. Last night she became febrile, developed a severe headache, and had multiple episodes of emesis. Today, she became increasingly confused and disoriented. Rapid influenza screen is negative. Serum electrolytes and complete blood count are normal. Glasgow Coma Scale score is 11. No papilledema is noted and meningismus is not elicited. Soon after arrival to the pediatric intensive care unit, she decompensates, requiring intubation and initiation of vasopressor agents. Cerebrospinal fluid analysis demonstrates the following results: Red cell count 4 cells/mm3 White cell count 147 cells/mm3 Differential: 87% lymphocytes, 10% neutrophils Glucose 78 mg/dL Protein 110 mg/dL Gram stain No organisms seen Opening pressure 34 cmH2O Of the following, the area MOST likely to be abnormal on imaging of her brain and cervical spine is: A. cerebellum B. medulla oblongata C. parietal cortex D. peri-ventricular white matter

B. medulla oblongata This patient in the vignette has classic clinical and laboratory evidence of encephalitis and elevated intracranial pressure. Intracranial pressure (ICP) is defined as the pressure difference between spinal fluid in the cerebral ventricles and atmospheric pressure. This gradient is largely determined by cerebral blood flow, cerebro spinal fluid (CSF) production and circulation, and alteration of the cerebral architecture. Normal ICP values may be reported in cmH2O or mm Hg. Elevated ICP (intracranial hypertension) is poorly defined in pediatrics, with various studies using values of greater than 15 to 25 mm Hg or > 20 cmH2O. Common causes of intracranial hypertension include pseudotumor cerebri, space-occupying lesions, cerebral vascular accidents, certain pharmaceutical agents, autoimmune disorders, and central nervous system (CNS) infections. The history and CSF findings in this vignette suggest an infectious etiology, specifically enterovirus 71. This is a known cause of hand-foot-mouth disease with a peak incidence during the summer season. In a recent survey from the Agency for Healthcare Research and Quality (AHRQ) utilizing a nationwide inpatient sample dataset, viral encephalitis was responsible for 7.3 per 100,000 hospitalizations in the United States from 2000 to 2010. Of identifiable causes, enteroviral encephalitis was responsible for ~0.5 per 100,000 hospitalizations over this period. Bacterial, fungal and tuberculosis-associated meningitis are all well described causes of elevated ICP. Recently, late stage Lyme disease has also been associated with intracranial hypertension. With improvements in viral detection technology, cases of intracranial hypertension from viral central nervous system infections are increasingly reported. These include measles, hepatitis A and E, Varicella, Epstein-Barr, influenza, West Nile, and enterovirus. In a study from New York, intracranial hypertension was noted in 15% of viral encephalitis patients, with values ranging from 21 to 55 cmH2O. The pathophysiology of elevated ICP in CNS infections is poorly understood and likely varies in acute bacterial meningitis compared to viral encephalitis. In acute bacterial meningitis, the etiology of intracranial hypertension is likely a combination of meningeal inflammation causing increased CSF production, occlusion of arachnoid granulation tissue or cerebral veins, and obstruction of normal CSF circulation from inflammatory debris. Elevated ICP in viral encephalitis is thought to result initially from cytotoxic edema in the base of the brain that diminishes CSF outflow, followed by vasogenic edema later in the clinical course. In Japanese encephalitis, an organism known to infect the midbrain, lumbar puncture opening pressures of >25 cmH2O are noted in a majority of patients and correlated directly with increased mortality and long-term morbidity. There are currently over 70 serotypes of enterovirus. These can infect the meninges (meningitis), the brain (encephalitis), or both (meningoencephalitis). In enteroviral encephalitis, the midbrain, pons, medulla, and hippocampus are often involved. Involvement of the cerebral cortex can also be present, but is much less common. In a study from Taiwan on magnetic resonance imaging findings with enterovirus 71, the authors noted 75% of patients showed lesions in the medulla oblongata, pons, and midbrain. Autopsy evaluations of patients who have died from enterovirus encephalitis classically demonstrate widespread inflammation in the central grey matter of the medulla oblongata and adjacent spinal cord, consistent with the term "rhombencephalitis." Other viruses also show a predilection for the midbrain. Flaviviruses, such as Japanese encephalitis, infect the basal ganglia and thalamus. Inflammation in the brain stem and adjacent areas often involves vital vasomotor and respiratory control centers and may explain the rapid deterioration often noted in these patients. Treatment of elevated ICP in viral encephalitis is largely supportive. Several authors advocate for placement of ICP monitoring devices to assist in management. A recent large study from India compared aggressive control of ICP in CNS infections with use of osmotic agents, increased sedation, or brief hyperventilation to cerebral perfusion pressure (CPP) targeted management and showed that preservation of blood pressure with resultant optimization of CPP had a larger clinical impact, dropping mortality rate to 18%. A Cochrane review in 2013 showed no benefit from use of osmotic therapy in both adult and pediatric patients with acute CNS infections. PREP Pearls Viral encephalitis is an increasingly recognized cause of intracranial hypertension. The pathophysiology of intracranial hypertension in viral encephalitis likely results from infection and inflammation at the base of the brain. Certain viruses such as enterovirus 71 and Japanese encephalitis virus infect the midbrain and have a higher likelihood of causing intracranial hypertension. ABP Content Specifications(s)/Content Area Know that increased intracranial pressure may complicate central nervous system infection Plan the management of increased intracranial pressure in a patient with a central nervous system infection Suggested Readings Beal JC. Increased intracranial pressure in the setting of Enterovirus and other viral meningitides. Neurol Res Int. 2017;2017:2854043. doi: 10.1155/2017/2854043 George BP, Schneider EB, Venkatesan A. Encephalitis hospitalization rates and inpatient mortality in the United States, 2000-2010. PLoS One. 2014;9(9):e104169. doi: 10.1371/journal.pone.0104169 Kumar G, Kalita J, Misra UK. Raised intracranial pressure in acute viral encephalitis. Clin Neurol Neurosurg. 2009;111(5):399-406. doi: 10.1016/j.clineuro.2009.03.004 Kumar R, Singhi S, Singhi P, et al. Randomized controlled trial comparing cerebral perfusion pressure-targeted therapy versus intracranial pressure-targeted therapy for raised intracranial pressure due to acute CNS infections in children. Crit Care Med. 2014;42(8):1775-1787. doi: 10.1097/CCM.0000000000000298 Shen WC, Chiu HH, Chow KC, Tsai CH. MR imaging findings of enteroviral encephalomyelitis: an outbreak in Taiwan. AJNR Am J Neuroradiol. 1999;20(10):1889-1895. http://www.ajnr.org/content/20/10/1889/tab-article-info

A 4-year-old boy who underwent the modified Fontan procedure is admitted to the pediatric intensive care unit. He was weaned from mechanical ventilation and his trachea was extubated earlier today. Anesthesia records do not report vomiting during induction. Following extubation, he developed acute respiratory distress with irregular and erratic chest movements. Vital signs include a regular heart rate of 118 beats/min, blood pressure of 107/65 mm Hg, and SpO2 of 91% on room air. There is no stridor and auscultation does not reveal rales. A chest radiograph is obtained and is shown in the Figure. Of the following, the MOST likely diagnosis is: A. aspiration pneumonia B. phrenic nerve injury C. subglottic edema D. thrombosis and total occlusion of the Fontan's lateral tunnel

B. phrenic nerve injury The diaphragm is innervated by the phrenic nerve that originates in the neck and descends into the chest to supply the diaphragm in a bilateral fashion. The phrenic nerve is a mixed nerve with both sensory and motor functions. The sensory portion of the phrenic nerve supplies the central parts of the diaphragm and the adjacent pleura, peritoneum and the pericardium; while the motor portion of the phrenic nerve supplies the ipsilateral hemidiaphragm. The phrenic nerve originates primarily from the fourth cervical spinal root (C4), but there are also branches contributed from the third (C3) and fifth (C5) cervical roots. After the nerve is formed from the ventral rami of C3, C4, and C5 along with branches from cervical plexus, the nerve descends from the lateral border of the anterior scalene muscle, then over the anterior border of this muscle and subsequently passes through the prevertebral layer of deep fascia. The course of the phrenic nerve into the chest differs on the right compared to the left side. On the right side, the phrenic nerve passes anterior to the brachial plexus, then the subclavian artery, then anterior to the hilum of the right lung, passing over the pericardium on its way to innervate the right hemidiaphragm. On the left side, the phrenic nerve also passes anterior to the brachial plexus, anterior to the subclavian artery, crosses the vagus nerve and the aortic arch, passes anterior to the hilum of the left lung and then over the pericardium on its way to supply the left hemidiaphragm. The phrenic nerve may be injured during delivery of an infant if there is significant traction on or hyperextension of the neck. Later in life the phrenic nerve may be injured following trauma to the neck, during percutaneous insertion of a subclavian central venous catheter and most commonly during repair of congenital cardiac defects. Prior to the widespread availability of cardiac surgical procedures in infants and children, phrenic nerve injury was most commonly seen following a difficult birth of a large baby. The injury to the nerve in this setting occurred as it passes anterior to the brachial plexus; and it was common for these neonates to have associated brachial plexus injuries. Most cases were unilateral and the right side was more commonly injured than the left side. These newborns also sometimes developed acute respiratory insufficiency. Older data shows that some of these newborns improved over a period of several weeks; however, if no further improvement was noted after one month, diaphragmatic plication was indicated and performed. Currently, the most common clinical setting in which phrenic nerve injury and diaphragmatic paralysis occurs is following repair of congenital heart defects, with an incidence ranging from 0.5% to 5%. Phrenic nerve injury is more commonly observed after repair of transposition of the great vessels, the Norwood procedure, complete repair of tetralogy of Fallot, Glenn anastomosis, and the modified Fontan procedure as seen in the case in the vignette. The nerve may be severed during the surgical procedure, or it may be stretched enough to lead to temporary loss of function. Topical packing with ice or ice slush around the heart, used to induce cooling for the cardiac procedure, may damage the phrenic nerve. The clinical manifestations of phrenic nerve injury are variable. In the most common clinical situation following repair of congenital heart disease the infant may be asymptomatic while receiving invasive mechanical ventilation, but may fail attempts at weaning from mechanical ventilation. Following liberation from mechanical ventilation and extubation of the trachea, the patient may exhibit respiratory distress with tachypnea and retractions. With inspiration, the healthy hemidiaphragm moves downward and this creates negative intrathoracic pressure. Concurrently, the paralyzed hemidiaphragm moves upward in response to the negative intrathoracic pressure as it is "sucked into" the thoracic cavity. The abdominal contents move upward in the hemithorax on the side of the paralyzed diaphragm. This "paradoxical' motion does not allow the lung to expand on the side where the hemidiaphragm is paralyzed. This results in respiratory distress, ineffective ventilation, and eventually atelectasis with hypoxemia. A young child's chest is very compliant and therefore it tends to collapse during inspiration in response to the negative intrapleural pressure. Without intervention, the patient will eventually develop ipsilateral atelectasis and respiratory failure requiring mechanical ventilation. The chest radiograph in the vignette shows the characteristic elevation of the hemidiaphragm (Figure). The diaphragm is considered elevated when the right side is higher than the left by greater than 2 intercostal spaces and/or the left is higher than the right hemidiaphragm by greater than 1 intercostal space on the plain chest radiograph. The diagnosis of diaphragmatic paralysis is confirmed by lack of mobility of the affected hemidiaphragm during fluoroscopy or real-time ultrasonography. Prolonged mechanical ventilation has been the predominant approach while awaiting for recovery from phrenic nerve injury. However, plication of the diaphragm has become a more acceptable approach to diaphragmatic paralysis. Plication prevents paradoxical motion of the affected hemidiaphragm improving pulmonary mechanics. The length of mechanical ventilation prior to plication ranges from 1-7 weeks suggesting significant variations among clinicians in their approach to this problem. Patients who undergo plication of the diaphragm are usually liberated from mechanical ventilation and the diaphragmatic elevation on the affected side returns to normal in the vast majority of these patients. Patients who have undergone univentricular repair and have developed diaphragmatic paralysis due to phrenic nerve injury should be given higher consideration for early diaphragmatic plication because prolonged positive pressure has more adverse effects on venous return. Endoscopic minimally invasive techniques are being increasingly utilized for diaphragmatic plication with good results, even in infants, and should be considered in this clinical setting. A new ferromagnetic surgical dissection system, used to minimize adjacent tissue damage during surgery, has been utilized in an attempt to lower the risk of damage to phrenic nerve. In one study the risk of phrenic nerve injury was lower compared to control patients who were undergoing reoperation for congenital cardiac defects. This device may be promising in lowering the risk of phrenic nerve injury in children undergoing cardiothoracic surgeries. Aspiration pneumonia is more likely to be associated with findings on auscultation, usually manifests radiographic abnormalities in the lung parenchyma, and is most likely to involve the superior segment of the right lower lobe. Thrombosis of the lateral tunnel (that is created as part of the modified Fontan procedure) would likely result in circulatory shock and hypotension, findings that are not exhibited by the patient in the vignette. Subglottic edema would produce significant stridor and prolonged inspiratory phase that are not seen in the patient in the vignette. PREP Pearls If the left hemidiaphragm is higher than the right hemidiaphragm by greater than 1 intercostal space on a chest radiograph, diaphragmatic paralysis on the left should be suspected. The right hemidiaphragm is normally slightly higher than the left hemidiaphragm; however, if this elevation is greater than 2 intercostal spaces, paralysis of the right hemidiaphragm should be suspected. Lack of mobility on real-time ultrasound or on fluoroscopy confirms diaphragmatic paralysis. Early plication of the paralyzed hemidiaphragm lowers the length of mechanical ventilation and hospital length of stay. A new ferromagnetic surgical dissection system may be promising in lowering the risk of phrenic nerve injury. ABP Content Specifications(s)/Content Area Know the causes of phrenic nerve injury (birth trauma, surgical trauma) Plan the management of a patient with diaphragmatic paralysis Suggested Readings Floh AA, Zafuallah I, MacDonald C, Honjo O, Fan CS, Laussen PC. The advantage of early plication in children diagnosed with diaphragm paresis. J Thorac Cardiovasc Surg. 2017;154(5): 1715-1721. doi: 10.1016/j.jtcvs.2017.05.109 Shinkawa T, Holloway J, Tang X,Gossett JM, Imamura M. A ferromagnetic surgical system reduces risk of phrenic nerve injury in redo congenital cardiac surgery. Interact Cardiovasc Thorac Surg. 2017;24(5):802-803. doi: 10.1093/icvts/ivw444 Smith BM, Ezeokoli NJ, Kipps AK, Azakie A, Meadows JJ. Course, predictors of diaphragm recovery after phrenic nerve injury during pediatric cardiac surgery. Ann Thorac Surg. 2013;96(3):938- 942. doi: 10.1016/j.athoracsur.2013.05.057

You are caring for an 18-month-old boy who was born at 26 weeks gestation. The child has a large ventricular septal defect and presented with a viral pneumonia, pulmonary hypertension, and acute respiratory failure. He is intubated and mechanically ventilated. You are reviewing the current laboratory results on hospital day 1 and discussing current treatment plans with your fellow. Admission laboratory results Hospital day 1 laboratory resultsArterial blood gaspH 7.2PaCO2 80 mm HgPaO2 76 mm HgHCO3- 34 mmol/LpH 7.47PaCO2 38 mm HgPaO2 93 mm HgHCO3- 32 mmol/LNa+129 mEq/L131 mEq/LK+3.2 mEq/L2.9 mEq/LCl-86 mEq/L83 mEq/LCO235 mEq/L36 mEq/LLactate2.3 mmol/L1.7 mmol/L Of the following, his hemoglobin oxygen dissociation curve is MOST likely: A. shifted left with decreased affinity for oxygen B. shifted left with increased affinity for oxygen C. shifted right with decreased affinity for oxygen D. shifted right with increased affinity for oxygen

B. shifted left with increased affinity for oxygen Tissue oxygenation is directly related to oxygen delivery (DO2), which is determined by cardiac output (CO) and oxygen content (CaO2). Cardiac output is determined by heart rate (HR) and stroke volume (SV). Oxygen carrying capacity is determined by the quantity of oxygen bound to hemoglobin plus the quantity of oxygen dissolved in the plasma. DO2 = CO x CaO2CO = HR x SVCaO2 = (SaO2 x Hgb x 1.36) + (PaO2 x 0.003) An oxygen deficit to the tissues is manifested by a shift from aerobic to anaerobic metabolism resulting in production of lactate. Other factors that can affect tissue oxygenation include acid-base status, temperature, and 2,3 - diphosphoglycerate (DPG). Change in blood CO2 concentration to enhance oxygenation is known as the Bohr effect. This is best illustrated by the hemoglobin oxygen dissociation curve (Figure). As blood PCO2 decreases, pH increases (alkalosis) resulting in a left and upward shift of the curve and a decrease in the P50. The left and upward shift of the hemoglobin oxygen dissociation curve results in an increased affinity of oxygen to the hemoglobin molecule making this the correct answer. As blood PCO2 increases, pH decreases (acidosis) causing a right and downward shift of the curve and increase in P50. Shifting the hemoglobin oxygen dissociation curve right and downward results in less oxygen affinity to the hemoglobin molecule facilitating oxygen unloading at the tissue level. In addition to acid-base status, increased temperature, and increased 2,3-DPG shift the hemoglobin oxygen dissociation curve to the right. The hemoglobin oxygen dissociation curve shifts left with decreased temperature, fetal hemoglobin, methemoglobin, carboxyhemoglobin, and a reduction of 2,3-DPG. The 2,3-DPG levels are increased in patients with anemia, chronic hypoxia, and hyperthyroidism. The 2,3-DPG levels are decreased in stored blood and erythrocyte enzyme deficiencies. PREP Pearls Acid base status affects tissue oxygenation by shifting the hemoglobin oxygen dissociation curve. Increased temperature, and increased 2,3-DPG cause a shift of the hemoglobin oxygen dissociation curve. Fetal hemoglobin causes a left shift of the hemoglobin oxygen dissociation curve increasing oxygen affinity for the hemoglobin molecule. ABP Content Specifications(s)/Content Area Understand how acidosis affects tissue perfusion Understand how alkalosis affects tissue perfusion Suggested Readings Almodovar MC, Kulik TJ, Charpie JR. Assessment of cardiovascular Function. In: Fuhrman and Zimmerman. Pediatric Critical Care. 4th Edition. Philadelphia, PA: Elsevier Saunders; 2011:246-248. Hall JE. Guyton and Hall. Textbook of Medical Physiology. 11th Edition. Philadelphia, PA: Elsevier Saunders; 2006:504-507.

A 4-year-old girl is admitted to the hospital for respiratory distress in the setting of influenza A infection. Her initial chest radiograph is consistent with a viral pneumonia with scattered atelectasis and some perihilar opacities. On hospital day 4 she develops a new high fever (>40°C), leukocytosis, and increasing respiratory distress requiring endotracheal intubation and transfer to the PICU. Her chest radiograph shows new bilateral parenchymal opacities and a left-sided pleural effusion. There is concern about a superimposed bacterial pneumonia, and broad-spectrum antibiotics are initiated. She continues to worsen over the next several days and an endotracheal aspirate culture grows methicillin-resistant Staphylococcus aureus. Her chest radiograph on hospital day 7 shows increased left-sided pleural effusion (Figure). She remains febrile with an elevated white blood cell count. A needle thoracentesis of the effusion is performed and fluid is sent for analysis. Gram stain of the fluid reveals gram-positive cocci in clusters. Further laboratory findings are as follows: Laboratory Test Results Serum Pleural Fluid White blood cell count, per μL (per L) 33,000 (33 × 109) 100,000 (100 × 109) Lactate dehydrogenase, U/L 203 1,075 Total protein, g/dL (g/L) 6.2 (62) 15 (150) Glucose, mg/dL (mmol/L) 138 (7.7) 50 (2.8) A chest tube is placed and initially drained 250 mL of cloudy fluid. Two days later, ongoing chest tube drainage is minimal and the appearance of the chest film has not materially changed. Of the following, the BEST next step in treatment for this patient is A. aggressive diuresis B. decortication C. fibrinolytic therapy D. pleurodesis

C. fibrinolytic therapy The patient in this vignette has an exudative parapneumonic effusion associated with methicillin-resistant Staphylococcus aureus pneumonia. Pleural effusions may be classified as transudative or exudative. Transudative effusions are the result of alterations in Starling forces leading to leakage of fluid into the pleural space. Starling's hypothesis states that fluid movement between the intravascular and extravascular space is dependent on the difference between the hydrostatic and oncotic pressures of each space. When the intravascular hydrostatic pressure exceeds the interstitial hydrostatic pressure, then fluid moves into the interstitium. Conversely, when intravascular oncotic pressure is lower than the interstitial oncotic pressure, then fluid moves into the interstitial space. In contrast, exudative effusion occurs when there is local disruption of tissue integrity due to cell injury and/or inflammation. This disruption in the local architecture allows for both fluid and larger molecules such as protein and cells to move from the intravascular to the interstitial space. Light's criteria have long been used to help differentiate transudative and exudative effusions and are highly sensitive. If at least one of the following criteria are met, then the effusion is classified as an exudate: Pleural fluid to serum protein ratio of >0.5 Pleural fluid to serum lactate dehydrogenase (LDH) ratio of >0.6 Pleural fluid LDH level of >0.6 IU/L or two-thirds the upper limit of serum normal The LDH found in exudates is thought to be released from polymorphonuclear cells. Examples of disease processes associated with transudative effusions include left-sided heart failure, nephrotic syndrome, cirrhosis, and renal failure. Left-sided heart failure leads to increased pulmonary capillary hydrostatic pressure. Cirrhosis, nephrotic syndrome, and renal failure are associated with low levels of intravascular protein and decreased intravascular oncotic pressure. These effusions are composed of serous fluid. Treatment of the underlying disease process combined with diuretic therapy and/or protein replacement is often successful in these patients. In pediatrics, the most common exudative effusion is a parapneumonic effusion secondary to viral or bacterial pneumonia. Other less common types of exudative effusions in pediatrics include hemothoraces and chylothoraces related to thoracic surgery or trauma. Parapneumonic effusions are seen in up to 10% of children with pneumonia and in approximately one quarter of children hospitalized for pneumonia. Fortunately, the mortality for parapneumonic effusions in children is much less than that in the adult population where chronic lung disease is more common. The most common associated pathogens are S aureus and Streptococcus pneumoniae, but effusions may also be seen with other bacterial infections (eg, Mycoplasma pneumonia) and with viral infections. The incidence of S pneumoniae infection has fallen dramatically since the introduction of the 13-valent pneumococcal vaccine. The progression of parapneumonic effusion typically occurs in 3 stages. Initially, pleural inflammation leads to leakage of fluid, proteins, and a limited number of leukocytes into the pleural space. This is known as a simple effusion and the fluid is usually still sterile. With continued inflammation and infection, bacteria eventually invade this space, leading to the formation of a fibropurulent effusion, also known as an empyema. During this time development of fibrinous strands commonly leads to loculation of the fluid. Findings suggestive of an empyema in the pleural fluid include gross purulent appearance, decreasing pH, a leukocyte count of greater than 50,000 cells/µL, a glucose level less than 50 mg/dL, and an LDH level of greater than 1,000 U/L. Loculated parapneumonic effusions are also known as "complicated" parapneumonic effusions, and on decubitus chest radiograph they do not form the smooth layer seen in the setting of a simple effusion. The first 2 phases of parapneumonic effusion formation occur within the first 2 weeks of infection. After this, the effusion becomes organized. In this stage, fibrous tissue forms on the parietal and visceral pleura forming a restrictive "peel" that inhibits full lung expansion. Simple parapneumonic effusions in a clinically stable child may be treated with antibiotics and observation for 48 to 72 hours. Approximately 50% of children with moderate to large effusions treated with antibiotics alone will ultimately need drainage of the fluid. Larger size, complex nature, and declining clinical status are all indications for more invasive management. Drainage via needle thoracentesis is acceptable for simple effusions, however many children still require further placement of a chest tube, so this practice is becoming less common. Fluid should be sent for cell count and Gram stain as well as LDH, glucose, and protein levels. If a noninfectious etiology such as malignancy or rheumatologic process is suspected, further testing may be indicated. The placement of a catheter for continuous drainage is the most common course of action for treatment of moderate to large parapneumonic effusions. Historically, surgical chest tubes were thought to provide superior drainage of the viscous purulent fluid, but studies have not supported this opinion. Pigtail catheters tend to be less painful and better tolerated. Bedside ultrasonography guidance can aid in the optimal placement of the catheter. Patients who do not have clinical improvement in the first several days after chest tube placement or who have complex parapneumonic effusions require management with fibrinolytic therapy or video-assisted thoracoscopic surgery. It may be reasonable to start with fibrinolytic therapy instilled into a chest tube since one is frequently already in place, and this process is less invasive than proceeding directly to video-assisted thoracoscopic surgery; however, either is acceptable as a next step. Most children will respond to fibrinolytic therapy but a small percent (approximately 15%) will still require surgical intervention. Alteplase is the most commonly used fibrinolytic in the United States. Failure of fibrinolytic therapy, extensive loculations, and continued septic physiology despite adequate fluid drainage are all indications to proceed to video-assisted thoracoscopic surgery. In this procedure, the pleural space can be directly visualized, and all areas of loculation and adhesion can be lysed for thorough drainage. Studies have shown that advancement to video-assisted thoracoscopic surgery early can help decrease hospital length of stay. Parapneumonic effusions that advance to the organized phase will require open decortication to remove the thick fibrinous rind on the parietal pleura and allow for full lung re-expansion. This is a more extensive procedure requiring a thoracotomy incision and is rarely required in pediatric patients. Although nephrotic syndrome has been reported in patients with influenza A infection, this patient has normal protein levels that make this an unlikely cause of her parapneumonic effusion. Left-sided heart failure leads to a transudative effusion, not an exudative effusion, due to increases in pulmonary capillary hydrostatic pressure and would not require decortication for treatment. A hemothorax, or presence of red blood cells in the pleural space, is an exudative process (not a transudative process) and is most commonly due to retained blood in the pleural space following thoracic surgery or trauma. Clinically significant hemothoraces require drainage and will not respond to diuresis. PREP Pearls Parapneumonic effusions may be transudates or exudates and may be differentiated using Light's criteria. The most common cause of exudative parapneumonic effusion in children is bacterial pneumonia. Simple effusions respond well to drainage alone. More complex effusions require invasive intervention with fibrinolytic therapy, diuresis, or, rarely, decortication. ABP Content Specifications(s)/Content Area Differentiate among the causes of pleural effusion Plan the management for a patient with complicated parapneumonic effusion Suggested Readings George J. Transudate vs exudate. [Khan academy video.] https://www.khanacademy.org/science/health-and-medicine/respiratory-system-diseases/pleural-effusion-2/v/transudate-vs-exudate. Light RW. Parapneumonic effusions and empyema. Proc Am Thorac Soc. 2006;3(1):75-80. doi:10.1513/pats.200510-113JH.

A 20-month-old former 26-week premature infant is admitted to the pediatric intensive care unit with altered mental status. His grandmother, who was babysitting for him, is not sure if he could have gotten in to medications that were in her purse. She noted that he was fine yesterday, but today has been stumbling and vomited several times after being put down for his nap. He has not had fever. She is not familiar with his neonatal intensive care unit course, but she knows that he is not on any medications and has not been on oxygen for many months. Vital signs: 37.2℃, heart rate 80 beats/min, 10 breaths/min, SpO2 96% on room air, blood pressure 132/70 mm Hg. On examination, you find a somnolent child in no distress who responds to examination with a high-pitched cry. His pupils are 3 mm, reactive, and in a downward gaze. He has tubing palpable in his right neck. There is a healed small scar along his left thorax and a healed former gastrostomy tube site. No rash is observed. Heart, lung, and abdominal examinations are unremarkable. Of the following, the one that will most likely confirm your diagnosis is: A. blood cultures B. echocardiogram C. head computed tomography scan D. urine drug screen

C. head computed tomography scan The child in the vignette is hypertensive, bradycardic, and bradypneic as well as afebrile. This constellation of symptoms combined with your physical examination finding of the presence of tubing consistent with a ventriculo-peritoneal/-pleural/-atrial shunt makes hydrocephalus secondary to shunt failure the likely diagnosis. Brain tumor is possible but unlikely with only a few hours of symptoms. Hydrocephalus results from increased cerebral spinal fluid (CSF) production, impeded CSF flow, or impairment in CSF absorption. Obstructive hydrocephalus results, as the name suggests, from functional or anatomical blockage of the CSF flow resulting in the expansion of the proximal chamber or space; whereas, nonobstructive hydrocephalus is the result of excess production or decreased absorption of CSF. The "bulk flow" concept was first proposed by Cushing in 1926. Our basic understanding of CSF flow is that CSF is produced primarily by the choroid plexus composed of tufts of capillaries with thin fenestrated endothelial cells and ependymal cells with microvilli in the lateral ventricles, flows through third and fourth ventricles, cisterns, and subarachnoid space, then is absorbed into the dural venous sinuses via arachnoid villi (Figure). In recent years it has become evident that the process of CSF production, flow, and reabsorption is much more complex than previously defined, with intracranial pulsations and lymph movement contributing to flow. This is beyond the scope of this review. Production of CSF occurs by several mechanisms: diffusion, pinocytosis, and active transfer. Adults have about 100 to 150 mL of CSF (infants have ~50 mL) and make ~0.4 mL/min, replenishing the fluid every 4 to 6 hours. The choroid plexus accounts for ~80% of CSF production and the brain interstitial fluid constitutes the remainder. The constant exchange between these 2 is termed the "glymphatic system." Historically, the flow rate for CSF in adults is reported to be 0.4 to 0.8 mL/min, yet more recent data using magnetic resonance imaging indicates it may be twice that. In children younger than 2 years and adults with normal pressure hydrocephalus, caudocranial (reverse) flow has been observed. Increased CSF production may be caused by congenital choroid plexus hyperplasia or a choroid plexus papilloma, which accounts for 1% to 4% of pediatric brain tumors, usually presenting at <2 years of age. These conditions may result in vast quantities of CSF production, up to 5 L/day being reported in a 6-month-old infant. Additionally, idiopathic intracranial hypertension (pseudotumor cerebri) and infection-related hydrocephalus are thought to be the result of overzealous CSF production. Cerebral spinal fluid hypersecretion has been proposed as one contributor to pediatric hydrocephalus. Cerebral spinal fluid production can be decreased surgically via choroid plexus ablation or pharmacologically. Surgery is the best option for hyperplasia or choroid plexus tumor and frequently is combined with third ventriculostomy. Carbonic anhydrase inhibitors, such as acetazolamide and topiramate, are effective in modestly decreasing CSF production by preventing central nervous system neurons from discharging excessively and affecting HCO3- and Na+ availability. Acetazolamide yields a 30% to 60% decrease in CSF production. Combination therapy using acetazolamide and a loop diuretic was found to be neither effective nor safe in posthemorrhagic hydrocephalus. Absorption of CSF occurs primarily in the arachnoid villi, but drainage into the lymphatic system and brain parenchyma itself also occurs, though the extent is unclear. Studies done in the 1960s on adults and children with brain tumors demonstrated maximal CSF resorption of 1.3 mL/min. Interestingly, newer studies have demonstrated that children under 2 years of age do not actually have functional arachnoid granulations. Decreased CSF absorption has a number of etiologies including infection (eg, congenital infection, such as TORCH, or acquired infection), presence of blood (which decreases arachnoid villi resorption capability), inflammation, or increased venous pressure (seen with Vein of Galen malformation). Cerebral spinal fluid drainage can be surgically augmented via placement of a ventriculoperitoneal, ventriculopleural, or ventriculoatrial shunt. Within months to years, microglia and astrocytes migrate and may ultimately result in noninfectious catheter obstruction. Shunt infection rates average ~6%, with 90% of those occurring in the 6 months after placement. Use of antibiotic impregnated catheters appear to have little impact. Most infections are with skin flora despite many efforts to improve operating suite processes. Subsequent reinfection risk is high, occurring in ~26% of cases, likely within ~2 months of the 1st infection. Overdrainage of CSF can occur, especially in ventriculopleural shunts exposed to negative intrathoracic pressure. Physical shunt catheter failure may occur due to fracture, disconnection, or migration. PREP Pearls Carbonic anhydrase inhibitors, such as acetazolamide and topiramate, are effective in modestly decreasing cerebral spinal fluid production. After a first shunt infection, the child has a ~26% chance of experiencing reinfection; likely within ~2 months of the first infection. Children under 2 years of age may not actually have functional arachnoid granulations. ABP Content Specifications(s)/Content Area Recognize that increased cerebrospinal fluid production and decreased reabsorption can both cause hydrocephalus Know the basic physiology of cerebrospinal fluid production, absorption, and circulation Suggested Readings Brinker T, Stopa E, Morrison J, Klinge P. A new look at cerebrospinal fluid circulation. Fluids Barriers CNS. 2014;11:10. doi: 10.1186/2045-8118-11-10 Hanak BW, Bonow RH, Harris CA, Browd SR. Cerebrospinal fluid shunting complications in children. Pediatr Neurosurg. 2017;52(6):381-400. doi: 10.1159/000452840 Karimy JK, Duran D, Hu JK, et al. Cerebrospinal fluid hypersecretion in pediatric hydrocephalus. Neurosurg Focus. 2016;41(5):E10. doi: 10.3171/2016.8.FOCUS16278 Limbrick DD Jr, Baird LC, Klimo P Jr, et al; Pediatric Hydrocephalus Systematic Review and Evidence-Based Guidelines Task Force. Pediatric hydrocephalus: systematic literature review and evidence-based guidelines. Part 4: cerebrospinal fluid shunt or endoscopic third ventriculostomy for the treatment of hydrocephalus in children. J Neurosurg Pediatr. 2014;14 Suppl 1:30-34. doi: 10.3171/2014.7.PEDS14324 Raper D, Louveau A, Kipnis J. How do meningeal lymphatic vessels drain the CNS? Trends Neurosci. 2016;39(9):581-586. doi: 10.1016/j.tins.2016.07.001

A 17-year-old adolescent girl presents to the emergency department at 7 AM after being found unresponsive in her bedroom by her mother. Empty bottles of both acetaminophen and oxycodone were found near her. Her mother states that the patient was last seen at 11 PM the previous evening. Her vital signs are pulse 50 beats/min, blood pressure 90/40 mm Hg, respiratory rate 12 breaths/min, pulse oximetry 90% on ambient air, and temperature 36°C. Initial laboratory values are as follows: Arterial Blood Gas Result pH 7.15 PaCO2 35 mm Hg PaO2 60 mm Hg Laboratory Test HCO3 12 mM Sodium 140 mM Potassium 4 mM Chloride 100 mM Blood Urea Nitrogen 18 mg/dL Creatinine 1 mg/dL Serum Acetaminophen Level 400 µg/mL (400 mg/L) Ammonia 30 µM Aspartate Transaminase (AST; SGOT) 100 U/L Alanine Transaminase (ALT; SGPT) 60 U/L Unconjugated Bilirubin 0.4 mg/dL Conjugated Bilirubin 0.2 mg/dL Prothrombin Time 14 seconds After a 20 mL/kg normal saline bolus and naloxone, the patient's mental status improves, her respiratory rate increases to 25 breaths/min, and blood pressure increases to 100/50 mm Hg. She is started on intravenous N-acetylcysteine and transferred to the intensive care unit for continued monitoring. Sixty hours after intensive care unit admission, the patient becomes confused and then somnolent. Repeat laboratory values show the following: Laboratory Test Result Serum Acetaminophen Level 60 µg/mL (60 mg/L) Ammonia 180 µM Aspartate Transaminase (AST; SGOT) 3,000 U/L Alanine Transaminase (ALT; SGPT) 3,000 U/L Unconjugated Bilirubin 2.4 mg/dL Conjugated Bilirubin 1.2 mg/dL Prothrombin Time 18 seconds Of the following, the BEST next therapy is: A. bolus of N-acetylcysteine, intravenous B. enteral lactulose infusion C. hypertonic sodium chloride intravenous infusion D. therapeutic plasma exchange

C. hypertonic sodium chloride intravenous infusion This patient in the vignette has developed grade III hepatic encephalopathy (HE) due to acetaminophen (APAP)-induced hepatotoxicity and acute liver failure. Acetaminophen-induced liver failure continues to be the leading cause of acute liver injury and failure in the United States and Europe, accounting for one-sixth of acute liver failure leading to liver transplantation listing. The mechanism of action is through the formation of the toxic metabolite N-acetyl-p-benzoquinone imine (NAPQI) by the cytochrome P450 enzyme, CYP2E1. Normally, NAPQI is cleared through conjugation by glutathione and excreted in the urine. However, if APAP is taken in sufficient quantity at once, glutathione stores may become depleted and intracellular NAPQI concentrations start to rise. The loss of the antioxidant glutathione leads to downstream oxidative stress within the cell. Additionally, NAPQI interacts with multiple intracellular proteins, disrupting their functions. Taken together, this leads to hepatocyte necrosis and, if the acetaminophen dose is large enough, acute liver failure. Histologic examination of APAP-injured livers show pericentral necrosis. Currently, the gold-standard of treatment to attenuate APAP-induced liver injury/failure is N-acetylcysteine (NAC), which serves to replete glutathione stores in the liver and aid in NAPQI conjugation and elimination. The management of acute liver failure consists of: 1) treatment of the underlying cause, if the cause is known and a treatment is available (eg, NAC treatment for APAP), and 2) supportive care. The liver is a complex organ that serves multiple functions, including protein synthesis (eg, clotting factors and albumin), gluconeogenesis, toxin filtration and removal, bilirubin excretion, maintenance of vasoactive tone, and immunity. In patients with end-stage liver disease (both acute and chronic), liver transplantation is the long-term treatment. In patients with liver failure, ammonia and other metabolites accumulate and lead to HE, cerebral edema, and eventual brain death. Although the exact mechanisms through which hyperammonemia precipitates HE are not clear, it is generally thought that elevated ammonia levels in the brain causes: 1) neuronal excitotoxicity, and 2) astrocyte swelling, leading to cerebral edema and, eventually, death. Treatment for HE revolves around its prevention, recognition, and supportive care until liver recovery or transplantation. Hepatic encephalopathy is graded from I-IV, with increasing grades portending a worse prognosis: At lower grades, efforts to restrict protein intake (dietary protein), protein turnover (gastrointestinal bleeding), and protein/nitrogen absorption (through intestinal bacteria) should be instituted. Monitor and start prophylaxis for gastrointestinal bleeding and initiate lactulose with/without rifaximin to limit protein/nitrogen absorption. At higher grades of HE (III & IV), patients are at increasing risk for cerebral edema and should be monitored more closely. Intubation and mechanical ventilation should be instituted to protect the airway and to ensure normoventilation and minimize excessive cerebral blood flow secondary to hypercapnia. The head of the bed should be elevated to 30° to facilitate cerebral venous drainage. Hypertonic saline should be initiated to maintain a serum sodium level of 145 to 155 mmol/L. Treatment with hypertonic saline in a randomized controlled trial comparing standard of care to hypertonic saline (3% sodium chloride) plus standard of care showed reduced incidence and severity of intracranial hypertension than with standard of care alone, although overall survival was unchanged. For patients at institutions with neurosurgical expertise and awaiting liver transplantation, intracranial pressure monitoring is also recommended to help guide therapy. For the patient in the vignette, the best therapy to institute next is administration of hypertonic sodium chloride to achieve a serum sodium level of 145 to 155 mmol/L. Enteral lactulose, while useful in attenuating rising ammonia levels, will not have an immediate effect in this case. An additional bolus of NAC is not warranted at this time since she has already received treatment for an acute APAP overdose. Finally, therapeutic plasma exchange is not necessary, as there is no evidence of bleeding. PREP Pearls Cerebral edema due to hepatic encephalopathy is a leading cause of death in acute liver failure. Treatment of severe cases of hepatic encephalopathy is primarily supportive, with a goal serum sodium level of 145-155 mM. ABP Content Specifications(s)/Content Area Know the principles of treatment of fulminant hepatic failure Suggested Readings Larson AM, Polson J, Fontana RJ, et al. Acetaminophen-induced acute liver failure: results of a United States multicenter, prospective study. Hepatology. 2005;42(6):1364-1372. doi: 10.1002/hep.20948 Lee WM, Larson AM, Stravitz RT. AASLD position paper: the management of acute liver failure: update 2011. Hepatology. 2011:1-22. Lee WM, Squires RH Jr, Nyberg SL, Doo E, Hoofnagle JH. Acute liver failure: summary of a workshop. Hepatology. 2008;47(4):1401-1415. doi: 10.1002/hep.22177 Murphy N, Auzinger G, Bernel W, Wendon J. The effect of hypertonic sodium chloride on intracranial pressure in patients with acute liver failure. Hepatology. 2004;39(2):464-470. doi: 10.1002/hep.20056 Squires JE, McKiernan P, Squires RH. Acute liver failure: an update. Clin Liver Dis. 2018; 22:773-805. doi: 10.1016/j.cld.2018.06.009

A 6-year-old boy presents to the emergency department in respiratory distress and is admitted to the intensive care unit. He is noted to have wheezing on pulmonary examination. Treatment with albuterol nebulization therapy has minimal effect. Physical examination is significant for low-grade fever and bilateral otitis media. Parents report he has had numerous "lung problems" and is followed by pulmonary medicine. Prior notes from pulmonary colleagues show multiple negative sweat chloride analyses and nondiagnostic genetic screens for cystic fibrosis. Pulmonary function testing demonstrates a decreased forced expiratory volume at 1 second without reversibility following bronchodilator therapy. A chest CT is obtained (Figure). Of the following, which test is indicated when considering the implications of this patient's history and radiographic findings? A. α-1 antitrypsin activity B. human leukocyte antigen B 27 C. immunoglobulin levels D. PHOX2B genetic analysis

C. immunoglobulin levels The chest CT in this scenario shows evidence of diffuse bronchiectasis. When evaluating bronchiectasis not due to cystic fibrosis, primary immune deficiencies must be considered. Some authors estimate up to 10% of recurrent sino-pulmonary bacterial infections in the pediatric-aged patient are associated with immunodeficiencies. Primary immunodeficiencies are rare in pediatrics, occurring in 1 per 10,000 births. The exception is immunoglobulin (IgA) deficiency with a relative population frequency of 1 in 700. Approximately 50% to 70% of primary immune disorders involve defects in B cells, including antibody deficiencies. T cell and combined B cell/T cell defects are the next most prevalent at 20% to 30%. Antibody deficiencies typically present prior to 12 months of age. Examples of antibody deficiency include x-linked agammaglobulinemia, hyper immunoglobulin (IgM) syndrome, selective immunoglobulin (IgG) subclass deficiency, and common variable immunodeficiency. Antibody deficiencies lead to recurrent sinopulmonary infections, typically caused by encapsulated bacteria including Streptococcus pneumoniae, Hemophilus influenzae, Moraxella catarrhalis, Pseudomonas aeruginosa, and Staphylococcus aureus. The prototype antibody deficiency syndrome is X-linked agammaglobulinemia. In this disease, all immunoglobulin subclasses are extremely low and there is an inability to make specific antibodies to either infections or immunizations. Pre-B cells are noted in the bone marrow, but circulating B cells are absent, as are plasma cells in lymph nodes. Patients typically present with recurrent pyogenic infections. Mean age of onset is 6 months of age as maternally acquired antibodies wane. Children typically present with severe invasive infections such as life-threatening pneumonia, meningitis, or septic shock. Over half of these patients eventually develop radiographic findings of chronic lung disease. Immunoglobulin (IgG) subclass deficiencies are often asymptomatic. Individuals may have normal total IgG, but one or more subclasses will be below the fifth percentile. Recurrent sinopulmonary infections are reported in children with deficiencies in IgG2 or IgG3 most commonly, while IgG1 deficiencies are seen in adults. IgG4 levels can be very low or absent in Caucasian populations and likely do not contribute to susceptibility to infections. IgG subclass deficiencies can also be seen with other immunodeficiencies such as Wiskott-Aldrich, ataxia-telangiectasia, hyper-IgM syndrome or common variable immunodeficiency. IgA deficiency is by far the most common primary immunodeficiency, occurring with an incidence of 1 in 700 (incidence of diabetes mellitus in the U.S. is 1 in 600). It was initially believed that IgA deficiency did not increase susceptibility to infections. However, a recent study from Sweden of over 2,100 individuals with IgA deficiency showed a 17% increase in all cause infections compared to controls. These infections mostly involved the respiratory and gastrointestinal tracts. Recurrent bacterial infections of the upper and lower respiratory tract are the most frequent clinical presentation of antibody-associated immunodeficiencies. B cells from lymphoid tissue in the upper respiratory tract preferentially produce IgA, while B cells from the lower respiratory tract produce more IgG. IgA is secreted unto and protects the stratified columnar epithelium of the upper respiratory tract while IgG protects the pulmonary interstitial space and alveoli. The role antibodies have in preventing sinopulmonary disease is complex. They prevent mucosal attachment, block cellular entry, or expedite destruction of bacteria and viruses. Additionally, antibody absence impairs complement activation, as well as neutrophil chemotaxis and phagocytosis. Early identification of pulmonary disease and intervention with antibiotic therapy (+/- IV immunoglobulin) improves morbidity and mortality in these patients. In addition to acute lung infections, recent evidence suggests antibody deficiencies play a role in chronic lung disease processes such as interstitial lung disease and bronchiectasis. The interstitial lung disease associated with immunoglobulin deficiencies shows histopathologic features of lymphocytic interstitial pneumonitis and granulomatous inflammatory response. In a recent study from the United Kingdom, 47% of immunoglobulin deficient patients developed bronchiectasis, a persistent dilation of the bronchial airway. Bronchiectasis is characterized by worsening airflow obstruction and clinically presents as cough, shortness of breath, and sputum production. Symptoms do not improve with bronchodilator therapy. Bronchiectasis is frequently seen in cystic fibrosis patients and is felt to be secondary to repeated infections in this population. Bronchiectasis is also noted in patients with common variable immunodeficiency, X-linked agammaglobulinemia, and IgG subclass deficiency both with and without IgA deficiency. In a large pediatric study of 989 patients with noncystic fibrosis bronchiectasis, 16% were associated with primary immunodeficiencies and 3% with secondary immunodeficiencies. In patients with primary antibody deficiencies, bronchiectasis may progress despite antibiotic and IV immunoglobulin prophylaxis. The pathogenesis of bronchiectasis in these patients is unclear, but may be due to recurrent infections leading to a chronic inflammatory state with resultant airway remodeling. In addition to broad-spectrum antibiotics, intravenous immunoglobulin (IVIG) therapy is indicated when bronchiectasis is due to a primary antibody deficiency. Investigators have shown that an inverse relationship exists between IgG levels and bronchiectasis. Those with the lowest IgG levels tend to develop more severe bronchiectasis, suggesting that delays in diagnosis as well as insufficient IVIG therapy contribute to disease pathology. Data on specific antibiotic choice in this patient population is lacking. A reasonable choice might include a macrolide, both for its bactericidal potential but also for its anti-inflammatory action. Of the choices offered, quantitative and qualitative immunoglobulin screening is the most reasonable. PHOX2B genetic mutations should be considered in the evaluation of central hypoventilation syndromes. Alpha-1 antitrypsin deficiency can cause severe emphysema, especially following tobacco exposure. Bronchiectasis has been reported, but in the adult patient population. Human leukocyte antigen B27 testing is indicated for ankylosing spondylitis, which can rarely present as diffuse interstitial lung disease, but typically does not demonstrate bronchiectasis. PREP Pearls Primary immunodeficiencies frequently present as recurrent pneumonias in the first year of life, particularly with encapsulated organisms. Patients with primary immunoglobulin deficiencies can develop severe bronchiectasis and chronic lung disease. Treatment of bronchiectasis in patients with primary antibody deficiency involves both broad-spectrum antibiotics and intravenous immunoglobulin therapy. ABP Content Specifications(s)/Content Area Understand the role of immunoglobulins in pulmonary host defense Suggested Readings Alvarez B, Arcos J, Fernandez-Guerrero ML. Pulmonary infectious diseases in patients with primary immunodeficiencies and those treated with biologic immunomodulating agents. Curr Opin Pulm Med. 2011;17(3):172-179. doi: 10.1097/MCP.0b013e3283455c0b Berger M, Geng B, Cameron DW, Murphy LM, Schulman ES. Primary immune deficiency diseases as unrecognized causes of chronic respiratory disease. Respir Med. 2017;132:181-188. doi: 10.1016/j.rmed.2017.10.016 Schussler E, Beasley MB, Maglione PJ. Lung disease in primary antibody deficiencies. J Allergy Clin Immunol Pract. 2016; 4(6):1039-1052. doi: 10.1016/j.jaip.2016.08.005 Verma N, Grimbacher B, Hurst JR. Lung disease in primary antibody deficiency. Lancet Respir Med. 2015;3(8):651-660. doi: 10.1016/S2213-2600(15)00202-7

A 7-year-old boy is hospitalized because of fatigue and shortness of breath that have been progressively worsening over the past 3 weeks. He was diagnosed with acute lymphoblastic leukemia at age 4 years and completed treatment a year ago, with no sign of disease recurrence. On admission, he appears tired but is in no distress. Vital signs include temperature 36.8°C, heart rate 80 beats/min, blood pressure 95/56 mm Hg (mean 69 mm Hg), and respiratory rate 18 breaths/min. His skin is pale but warm, and distal pulses are palpable. Lungs are clear and heart examination shows an S3 gallop. Chest radiograph shows mild interstitial pulmonary edema and cardiomegaly, and an echocardiogram shows ventricular dilatation and decreased ejection fraction, with thinning of the ventricular wall. Of note, in a routine oncology clinic visit a year ago, his vital signs were: temperature 37.0°C, heart rate 75 beats/min, blood pressure 91/58 mm Hg (mean 69 mm Hg), and respiratory rate 16 breaths/min. A chest radiograph showed normal heart size. Compared to his status on that previous clinic visit, his left ventricular afterload now is MOST likely: A. cannot be determined from this information B. decreased C. increased D. unchanged

C. increased The patient has experienced new onset heart failure with cardiac dilatation, most probably caused by exposure to anthracycline chemotherapy for treatment of his leukemia. Left ventricular afterload, conceptually the workload that the ventricle has to shoulder, is most often assessed by measuring arterial blood pressure, which during systole is essentially the same as intraventricular pressure unless there is aortic stenosis. In this scenario, however, the blood pressure in the 2 situations is not materially different. Afterload is more properly considered as the workload for cardiac muscle fibers, which is determined not by the pressure inside the ventricle but by ventricular wall stress. While ventricular wall stress is indeed proportional to ventricular pressure, it is not solely determined by pressure, and is also related to ventricular size and wall thickness, as noted below. Wall stress ∝ P x r / h (P = ventricular pressure; r = ventricular radius; h = wall thickness) This equation is derived from the relationship of LaPlace, which states that for a thin-walled sphere or cylinder, that wall tension is proportionate to the pressure and the radius. Of course, the left ventricle is neither spherical nor thin-walled, to say nothing of the right ventricle, but the principle remains instructive. For the patient in the vignette, therefore, both cardiac dilation and thinning of the ventricular wall have resulted in increased wall stress and therefore increased afterload, despite an unchanged arterial pressure. Afterload is one of the determinants of stroke volume and hence of cardiac output. Increased afterload decreases stroke volume because increased wall stress, that is, increased tension on cardiac muscle fibers, decreases the speed at which the muscle fibers shorten, and hence decreases the rate at which blood is ejected from the ventricle. Since the amount of time for ejection during the cardiac cycle is relatively fixed, reduced ejection speed must necessarily reduce the amount of blood ejected, and thereby reduce stroke volume. It should be noted, however, that the healthy, nondiseased heart has considerable functional reserve and is generally resistant to increased afterload, in contrast to the failing heart, which may be exquisitely sensitive to increased afterload. The heart may compensate for increased afterload by an increased inotropic state. Inotropy is primarily regulated by the autonomic nervous system, with an increase in sympathetic tone resulting in increased inotropy, as will release of endogenous epinephrine. Clinically, inotropy is increased by administration of inotropic drugs (eg, epinephrine, dopamine, milrinone). Physiologically, inotropic state is manifested by increased tension generated by the cardiac myocytes that results in increased velocity of myocyte fiber shortening. This results in a more rapid rise in pressure in the ventricular chamber during systole, more rapid ejection of blood, and therefore increased stroke volume. PREP Pearls Ventricular afterload is best characterized by ventricular wall stress, which is proportional to pressure and chamber radius, and inversely proportional to wall thickness. Both changes in afterload and changes in inotropy effect stroke volume via effects on the velocity of myocardial muscle fiber shortening. ABP Content Specifications(s)/Content Area Understand the concept of wall stress and its effect on stroke volume Understand how contractility affects ventricular stroke volume Suggested Readings Fernandez-Jimenez R, Hoit BD, Walsh RA, Fuster V, Ibanez B. Normal Physiology of the Cardiovascular System. In: Fuster V, Harrington RA, Narula J, Eapen ZJ, eds. Hurst's The Heart. 14th ed. New York, NY: McGraw Hill; 2017 . Klabunde RE. Cardiovascular Physiology Concepts. 2nd ed. Baltimore, MD: Lippincott Williams & Wilkins; 2012.

A 6-year-old boy was pulled out of a house that was on fire by the emergency medical services and is brought to the emergency department in a coma. On physical examination, he withdraws in response to painful stimuli applied to the big toe he makes incomprehensible sounds, and he opens eyes in response to painful stimuli (Glasgow Coma Scale score of 8). Vital signs include heart rate of 159 beats/min, respiratory rate of 43 breaths/min, blood pressure of 90/60 mm Hg, and the pulse oximeter reads 99% while he is breathing ambient air. Soot in the nostrils is noted, but there is no stridor. Bilateral diffuse rales are noted upon auscultation and there are multiple areas of burns over the torso. Of the following, which is most likely erroneously high in this patient? A. Glasgow coma scale score B. heart rate C. pulse oximetry D. respiratory rate

C. pulse oximetry The child in the vignette is a victim of smoke inhalation and thermal injuries, which are often associated with carbon monoxide inhalation and poisoning. Of the clinical findings provided in the vignette, the child is likely to have pain that can lead to tachycardia.The volume depletion associated with fluid loss from the burn areas can lead to low blood pressure. The respiratory rate is likely to be high due to the effects of smoke inhalation on oxygenation and ventilation and is appropriately elevated. On physical examination, he withdraws in response to painful stimuli (4 on the motor response component of the Glasgow coma scale [GCS]), he makes incomprehensible sounds (2 on the verbal component of the GCS) and opens eyes in response to painful stimuli (2 on the eye opening component of GCS), thus the total score on the GCS is correctly calculated as 8. However, the pulse oximetry reading is likely to be erroneously high due to the high level of carboxyhemoglobin that is probably elevated as a result of carbon monoxide poisoning. Depression of mental status occurs with a carboxyhemoglobin levels greater than 30%. Carbon monoxide poisoning is a clinical condition that is often seen in victims of smoke inhalation. The smoke that is generated from the fire contains high levels of carbon monoxide that is inhaled and once it reaches the blood it binds to hemoglobin with very high affinity, forming carboxyhemoglobin. Carboxyhemoglobin interferes with the standard pulse oximetry reading and produces an erroneously high oxyhemoglobin saturation (SpO2). Pulse oximetry is a noninvasive technique that has revolutionized assessment of oxygenation in critically ill patients. The standard pulse oximetry has a diode; one of the diodes transmits light in the red (660 nm wavelength) and infrared (940 nm wavelength) and the other diode detects the change in the absorption of light across the tissues (Figure). In order to provide a measure of oxygen hemoglobin saturation, the pulse oximeter generates ratios of intensity of light, red (R)/infrared (IR), transmitted through the tissues at each wavelength. Furthermore, the pulse oximeter measures light absorption in the background when the blood in the capillaries and veins is more or less stationary and during the pulsatile phase of arterial blood flow. The ratio of absorption at these 2 wavelength [R/IR] is translated into a percentage of oxygen saturation. Under most circumstances the pulse oximetry reading is a reflection of the degree of hemoglobin saturation. The standard pulse oximeter assumes that there are only 2 light absorbers in the blood, namely reduced hemoglobin (absorbs light at 660 nm) and oxyhemoglobin (absorbs light at 940 nm). If any types of abnormal hemoglobin that absorb light in the same range are present in the blood, such as carboxyhemoglobin or methemoglobin, the pulse oximeter's detected ratio becomes invalid and it gives falsely elevated readings, in the case of carboxyhemoglobin, or a fixed result at 85%, in the case of methemoglobin. This is because carboxyhemoglobin and oxyhemoglobin both absorb red light near 660 nm wavelength and carboxyhemoglobin absorbs minimally near 940 nm wavelength, producing the erroneous readings. Indeed it has been demonstrated that even when the blood carboxyhemoglobin level is significantly elevated at 70%, the pulse oximetry displays SpO2 in the low 90s. Methemoglobin absorbs light in the 660 nm (R) as well as in the 940 nm (IR) leading to an R/IR of 1.0 which corresponds to SpO2 of 85%. Consequently in the clinical setting of methemoglobinemia the SpO2 remains fixed at 85% regardless of the severity of the methemoglobinemia. Newer types of pulse oximeters that detect 5 wavelengths instead of the traditional 2 wavelengths are available on the market . These pulse oximeters are capable of accurately detecting other abnormal types of hemoglobin, including carboxyhemoglobin and methemoglobin. This represents new advances in the pulse oximetry technology and will likely be helpful in diagnosing carbon monoxide poisoning in the field by emergency medical services so that health care providers can provide appropriate care in a timely manner for these patients. PREP Pearls Conventional pulse oximetry give erroneously high oxyhemoglobin saturation in the setting of carbon monoxide exposure. Both carboxyhemoglobin and methemoglobin absorb light in the same range of the conventional pulse oximetry and interfere with accurate reading. Significant methemoglobinemia produces an oxyhemoglobin saturation of 85% regardless of the actual level of oxyhemoglobin. Newer types of pulse oximetry are capable of measuring oxyhemoglobin, carboxyhemoglobin, and methemoglobin more accurately. ABP Content Specifications(s)/Content Area Know that carboxyhemoglobin concentrations can result in erroneously high oxygen saturation of hemoglobin during pulse oximetry Suggested Readings Barker SJ, Curry J, Redford D, Morgan S. Measurement of carboxyhemoglobin and methemoglobin by pulse oximetry. Anesthesiology. 2006;105(5):892-897. Buckley RG, Aks SE, Eshom JL, Rydman R, Schaider J, Shayne P. The pulse oximetry gap in carbon monoxide intoxication. Ann Emerg Med. 1994;24(2):252-255. Fouzas S, Priftis KN, Anthracopoulos MB. Pulse oximetry in pediatric practice. Pediatrics. 2011;128(4):740-752. doi: 10.1542/peds.2011-0271.

The Video was obtained during cardiac catheterization from a 15-year-old adolescent boy with complex congenital heart disease who has undergone multiple cardiac surgical procedures since birth. Subsequently during the procedure, he suddenly develops bradycardia that progresses to asystole and hemopericardium is detected by echocardiography. Cardiopulmonary resuscitation is initiated with tracheal intubation, positive pressure ventilation and chest compressions. Of the following, the statement that MOST accurately describes this patient's clinical condition is: A. cardiopulmonary resuscitation with chest compression has high efficacy in this patient B. the measured pressure in the superior vena cava will be significantly lower than the pressure in the superior pulmonary veins when the cardiac output returns to normal C. pulsus paradoxus is unlikely if the hemopericardium progresses to cardiac tamponade D. there is a linear relationship between mean airway pressure and cardiac output in this patient

C. pulsus paradoxus is unlikely if the hemopericardium progresses to cardiac tamponade Congenital cardiac defects that lead to single ventricle physiology include, but are not limited to, tricuspid atresia where the right ventricle is absent or dysplastic and hypoplastic left heart syndrome where the left ventricle is rudimentary. These patients undergo staged surgical procedures with the aim of separating the pulmonary and systemic circulations. The final step in the series of these cardiac surgical procedures is the modified Fontan procedure. Following the completion of the modified Fontan procedure, the superior vena cava is connected to the pulmonary artery so that venous blood from superior vena cava directly enters into the pulmonary artery as seen in Video. The blood from the inferior vena cava is directed via a lateral tunnel inside the right atrium into the pulmonary artery. Thus, the venous blood is entirely directed into the pulmonary circulation without an intervening ventricle. Normally, the pulmonary and the systemic circulations are connected in series and driven by 2 pumps, namely the right and left ventricles, respectively. In single ventricle physiology with Fontan circulation, there is no pump to propel blood into pulmonary arteries since the systemic veins (Video) are connected directly with the pulmonary arteries. Patients with a single ventricle and Fontan circulation need a sustained high central venous pressure. They are volume sensitive and should always be optimally volume repleted since the pulmonary blood flow is dependent on adequate preload. Acute volume loss such as gastroenteritis with vomiting and diarrhea can lead to acute decrease in intravascular volume in these patients with subsequent development of low cardiac output. The pulmonary artery pressure in turn must be higher than the pulmonary vein pressure in order for oxygenated blood from the lungs to reach the left atrium and finally the ventricle (which can morphologically be either a right or a left ventricle depending on the underlying cardiac defect) from where blood is ejected into the systemic circulation. Pulsus paradoxus is an exaggerated decline in the systolic blood pressure during inspiration. The major mechanism of pulsus paradoxus is increased venous return to the right ventricle during inspiration, with resultant lateral shift of the interventricular septum and hence decreased filling of the left ventricle, leading to decreased cardiac output and decreased blood pressure. Additional mechanisms include, compression of the pulmonary vasculature impeding blood flow to the left atrium with the ultimate decrease in cardiac output and blood pressure. There is also increased right ventricular afterload, partly because of increased pulmonary vascular resistance that can lead to shifting of the interventricular septum towards the left ventricle further limiting left ventricular filling, a process referred to as ventricular interdependence. Patients with single ventricle physiology such as the patient in the vignette, may not manifest pulsus paradoxus in the setting of cardiac tamponade or airway obstruction, regardless of the severity of these clinical entities because of the absence of the phenomenon of ventricular interdependence. In patients with single ventricle and Fontan physiology, it has been reported that there is an inverse relationship between the mean airway pressure and cardiac output when these patients are receiving positive pressure ventilation. The ventilatory settings should be adjusted to maintain the lowest mean airway pressure that will be associated with the most optimum lung volume and the most desirable gas exchange. Evidence continues to accumulate that show that improving coronary and cerebral perfusion pressures during cardiopulmonary resuscitation (CPR) improve survival and neurological outcome. Patients with a single ventricle and cavopulmonary anastomosis have a number of cardiopulmonary characteristics that interfere with accomplishing these goals. For example, during the compression phase of CPR, there is no compressible chamber between the systemic venous return and the pulmonary vascular tree. This renders the compression and release mechanism of CPR less effective and the venous blood from the pulmonary arteries can potentially flow backward from the chest into the abdomen. On the other hand during the decompression phase of the CPR when the chest wall is allowed to recoil and there is a transient reduction in intrathoracic pressure, systemic venous return from the vena cava must passively cross the pulmonary vascular bed and return to the common atrium and then the common ventricle. This latter phenomenon also introduces limitations on the efficiency of chest compression during CPR in patients with single ventricle physiology. Hypercarbia and acidosis that prevail during CPR are recognized to increase pulmonary vascular resistance further limiting pulmonary blood flow. These pathophysiological mechanisms may explain the limited effectiveness of CPR and high prevalence of neurological complications and mortality in patients with single ventricle who require CPR. PREP Pearls Patients with single ventricle and Fontan circulation are more sensitive to volume depletion. Patients with single ventricle and Fontan circulation do not manifest pulsus paradoxus in the setting of cardiac tamponade. When mechanically ventilated with positive pressure breaths, patients with Fontan circulation should received the lowest mean airway pressure that will provide optimum lung expansion in order to avoid the deleterious effects of positive pressure ventilation on cardiac output. ABP Content Specifications(s)/Content Area Understand the pathophysiology of right or left single ventricle Suggested Readings Jolley M, Colan SD, Rhodes J, DiNardo J. Fontan physiology revisited. Anesth Analg. 2015;121(1):172-182. doi: 10.1213/ANE.0000000000000717 Lok JM, Spevak PH, Nichols DG. Tricuspid atresia. In: Nichols DG, Ungerleider RM, Spevak PH, eds. Critical Heart Disease in Infants and Children. 2nd Ed. Philadelphia, PA: Mosby;2006:799-822. Park JB, Song IK, Lee JH, et al. Optimal compression position for patients with a single ventricle during cardiopulmonary resuscitation. Pediatr Crit Care Med. 2016;17(4):303-306. doi: 10.1097/PCC.0000000000000658

A previously healthy 12-year-old boy is being managed in the pediatric intensive care unit after sustaining a traumatic brain injury with resultant diffuse axonal injury to the midbrain and hypothalamus. On hospital day 3, new-onset seizure activity develops despite prophylactic anticonvulsive therapy. The patient is afebrile, capillary refill time is 2 to 3 seconds, and oxygen saturation by pulse oximetry is 97% on low ventilatory support. The results of neurologic imaging are unchanged. Laboratory studies reveal the following values: Laboratory Test Result Serum sodium 118 mEq/L (118 mmol/L) Potassium 3.8 mEq/L (3.8 mmol/L) Chloride 96 mEq/L (96 mmol/L) Bicarbonate 27 mEq/L (27 mmol/L) BUN 31 mg/dL (11 mmol/L) Creatinine 0.4 mg/dL (35.3 µmol/L) Glucose 112 mg/dL (6.2 mmol/L) Serum osmolality 253 mOsm/kg (253 mmol/kg) On review of the patient's chart, you note that his sodium level was normal that morning, and his urine output has dropped to 0.6 mL/kg/h over the past few hours. After administration of hypertonic saline, seizures resolve. Of the following, the BEST next step in management is to A. administer intravenous hydrocortisone B. increase the anticonvulsive therapy dose C. restrict fluids D. start a vasopressin infusion

C. restrict fluids The patient in the vignette has symptomatic hyponatremia secondary to syndrome of inappropriate antidiuretic hormone secretion (SIADH). Fluid restriction is the mainstay of treatment of this condition after stabilization and management of acute symptoms. SIADH is one of the most common water and electrolyte disorders encountered in pediatric critical care, and the associated metabolic derangements can be life-threatening if not recognized and managed promptly. Affected patients manifest with hyponatremia in the setting of decreased urine output, euvolemia, and normal total body sodium. Serum sodium level is often less than 120 mEq/L (120 mmol/L), plasma osmolality is low, urine sodium level and osmolality are elevated, and urine output is less than 1 mL/kg/h. Physical findings vary based on the cause of SIADH and the severity of hyponatremia, but euvolemia (or slight hypervolemia) is the rule, and volume status can be crucial in distinguishing this condition from other causes of hyponatremia. The following is a list of clinical criteria for SIADH: Hyponatremia (serum sodium < 135 mEq/L [135 mmol/L]) Decreased plasma osmolality (< 280 mOsm/kg [280 mmol/kg]) Elevated urine sodium (> 20 mEq/L [20 mmol/L]) Elevated urine osmolality (> 100 mEq/L [100 mmol/L]) Clinical euvolemia or slight hypervolemia Absence of other causes of hyponatremia Under normal physiologic conditions, antidiuretic hormone (ADH) is synthesized in the hypothalamus and secreted from the posterior pituitary gland to maintain water homeostasis. This balance is achieved by the regulation of free water excretion from the kidneys. Changes in serum solutes and effective circulating volume are sensed by osmoreceptors and baroreceptors, respectively, and when osmolality is driven outside an individual's set range, ADH secretion changes accordingly. Increased serum osmolality leads to an increase in ADH secretion, which stimulates V2 receptors in the renal collecting duct. This results in a cascade of events culminating in the reabsorption of free water and the restoration of water homeostasis. SIADH is characterized by the continuous secretion or action of ADH despite low serum osmolality and normal or increased plasma volume, resulting in inappropriate water reabsorption. While volume expansion might be expected, patients remain euvolemic because the increase in extracellular free water downregulates aldosterone secretion, thus diminishing sodium reabsorption in the kidneys. The mild natriuresis that ensues prevents major volume expansion and is the reason for high urine sodium levels in patients with SIADH. Conditions associated with SIADH include the following: Central nervous system disorders (eg, trauma, hydrocephalus, demyelinating disease) Infections (eg, meningitis, encephalitis, brain abscesses, pneumonia) Pulmonary processes (eg, cystic fibrosis, pneumothoraces, asthma, effects of positive pressure ventilation) Malignancies (eg, brain tumors, lymphoma, carcinomas) Exposure to certain drugs (eg, chemotherapeutic agents, anticonvulsant medications) Complications from SIADH are primarily neurologic and stem from the brain's inability to adapt to changes in sodium and osmolality. These manifestations are directly related to the severity of hyponatremia and the rate at which it develops. Complications are rare when the sodium level is greater than 125 mEq/L (125 mmol/L). Patients with acute onset of hyponatremia or a sodium level lower than 120 mEq/L or both are more likely to present with life-threatening symptoms, including altered mental status, lethargy, generalized seizures, and coma. Treatment depends on the degree and rapidity of hyponatremia, whether the patient is symptomatic, and the underlying cause. Symptomatic hyponatremia is a true emergency, and immediate correction with hypertonic saline is recommended if patients display neurologic symptoms, or if the serum sodium level is less than 125 mEq/L (125 mmol/L) regardless of symptoms. Suggested interventions include delivering 3 to 5 mL/kg of 3% saline as an IV bolus or more slowly at an infusion rate of 1 mL/kg/h. Whether given as a bolus or infusion, 1 mL/kg of 3% saline will raise the serum sodium level approximately 1 mEq/L (1 mmol/L). Ideally hypertonic saline should be delivered through a central vein but can be delivered through peripheral or intraosseous access if necessary. Treatment should not be delayed while central venous access is attained. Once a patient is stabilized, management should include a strategy for slow correction and maintenance of sodium and water balance. Because SIADH is a disorder of inappropriate water reabsorption, the only way to restore homeostasis is by fluid restriction. If fluid restriction does not correct hyponatremia, the diagnosis of SIADH should be reconsidered. The patient in the vignette has life-threatening complications of SIADH, including severe hyponatremia with seizures. Hemodynamic stabilization and emergent correction of hyponatremia with hypertonic saline is indicated. Following stabilization, the patient will ultimately require management with a fluid restriction strategy tailored to slowly restore sodium and fluid balance. In the absence of other causes of hyponatremia, adrenal insufficiency should be considered, and ultimately treated with hydrocortisone. However, this condition is generally associated with hyperkalemia and hypoglycemia. Vasopressin infusion would be indicated for treatment of central diabetes insipidus, not SIADH. Anticonvulsant medications are not indicated for seizures secondary to hyponatremia. PREP Pearls Syndrome of inappropriate antidiuretic hormone is a frequent cause of hyponatremia in critically ill children and should be considered in any susceptible patient with hypotonic hyponatremia and euvolemia. Acute management of syndrome of inappropriate antidiuretic hormone should include correction of symptomatic hyponatremia, and ultimately fluid restriction with diligent attention to volume status and electrolyte monitoring. Hyponatremia of any cause is associated with increased morbidity and mortality in the pediatric intensive care unit. Affected patients with neurologic symptoms including altered mental status or seizures should be treated with hypertonic saline without delay. ABP Content Specifications(s)/Content Area Recognize SIADH as a cause of hyponatremia Understand the mechanism of hyponatremia during SIADH Plan appropriate therapy for the life-threatening complications of hyponatremia due to SIADH Understand the pathophysiology of sodium and water balance in a patient with SIADH Recognize seizure as a complication of SIADH Suggested Readings Lynch RE, Wood EG. Fluid and electrolyte issues in pediatric critical illness. In: Fuhrman BP, Zimmerman JJ, eds. Pediatric Critical Care. 4th ed. Philadelphia, PA: Elsevier; 2011:944-948. Moritz ML, Ayus JC. Disorders of water metabolism in children: hyponatremia and hypernatremia. Pediatr Rev. 2002;23(11):371-380. Wang J, Xu E, Xiao Y. Isotonic versus hypotonic maintenance IV fluids in hospitalized children: a meta-analysis. Pediatrics. 2014;133(1):105-113. doi:10.1542/peds.2013-2041.

A 4-month-old male infant is admitted to the pediatric intensive care unit (PICU) from an outside emergency center after presenting with seizures and respiratory distress. The mother says the baby took a bottle well, and she left to get groceries. Her boyfriend called her an hour later because the infant rolled off the couch. He noticed the boy appeared to be choking, turned blue, and was making a funny gurgling sound. On admission to the PICU, the infant is intubated, and vital signs reveal a heart rate of 184 beats/min, blood pressure of 92/45 mm Hg, and temperature of 34.6°C. He appears pale. Physical examination reveals minimally reactive pupils that are 6 mm and symmetrical. He is unresponsive to noxious stimulation. His capillary refill time is 4 seconds. The abdomen is soft and distended. Laboratory results are pending. The chest radiograph shows posterior rib fractures. Of the following, which constellation of findings is MOST likely in this infant? A. epidural hematoma, clavicular fracture, and retinal hemorrhages B. epidural hematoma, diffuse axonal injury, and retinal hemorrhages C. subdural hemorrhage, cerebral edema, and retinal hemorrhages D. subdural hemorrhage, simple long bone fractures, and conjunctival petechiae

C. subdural hemorrhage, cerebral edema, and retinal hemorrhages Nonaccidental head trauma continues to result in substantial morbidity and mortality in infants and children under 2 years of age. The incidence of abusive head trauma, commonly known as shaken baby syndrome, is between 14 and 40 cases per 100,000 infants younger than 1 year of age. A slight male predominance is noted in patients who sustain abusive head trauma. The American Academy of Pediatrics encourages use of the term abusive head trauma to describe the clinical findings and not the mechanism of injury that occurs in this patient population. Many survivors of abusive head trauma have suffered prior abuse and lack signs of external trauma. In less severe cases, injuries may go unrecognized because of the diverse presentation of symptoms. Nonspecific symptoms such as poor feeding, vomiting or gastrointestinal issues, irritability, seizures, altered mental status, and fever are common. In severe cases, children present with life-threatening complications, including manifestations of cerebral edema with intracranial hypertension, respiratory depression, coma, and cardiopulmonary arrest, with no explanation for the severity of the symptoms. The caretaker frequently provides an inconsistent and changing medical history. Treatment delay is common because the perpetrator may believe that shaking resolved the problem or may hope the child will recover without medical care. This delay in treatment results in a prolonged hypoxic insult, with cerebral edema contributing to the poor neurologic outcome associated with abusive head trauma. Bruising in any nonambulatory child should raise suspicion of abusive trauma and prompt investigation. Abusive head trauma results from severe acceleration-deceleration forces when the head of the child moves back and forth in a violent manner as the perpetrator abuses the child. Impact injury occurs in some cases of abusive head trauma as well. Rotational forces exerted on the brain result in shear injury. Infants are at greatest risk of sustaining injury related to abusive head trauma because their large head and weak neck muscles allow substantial movement of the head when they are violently shaken. Additionally, the lack of myelination and higher water content of the infant brain further increases the risk of shear injury from acceleration-deceleration forces. Linear and rotational forces applied to the brain from violent shaking stretch and tear bridging vessels, causing subarachnoid and subdural hemorrhages that are common intracranial findings on imaging studies. The presence of subarachnoid or subdural hemorrhage is strongly associated with abusive head trauma. Cerebral edema is commonly noted on imaging studies because of hypoxic ischemic injury associated with treatment delay for these children. Cervical spine injuries can occur because the infant's weak neck muscles cannot adequately support the disproportionately large head as acceleration and deceleration forces occur during violent shaking. Imaging studies may reveal ligamentous injury or cord injury such as axonal damage to the cervical spine. Accidental head injury is associated with more common neuroimaging findings, including epidural hematoma, intraparenchymal injury, and skull fractures. Retinal hemorrhages are commonly seen in many, but not all cases of abusive head trauma. Retinal hemorrhages have been reported in children with aneurysms, arteriovenous malformations, arachnoid cysts, severe trauma, vasculitis, coagulation disorders, leukemia, infection, and metabolic diseases. Retinal hemorrhages that can occur during birth (more common with vaginal delivery) typically resolve in 1 month. Retinal hemorrhages associated with abusive head trauma are the result of shearing forces resulting in vitreoretinal traction. The diffuse hemorrhagic retinopathy associated with abusive head trauma involves multiple layers of the retina extending to the ora serrata. This diffuse hemorrhagic retinopathy pattern is commonly seen in abused children. Splitting of the retinal layers (retinoschisis) and optic nerve sheath hemorrhage have also been noted in survivors of abusive head trauma. Involvement of a pediatric ophthalmologist is strongly recommended to document the number and extent of retinal hemorrhages in these patients. Skeletal injury, specifically rib fractures and long bone fractures, are associated with abusive head trauma in children. Rib fractures tend to be posterior and lateral and are the result of the child's chest being squeezed during shaking or slamming. Rib fractures have been noted in children who received closed chest compressions during cardiopulmonary resuscitation; however, these rib fractures tend to be anterior. The presence of healing rib fractures is strongly associated with abusive trauma requiring investigation. Acute rib fractures may be difficult to visualize on initial radiographs. A follow-up skeletal survey should be performed on children in 2 to 3 weeks. The more commonly identified long bone fractures associated with abusive head trauma are diaphyseal fractures of the humerus and femur. Skull fractures can occur with accidental or nonaccidental trauma. In cases of nonaccidental trauma, impact injury from slamming the child's head into a stationary object can result in a skull fracture. Midline fractures, occipital fractures, multiple complex fractures, and diastatic and depressed skull fractures are more suggestive of abuse if there is no associated history of accidental trauma. Blunt trauma to the abdomen can occur, resulting in visceral injury. Coagulation disturbances have also been reported in patients with abusive head trauma. Mandated reporting by health care providers who suspect abuse should not be delayed. The delay involving law enforcement and investigative personnel can result in loss of evidence needed to prosecute the perpetrator. Involvement of child abuse specialists or a hospital forensic team is strongly recommended. Abusive head trauma is a preventable injury, and efforts to educate parents and the public are key factors to reduce the number of children who become victims of abuse. PREP Pearls Abusive head trauma should be considered in any child for whom injuries are inconsistent with the provided history. Bruising in any nonambulatory child should raise the suspicion of abuse. ABP Content Specifications(s)/Content Area Understand the pathogenesis and pathophysiology of shaken impact injury Recognize the typical physical findings in a patient with shaken impact injury Suggested Readings American Academy of Pediatrics Committee on Child Abuse and Neglect. Shaken baby syndrome: rotational cranial injuries-technical report. Committee on Child Abuse and Neglect. Pediatrics. 2001;108(1):206-210. doi:10.1542/peds.108.1.206. Chiesa A, Duhaime AC. Abusive head trauma. Pediatr Clin North Am. 2009;56(2):317-331. doi:10.1016/j.pcl.2009.02.001. Christian CW, Block R; Committee on Child Abuse and Neglect, American Academy of Pediatrics. Abusive head trauma in infants and children. Pediatrics. 2009;123(5):1409-1411. doi:10.1542/peds.2009-0408. Greeley CS. Abusive head trauma: a review of the evidence base. AJR Am J Roentgenol. 2015;204(5):967-973. doi:10.2214/AJR.14.14191. Hinds T, Shalaby-Rana E, Jackson AM, Khademian Z. Aspects of abuse: abusive head trauma. Curr Probl Pediatr Adolesc Health Care. 2015;45(3):71-79. doi:10.1016/j.cppeds.2015.02.002. Levin AV. Retinal hemorrhage in abusive head trauma. Pediatrics. 2010;126(5):961-970. doi:10.1542/peds.2010-1220.

The pharmacokinetic characteristics of a new antibiotic are being studied to understand how it is eliminated from the body. After a single intravenous dose is administered to a volunteer, serial measurements of the drug's plasma concentration are made, as shown below. Time zero 2 hours 4 hours 6 hours 8 hours 100 ng/mL 50 ng/mL 25 ng/mL 12.5 ng/mL 6.25 ng/mL According to these data, which of the following would you conclude is true? A. a curve plotted with the absolute concentration of the drug on the y-axis and time on the x-axis would be linear B. a curve plotted with the logarithmic concentration of the drug on the y-axis and time on the x-axis would be nonlinear C. the drug's elimination follows first-order kinetics D. the drug's elimination follows zero-order kinetics

C. the drug's elimination follows first-order kinetics The concentration of the drug is declining by 50% of the previous concentration at each 2-hour interval. This suggests that this drug follows first-order kinetics, where the elimination is proportional to the drug concentration. Elimination is greater when the concentration is higher. The half-life remains constant, no matter how high the concentration of the drug. In first-order kinetics, the elimination processes are not saturated, and the body can adapt to the concentration of the drug. A curve plotted with the logarithmic concentration of the drug on the y-axis and time on the x-axis would be linear. More than 95% of the medications used in clinical practice follow first-order kinetics (Figure). In contrast, with zero-order kinetics, the elimination process is saturated. Only a fixed amount of the drug can be eliminated per unit time, independent of the drug concentration. In this case, a curve plotted with the absolute concentration of the drug on the y-axis and time on the x-axis would be linear. An example of zero-order kinetics is ethanol metabolism. The elimination system gets quickly saturated, and once this has happened, only a fixed amount of ethanol can be cleared per unit time. Zero-order kinetics is also called capacity-limited elimination or Michaelis-Menten elimination. PREP Pearls A fixed proportion of the drug is eliminated per time interval in first-order kinetics. Most medications used in clinical practice follow first-order kinetics. ABP Content Specifications(s)/Content Area Understand changes in serum drug concentration with drug elimination Know the effects of drug kinetics on drug serum concentrations as a function of time Suggested Readings Power BM, Forbes AM, van Heerden PV, Ilett KF. Pharmacokinetics of drugs used in critically ill adults. Clin Pharmacokinet. 1998;34(1):25-56. doi:10.2165/00003088-199834010-00002. Tripathi KD. Essentials of Medical Pharmacology. London, UK: JP Medical Ltd; 2013. Zuppa A. Pharmacology. In: Shaffner DH, Nichols DG, eds. Rogers' Textbook of Pediatric Intensive Care. 5th ed. Philadelphia, PA: Wolters Kluwer Health; 2015:chap 22.

Emergency medical services arrive to the emergency department (ED) with an 8-year-old girl following a motor vehicle collision. She was sitting, restrained, in the rear row. Her vehicle was "T-boned" on her side of the vehicle and she required extraction by emergency medical service. Initial Glasgow Coma Scale score was 13 at the scene. She was given 1 L of intravenous saline and given oxygen provided transport. On arrival to the ED, her vital signs are temperature 35°C, heart rate 120 beats/min; respiratory rate 35 breaths/min, blood pressure 96/50 mm Hg, SpO2 100% on 5 L/min face mask. There are multiple bruises on her face, trunk, right arm, and right leg. She is following commands, her pupils are equally round and reactive to light, and she complains of a headache. There is scalp bogginess over her right ear. Cardiac examination is remarkable for tachycardia, but without gallop or rub. Her distal pulses are 1+. Breath sounds are equal bilaterally. Her abdomen is soft and nondistended. There is no rebound tenderness. There is no pelvic instability and there are no obvious limb deformities. The patient quickly becomes obtunded and requires emergent endotracheal intubation to protect her airway. She is rushed to the computed tomography scanner while being manually ventilated. Computed tomography of the head without contrast shows a small right epidural hematoma with 1 mm midline shift without cerebral edema or signs of herniation. As neurosurgery is evaluating the patient, she becomes acutely hypotensive and more tachycardic. The SpO2 decreases to 87% while she is being manually ventilated on FiO2 = 1.0. Pupils remain equally round and reactive to light. Of the following, the MOST appropriate immediate treatment is: A. closed reduction of a femur fracture B. emergent drainage of an epidural hematoma C. exploratory laparotomy to evaluate for intra-abdominal bleeding D. needle decompression thoracostomy

D. needle decompression thoracostomy The most likely cause of this patient's hypotension is a tension pneumothorax most likely due to excessive pulmonary pressures and volumes while bagging. A systematic review of tension pneumothoraces reported that up to 50% of them occurred due to barotrauma in mechanically ventilated patients. Cardiovascular changes associated with tension pneumothorax during mechanical ventilation included tachycardia (30%) and hypotension (66%) with approximately 30% of patients developing cardiac arrest. For patients on mechanical ventilation, there was a wide range of the incidence of hypoxemia among the included studies (11%-91%), with median PaO2/FiO2 ratio of ~73. This patient is receiving 100% oxygen while bagging, therefore minimizing the degree of hypoxemia. Diagnosis of tension pneumothorax can be made on physical examination alone with unilateral decrease of air entry, tympany on percussion, and tracheal deviation. However, these physical examination findings may not be easily detectable, especially in emergent or chaotic situations. Chest radiographs and/or chest ultrasound are used as confirmatory tests. In patients with tension pneumothoraces, hypotension occurs due to the increase in intrathoracic pressure. This leads to a decreased venous return to the right atrium, leading to decreased preload and cardiac output. For patients who are receiving mechanical ventilation, the effect of sedative/analgesic agents may blunt the cardiovascular response to obstructive shock and lead to worsening of hypotension. Treatment of tension pneumothoraces follows the obstructive shock pathway, including intravascular volume expansion, supplemental oxygen, and needle decompression of the chest. Rapid treatment is warranted, which often precludes waiting for confirmatory imaging if the clinical suspicion is high. Needle decompression is the classic emergent management and involves placement of a large-bore needle/catheter into a sterilely prepared site at the second intercostal space in the midclavicular line. A gush of air may be heard if a tension pneumothorax was decompressed with improvement in heart rate, blood pressure, and oxygen saturations. Although needle decompression can be life saving, there are inherent risks of the procedure, such as injury to large vessels (subclavian artery/vein) and the lungs. The intracranial hematoma is not likely the cause of hypotension, because increased intracranial pressure typically leads to hypertension early in the course (Cushing's triad). This patient has a documented epidural hematoma that should be addressed. However, her current hemodynamic compromise should first be addressed. Laparotomy is not indicated since her abdominal examination is benign and not the most likely source of her acute decompensation. Finally, there are no indications of a large fracture in her pelvis or long bones. PREP Pearls Tension pneumothorax can present as acute hypotension, especially during or after emergent endotracheal intubation. Hypoxemia may not be readily apparent in patients with pneumothoraces who are mechanically ventilated. ABP Content Specifications(s)/Content Area Recognize, diagnose, and plan treatment for a tension pneumothorax Suggested Readings Roberts DJ, Leigh-Smith S, Faris PD, et al. Clinical presentation of patients with tension pneumothorax: a systematic review. Ann Surg. 2015;261(6):1068-1078. doi: 10.1097/SLA.0000000000001073

A 9-year-old girl presents to the emergency department after experiencing a generalized tonic-clonic seizure. She is treated with lorazepam, undergoes intubation for airway protection, and is admitted to the pediatric intensive care unit. Computed tomography findings of her brain are normal. Comprehensive metabolic panel and complete blood cell counts are normal. Her family reports that she has a history of attention-deficit disorder. They also report that she had little sleep the night before because she was studying for school examinations. History reveals that the patient often experiences "whole body jerks" on awakening and frequently drops objects. The next morning she undergoes extubation without difficulty. Her neurologic examination findings are normal with the exception of extreme photosensitivity. While awaiting transfer to a general pediatric service, you observe a brief series of rapid tonic clonic movements of her upper extremities without significant alteration of her mental status. Of the following, the BEST medication that is indicated is A. carbamazepine B. lamotrigine C. phenytoin D. sodium valproate

D. sodium valproate Seizures are a common admitting diagnosis to the pediatric intensive care unit. Seizures can generate a number of different clinical presentations depending on the location of the unopposed neuronal excitation. This case demonstrates a classic description and presentation of juvenile myoclonic epilepsy. Myoclonic seizures present as a series of sudden, brief, shock-like involuntary movements. This should be distinguished from the persistent repetitive tonic-clonic muscle movements of status epilepticus. Myoclonic epilepsy can be focal or multifocal. Seizures can be subtle, resembling tremors, or violent. They can occur spontaneously or be induced. Juvenile myoclonic epilepsy is a common variant of myoclonic epilepsy. This syndrome was originally described in 1899 in a physician's son. In 1957, it was officially termed impulsive petit mal. It was reclassified again in 1989 and given the term juvenile myoclonic epilepsy. In 2012, international consensus defined juvenile myoclonic epilepsy as "myoclonic jerks without loss of consciousness predominantly occurring after awakening." Juvenile myoclonic epilepsy usually starts in adolescence and presents as sudden brief involuntary movement of the upper extremities. These seizures may make the patient drop or throw something, especially during morning hygiene. Juvenile myoclonic epilepsy can also involve the diaphragm, causing the patient to make an involuntary noise or gasp. Juvenile myoclonic epilepsy prevalence is difficult to determine, with estimates ranging from 2% to 12% of all epilepsy diagnoses. It is often precipitated by sudden awakening, lack of sleep, or excessive alcohol use. Interestingly, patients often complain of photosensitivity. In addition, a third of patients have a history of mood, anxiety, or mental health disorders. Investigators believe there is a genetic predisposition toward juvenile myoclonic epilepsy. To date, genetic linkage studies have identified 15 loci that are repeatedly identified in the juvenile myoclonic epilepsy population. The most common involves the Myclonin 1/EFHC1 domain found in roughly 9% of patients with juvenile myoclonic epilepsy. Mutations here are thought to affect cortical development leading to epileptogenic foci. Electroencephalography typically shows fast, rhythmic spike, and wave pattern with frontocentral predominance. There is typically a burst of 5 to 20 spikes, followed by a slow wave pattern (Figure). Magnetic resonance spectroscopy suggests that the origin of the neuronal excitability is in the frontal lobe and spreads to the thalamus. Juvenile myoclonic epilepsy can progress to tonic-clonic seizure activity, as in the vignette, or more rarely, absence seizures. Treatment of juvenile myoclonic epilepsy involves both modification of lifestyle choices and use of anticonvulsant medications. Patients should be counseled to avoid fatigue, excess alcohol use, sleep deprivation, and emphasize seizure precautions in the period following awakening. The antiepileptic drug sodium valproate is considered the drug of choice in juvenile myoclonic epilepsy, with a clinical success rate of over 80%. Some anticonvulsants can actually worsen the myoclonus and should be avoided. These include sodium channel-blocking anticonvulsants such as phenytoin and carbamazepine. Lamotrigine has been used as monotherapy for juvenile myoclonic epilepsy. Its use is somewhat controversial, with some authors stating it is as effective as valproate, while others report myoclonic seizure exacerbation. Lamotrigine may have a role in the treatment of juvenile myoclonic epilepsy in women interested in becoming pregnant, because it appears to be less teratogenic to the developing fetus than valproate. PREP Pearls Juvenile myoclonic epilepsy begins in adolescence, frequently involves the upper extremities, and has no alteration of consciousness. Sleep deprivation, fatigue, and alcohol use can exacerbate juvenile myoclonic seizures. Anticonvulsants such as phenytoin and carbamazepine can worsen the myoclonic seizure activity seen in juvenile myoclonic epilepsy and should be avoided. ABP Content Specifications(s)/Content Area Recognize myoclonic seizures Suggested Readings Baykan B, Wolf P. Juvenile myoclonic epilepsy as a spectrum disorder: a focused review. Seizure 2017;49:36-41. doi: 10.1016/j.seizure.2017.05.011 Striano P, Belcastro V. Treating myoclonic epilepsy in children: state of the art. Exp Opin Pharmacother. 2013;14(10):1355-1361. doi: 10.1517/14656566.2013.800045 Yacubian EM. Juvenile myoclonic epilepsy: challenges on its 60th anniversary. Seizure. 2017;44:48-52. doi: 10.1016/j.seizure.2016.09.005


Kaugnay na mga set ng pag-aaral

APUSH Chapter 1, APUSH Chapter 2, APUSH Chapter 3, APUSH Chapter 4, APUSH Chapter 5

View Set

Intro to Astronomy - Chapter 1 Lecture Notes

View Set

Women, Gender and Sexuality Studies Exam 1

View Set

Real Estate Principals 1 - Unit 12

View Set

Ethical and Legal Issues in Psychiatric Nursing

View Set

The Constitution: Principles of the Constitution.

View Set

Philosophy 110: morality/ethics 5.1

View Set